HESI Med Surg Combo

Ace your homework & exams now with Quizwiz!

Medical management for a Hemorrhagic stroke

keep patient calm, sedated, with oxygen, don't want any intracranial pressure so keep patient stress free with no straining (val salva), use stool softener, catheter, gentle repositioning, continuous monitoring

person to assess first

kids that have wheezing

Oliguric - Anuric phase of acute renal failure

lasts 1-8 weeks, the longer this phase lasts the poorer the prognosis

Anuria?

less than 100 ml/24 hrs

After LP client need what positioning

lie flat for 3 hours

Triaging patients

low BP 84/48

broken femur assess pt for

measles like red spots on chest and neck

What nursing action is necessary for the client with a flail chest?

. Encourage coughing and deep breathing.

Assistive devices

...

Coumadin

...

Plavix

...

fetal heart rate

110-160

If a patient has a positive Chvostek's sign what is a possible problem that could arise?

Airway ineffectiveness

How are renal injuries usually treated (Except for a clot)?

Conservatively (not rushed to surgery)

How many months between abd pain and diagnosis of renal cancer?

14 months

When should levothyroxine (Synthroid) be administered?

In the morning, 2 hours away from any other drug

Serosanguineous Drainage

Thin and red and is composed of serum and blood

Anorexia Nervosa

Consume 50% of all meals. An outcome statement should be measurable and provide observable behaviors that indicate the clients problem is resolving. Risk for hypokalemia

The nurse is caring for a client with VRE. The nurse should place the client on

Contact precaution

Pt taking Kava to help them sleep at night, what other drug would you be concerned about if they was taking

Levidopa

Treatment for Addison's disease?

Lifetime replacement of corticosteroids, patient carries an injection kit of dexamethasone. Also, encourage fluids and foods high in sodium

Transilluminator

Light source that helps detect the presence of fluid in the sinus cavities

Delirium

Occurs in response to a specific stressor: * Infection * Drug reaction * Substance intoxication or withdrawal. * Electrolyte imbalance. * Head trauma * Sleep deprivation

trigger

a substance, object, or agent that initiates or stimulates an action; in asthma, any stimuli that initiates the IgE

tracheotomy

a surgical incision into the trachea for the purpose of establishing an airway; performed below a blockage by a foreign body, tumor, or edema of the glottis.

16 years old that is pregnant

assess how the patient feels

Gonometer

assess the degree of joint flexion and extension.

Reflex Hammer

assesses deep tendon reflexes

If fluid is in the tissue rather than in the vascular space and there is decreased peripheral vascular resistance what is needed?

A fluid bolus

Cerebrovascular Disorders

A functional abnormality of the central nervous system (CNS). Occurs when the normal blood supply to the brain is disrupted

Twelve hours after chest tube insertion for hemothorax, the nurse notes that the client's drainage has decreased from 50 ml/hr to 5 ml/hr. What is the best initial action for the nurse to take?

Assess for kinks or dependent loops in the tubing.

The nurse receives the client's next scheduled bag of total parental nutrition (TPN) labeled with the additive NPH insulin. What action should the nurse implement?

return solution to pharmacy

chest percussion

rhythmic percussion of a patient's chest with cupped hands to loosen retained respiratory secretions.

Signs and Symptoms of Ischemic stroke all depend on _______________________, __________________ & ___________________.

Location of the obstructed vessel, size of the area of inadequate perfusion, & amount of collateral blood flow.

Treatment for Pulmonary Edema?

Lots of lasix, morphine, must have a foley

Why will dialysis get stopped?

Low BP, sudden severe headache, decreased level of consiousness

Ascending urinary reflux

Occurs when normal ureteral peristaltic pressure is met with an increase in urinary pressure occurring during bladder filling if the urinary bladder neck is obstructed. large residual occurs with this.

Complications associated with ARF

Pericarditis (fluid backs up), Tachy, fever, friction rub, seizures (due to increased BUN), anemia (due to bleeding from trauma)

Types of dialysis

Peritoneal (continuous and automated), hemodialysis

What is the highest priority nursing dx for clients in any type of renal failure?

Risk for imbalanced fluid volume.

Which statement reflects the highest priority nursing diagnosis for an older client recently admitted to the hospital for a new-onset cardiac dysrhythmia?

Risk for injury related to syncope and confusion

Medications during pregnancy

SAFE: * Prenatal vitamins * Tylenol UNSAFE: * Aspirin * Ibuprofen * Naproxen * Most cold medicines * Herbal medicines

Disorder of the pituitary gland that causes excess ADH hormone?

SIADH

Mannitol: Side Effects

SIDE EFFECTS: * Fluid & electrolyte imbalance. * Pulmonary edema from rapid shift of fluids. * N/V * Tachycardia from rapid fluid loss. * Acidosis. ADVERSE EFFECTS: * Disorientation * Confusion * Headache * Convulsions

Addison's disease priority intervention?

Safety

If a nurse is asked to adjust the settings on a ventilator what should the nurse do?

Say no, call respiratory therapy

he nurse includes frequent oral care in the plan of care for a client scheduled for an esophagogastrostomy for esophageal cancer. This intervention is included in the client's plan of care to address which nursing diagnosis?

risk for infection

While performing an assessment of a 3 year old client, the nurse notes bruises in various stages of healing on the concealed surfaces of the body. What action should the nurse take next

Contact the local reporting agency for suspected child abuse

The nurse is assessing a 5 month old. what is an expected finding

roll from ABD to back

Normal findings when assess patellar reflexes

the leg will extend

S/S of ICP

* Increased BP * Widened pulse pressure * Increased temp.

The nurse in a health clinic is reviewing prescriptions medications with several clients. It would be a priority for the nurse to follow up with the client who states

" I crush my verapamil (calan sr) to make it easier to swallow

The nurse has attended a staff development conference on Meniere's disease. Which of the following statements, if made by the nurse would require a follow up

"Assistive listening devices are required for these clients."

Pheochromocytoma: S/S

* Paroxysmal or sustained hypertension. * Severe headaches. * Palpitations * Flushing & profuse diaphoresis. * Pain in the chest or abdomen with nausea & vomiting. * Heat intolerance * Weight loss * Tremors * Hyperglycemia

Broca's

"Broken" speech, affects formation of words, frontal lobe (lower part), expressive aphasia. Slowed/slurred/broken speech, brain is working

Which instruction should the nurse teach a female client about the prevention of toxic shock syndrome?

"Change your tampon frequently."

The nurse is attending a staff development conference on medical treatment for various neurologic disorders. Which of the following statement would require follow up

"Clients with MS often receive immunoglobulin G (IVIG)."

An older female client with dementia is transferred from a long-term care unit to an acute care unit. The client's children express concern that their mother's confusion is worsening. How should the nurse respond?

"Confusion in an older person often follows relocation to new surroundings."

The nurse is completing an admission interview for a client with Parkinson disease. Which question will provide additional information about manifestations the client is likely to experience?

"Have you ever been 'frozen' in one spot, unable to move?

The nurse in a community health clinic is talking with the parent of a child with celiac disease. Which statement would require follow up by the nurse

"I can feed my child oatmeal and eggs for breakfast."

The nurse working on a maternity unit has become aware of the following client situation. It would be priority for the nurse to intervene if a client states

"I cannot breast feed because my nipples are cracked and sore."

pt is going through anger stage of grief when they say

"why is god doing this to me"

ABGs: (a)-acid (b)-base

* (a) 7.35-7.45 (b) pH * (b) 35-45 (a) pCO2 (Resp) * (a) 22-26 (b) HCO3 (Metab) * 80-100 pO2

Disseminated Intravascular Coagulation (DIC)

* A coagulation disorder, clotting factors used up, inability to form clots. * Hemorrhage occurs. * Is suspected when blood is oozing from 2 or more unexpected sites.

ABC's

* Airway * Breathing * Circulation

What is the diet for Acute Glomerulonephritis?

* Low sodium w/ NO added salt.

Serum Electrolytes

* Mg+ 1.5-2.5 * Phos 2.5-4.5 * K+ 3.5-5 * Ca 8.5-10.9 * Chl 95-105

Chemotherapy: Side Effects

* Neutropenia- Low WBCs. * Thrombocytopenia- Low Platelets. * Anemia- Low RBCs.

DIC: S/S

* Petechiae * Hematomas * Oozing from IV sites, drains, gums, & wounds. * GI & GU bleeding * Hemoptysis (coughing up blood). * Hypotension * Tachycardia * Pain

Recovery of Hemorrhagic Stroke plateaus at _______ months.

18

What percent of blood flow goes through the kidneys?

20-25%

The nurse in the community health setting is assessing the following clients. It would be a priority for the nurse to utilize a multidisciplinary approach for the client who is

21 years old, pregnant, unemployed, and has active TB

Normal Urine output?

400/ml/24 hrs or more

Mitral valve loctation

5th ICS mid clavicular line

Recovery of Ischemic stroke plateaus at _________ months.

6

Age baby can sit without support and hand dominance is evident

7-9 mths

What percent of all kidney tumors are malignant?

85

How many months between onset of hematuria and abd pain for renal cancer?

9 months

Glasgow Coma Scale

< 8 = coma.

Normal Bilirubin level

<1.0

ballottement

A light tap of the examining finger on the cervix causes fetus to rise in the amniotic fluid and then rebound to its original position; occurs at 16-18 weeks

A nurse working in a community health setting is performing primary health screenings. Which individual is at highest risk for contracting an HIV infection?

A 17-year-old who is sexually active with numerous partners

Debilitating anginal pain can be decreased in some clients by the administration of beta-blocking agents such as nadolol (Corgard). Which client requires the nurse to use extreme caution when administering Corgard?

A 47-year-old kindergarten teacher diagnosed with asthma 40 years ago

Cushing's disease

A metabolic disorder characterized by abnormally increased secretion (endogenous) of CORTISOL, caused by increased amounts of ACTH secreted by the pituitary gland.

What is the difference between the nursing diagnosis' Impaired Gas Exchange and Ineffective Airway?

ABG's are necessary for a nursing diagnosis of Impaired Gas Exchange

Stress can cause?

Adrenal insufficiency and Thyroid Storm

When does ventilator pneumonia develop?

After 48 hours

Nursing interventions for Diagnostic testing

Allergies to iodine, any metal in patient, closterphobia

The nurse is caring for a client with w soft tissue injury. The client reports using herbal remedy for 3 weeks prior to seeking health care but can not remember what was taken. The nurse recognize that which of the following herbal remedies can be utilized effectively for soft tissue injuries

Aloe vera

S/S of Hyperthyroidism

Atrial fibrillation, bruit or thrill over the thyroid, Diaphoresis, Palpatations, Tachycardia, Heat Intolerance, Weight loss

Pt with blood sugar of 59

Give D5 over 4 min

Gonorrhea

Gonorrhea in women is often asymptomatic and frquently dismissed or overlooked as a yeast discharge or UTI.

Metroprolol Beta adrenergic receptor antagonist

Can mask signs of hypoglycemia

Hypoglycemia

Cold & clammy, give em candty.

39. The nurse is scheduled to give a dose of ipratropium bromide by metered dose inhaler. The nurse would administer the right drug by selecting the inhaler with which of the following trade names? A. Vanceril B. Pulmicort C. AeroBid D. Atrovent

D. Atrovent The trade or brand name for ipratropium bromide, an anticholinergic medication, is Atrovent.

Ischemic cascade

Each step represents an opportunity for intervention to limit the extent of secondary brain damage caused by a stroke

The nurse is caring for a 72 year old client who was recently DX with breast cancer. The client is expressing feelings of depression and is asking the nurse, "Why me." According to Erikson, which developmental stage is the client experiencing

Ego integrity VS despair

Clincial manifestations of CRF

Electrolyte imbalance which includes weakness, diarhea, abd cramping, hyperkalemia, hypercalcemia, loss of blood pressure regulation, loss of erythropoietin

What do Creatinine and BUN do in ARF?

Increase

Reason for high pressure alarm on a ventilator

Increase bronchial secretions kink in tubing acute bronchospasms tubing is disconnected

A client with congestive heart failure and atrial fibrillation develops ventricular ectopy with a pattern of 8 ectopic beats/min. What action should the nurse take based on this observation?

Increase oxygen flow via nasal cannula.

The nurse is caring for a client who is 1 day post acute myocardial infarction. The client is receiving oxygen at 2 L/min via nasal cannula and has a peripheral saline lock. The nurse notes that the client is having eight PVCs per minute. Which intervention should the nurse implement first?

Increase the client's oxygen flow rate.

T3 and T4 levels in hyperthyroidism are increased or decreased?

Increased

An emaciated homeless client presents to the emergency department complaining of a productive cough with blood-tinged sputum and night sweats. What action is most important for the emergency department triage nurse to implement for this client?

Initiate airborne infection precautions.

Frontal Lobe

Initiation, Planning/Anticipation, Follow through, impulsivity, judgement, reasoning, abstract thinking, smell, motor planning, personality, emotionality, speaking, self monitoring

The nurse is preforming a ABD assessment. Put in order the correct sequence of assessment

Inspection, auscultation, palpation, percussion

Is a TIA ischemic or hemorrhagic?

Ischemic

A female client who received a nephrotoxic drug is admitted with acute renal failure and asks the nurse if she will need dialysis for the rest of her life. The nurse should explain which pathophysiologic consequence that supports the need for temporary dialysis until acute tubular necrosis subsides?

Oliguria

DKA

Only Type 1 diabetics, absence of insulin or inadequate amount of insulin causes ketones to be spilled in the urine, very high Blood sugar level

Phases of acute renal failure

Onset, Oliguric-anuric, diuretic and recovery

What can be done to prevent ventilator pneumonia?

Oral care Q2H

Therapy for stroke patients

Physical - waist down, gross motor skills, walking, transferring, walkers; Occupational - fine motor skills, eating, tying shoes, cognition; Speech - communication

A client is admitted to the hospital with a diagnosis of severe acute diverticulitis. Which nursing intervention has the highest priority

Place the client on NPO status.

Newborn

Postpartum or puerperal infection is any clinical infection of the genital canal that occurs within 28 days after miscarriage, induced abortion, or childbirth. The presence of a fever 24 hours after birth is the first indicator.

Use an ophthalmoscope to watch the clients pupil constrict when a strong light is shown on it.

Pupillary reactivity

The nurse is teaching a class on infant nutrition. The nurse should instruct the parents to introduce

Pureed chicken at 7 months

Therapy often used to treat hyperthyroidism is ____________

Radioactive Iodine or Thyroidectomy

Dehiscence

Seperation of surgical incision

Vulnerable populations

Survivors of violence that occurred at least 5 years ago. Vulnerable populations are those groups who have an increased risk of developing adverse health outcomes. Survivors of violence, even though the violence occurred more than 5 years ago, have an increased risk for adverse health outcomes.

Lasix teaching

avoid quick position changes

Concretization

The inability to abstract with overemphasis on detail

If a high pressure alarm goes off on a mechanical ventilator what needs to be done by the nurse?

The patient needs to be assessed for suction

Perfusion

The process in which blood carries oxygen and important nutrients to body tissues. Perfusion depends on many body compensatory responses, but also on the administration of appropriate fluid volumes to maintain vascular volume.

Hypothalamus controls what?

The release of hormones from the pituitary

Why does a stroke affect many areas of the brain?

The sensory areas in the brain are scattered everywhere

Occipital Lobe

Vision

Then nurse has attended a staff development conference on vitamins and minerals. Which of the following statements if made by the nurse would require follow up

Vitamin B6 supplements are given to help prevent macular degeneration

The nurse should initiate protective precaution for a client who has

WBC 2500 mm3

The infection control nurse is making rounds on a Med - Surg unit. The infection control nurse should intervene if they observe

Wearing a disposable surgical face mask when entering the room of a client with active pulmonary TB

The nurse is teaching a client about crutch walking. Which of the following statements if made by the client indicates a need for the further teaching

When I climb up the stairs I advance my affected leg first, with my crutches."

Renal Failure

When kidneys can no longer maintain the body's internal environment

chemoreceptor

a sensory nerve cell that responds to a change in the chemical composition (PaCO2 and pH) of the fluid around it.

chest physiotherapy

a series of maneuvers including percussion, vibration, and postural drainage designed to promote clearance of excessive respiratory secretions.

Name some Vitamin A foods

Yellow veggies, squash, pumpkin, carrots

cystic fibrosis

an autosomal recessive, multisystem disease characterized by altered function of the exocrine glands involving primarily the lungs, pancreas, and sweat glands.

tuberculosis

an infectious disease caused by Mycobacterium tuberculosis; usually involves the lungs but also occurs in the larynx, kidneys, bones, adrenal glands, lymph nodes, and meninges and can be disseminated throughout the body.

acute bronchitis

an inflammation of the lower respiratory tract that is usually due to infection.

Lanugo hair

anorexia

Pituitary gland is made of 2 parts ...

anterior and posterior

Myasthenia Gravis is treated with short acting _________ drugs.

anticholinesterase

AVMs

arterial venous malformation, occurs when arteries or veins come together improperly and are weaker. When one of these forms, the probability of a hemorrhage increases by 1% every year

Treatment for acute tubular necrosis?

bed rest, correct electrolytes, fluid volume and dialysis

The nurse has received report on 4 clients, who should the nurse assess first

been admitted with Guillian Barre syndrome and has begun plasmapheresis therapy

nasal polyps

benign mucous membrane masses that form slowly in response to repeated inflammation of the sinus or nasal mucosa and project into the nasal cavity.

Ischemic

blood flow is cut off, profusion is lost, possible tissue death, called a "brain attack", there are different types of ischemic strokes, sudden loss of function

Hemorrhagic

brain bleed, called "blowing a gasket", could be from anuerism but is some sort of a rupture. Can be from hypertension and a vessel has been blown. Pressure from extra blood causes intracranial pressure and because the skull doesn't move a stroke is caused

diaphragmatic breathing

breathing with the use of the diaphragm to achieve maximum inhalation and slow respiratory rate.

What is Grey Turner's sign?

bruising around the back and on flank, blood in the tissue

apraxia

can't carry out task, muscles don't get the message

Older clients are at risk for ARF due to decreased what?

cardiac output

Patient with HPV is at risk for

cervical cancer

If outflow drainage is inadequate with peritoneal dialysis what should the nurse do?

check for kinks, roll or turn patient

What is pleurisy?

chest pain aggravated by cough

Addison's disease is?

chronic adrenal insufficiency

Wernike's

clear speech, recognition not correct, "word salad", brain not working, in the temporal lobe in left hemisphere

Problems with one side neglect

infection, muscle wasting, skin breakdown, injury, safety issues. Approach the patient on the good side.

pleurisy (pleuritis)

inflammation of the pleura.

A contusion to the kidney can cause

intrarenal bleeding

Stoma is red, moist, and slightly raised

just document because this in normal

A male client has just undergone a laryngectomy and has a cuffed tracheostomy tube in place. When initiating bolus tube feedings postoperatively, when should the nurse inflate the cuff?

just prior to tube feeding

Addison s&s

dark bronze skin

Kava kava herb is for

decrease anxiety and stress

hemiplegia

paralysis

Catopril avoid

salt substitutes

Cancer Screen Testing

•Women- Mammogram yearly @40 yrs. •Men & Women- Colonoscopy @ 50 yrs. & then every 10 yrs. •Men- Prostate-specific antigen (PSA) test & Digital rectal examination (DRE) yearly @ 50 yr.

Right Hemispheric

paralysis or weakness on left side of body, left visual field deficit, left side neglect, spatial perceptual deficits, increased distractibility, impulsive behavior and poor judgement, lack of awareness of deficits, memory deficits

Hypocalcemia is common when what organ is injured during surgery?

parathyroid

pt to assess first

pt with AKA

Patient with sickle cell crisis what room is best to place them in

pt with acute pancreatitis

chronic pancreatitis

pulmonary disease state characterized by the presence of airflow obstruction caused by chronic bronchitis or emphysema; clinical use of the term indicates the presence of chronic bronchitis and/or emphysema; includes asthma, chronic bronchiectasis, chronic bronchitis, and emphysema.

bed to wheelchair transfer

raise HOB, help patient sit up, dangle legs

Erythema Marinatum

rash of Rheumatic Fever.

nonstress test (pregnancy)

reactive (normal)

allergic rhinitis

the reaction of the nasal mucosa to a specific allergen.

Why are renal traumas uncommon?

the rib cage and surrounding heavy muscles offer protection

quickening

the stage of pregnancy at which the mother first feels movements of the fetus

rhinoplasty

the surgical reconstruction of the nose.

elastic recoil

the tendency for the lungs to recoil or reduce in volume after being stretched or expanded.

postural drainage

the use of various positions to promote gravity drainage of bronchial secretions; coughing usually expels secretions of the trachea.

Renal traumas are caused by

traffic accidents, stabbing, gun shot wounds

panlobular emphysema

type of emphysema involving distention and destruction of the entire primary respiratory lobule; usually associated with "1-antitrypsin deficiency; also called chronic hypertrophic, diffuse, generalized, panlobular, or vesicular emphysema.

centrilobular emphysema

type of emphysema often associated with chronic bronchitis in which respiratory bronchioles enlarge, the walls are destroyed, and the bronchioles become confluent; characterized by enlargement of air spaces in the proximal part of the acinus, primarily at the level of the respiratory bronchioles.

The nurse is caring for a client who has left ventricular failure. Which of the following should the nurse recognize as being consistent with the DX

wheezing during expiration

The home health care nurse is assigned to see four clients who all live within three miles of each other. The nurse should first see the client

who has cancer of the esophagus and given away his favorite shirt

S/S of graft rejection

fever, fatigue, tenderness at site, anemia, increased BUN

Treatment for HHNS

hydration, IV insulin, restore electrolyte balance

Treatment for DKA

hydration, IV insulin, reverse acidosis, restore electrolyte balance

Prevention of DKA?

hydration, insulin and diabetes education

at risk for .... if have peptic ulcer

gastric cancer

The nurse is observing a staff member caring for clients. It would require immediate intervention if the nurse observes

giving OJ to a client who has a clear liquid diet

hypercapnia

greater than normal amounts of carbon dioxide in the blood (PaCO2 > 45 mm Hg); also called hypercarbia.

priority to see the patient first if

had a lap chole 3 hours ago and is reporting shoulder pain

The Nurse from the postpartum unit has been temporarily assigned to a med surg floor. It would be most appropriate to assign the nurse to the client who

had surgery two hours ago, appendectomy

homonymous hemianopsia

half vision

In caring for a client with acute diverticulitis, which assessment data warrant immediate nursing intervention? The client

has a rigid hard abdomen and elevated white blood cell count (WBC).

Clinical manifestations of renal cancer

hematuria, flank pain, palpable abd mass

Minor and Major lacerations can cause

hemorrhage

Medication treatment for Diabetes Insipidus?

high doses of desmopressin

BUN can be decreased by what diet?

high protein or low carb diets

What to do with meds for dialysis pt

hold 4 hours before give after

What is a significant cause of death for patients with ARF?

infection

S/S for pulmonary HTN?

distended neck veins and edema in extremities

Digoxin teaching

No aspirin, no antacids until 2 hrs before or after administration, visual disturbances = dig. tox

The classic signs of diabetes are:

PPPP - polyuria (peeing alot), polyphagia (hungry alot), polydipsia (thirsty alot)

What is needed at the bedside for a patient post thyroidectomy?

Trach kit, Oxygen, Suction

ptosis

spit

hemoptosis

spit blood

Degree of Burns

1.) SUPERFICAL PARTIAL THICKNESS: * 1'- Epidermis 2.) DEEP PARTIAL THICKNESS: * 2'- Epidermis & Dermis 3.) FULL THICKNESS: * 3'- Epidermis, Dermis, & SQ

hypocapnia

low arterial carbon dioxide pressure (PaCO2 < 35 mm Hg); also called hypocarbia.

Patient education for renal transplant?

immunosuppressive medications, follow up with MD

What is the earliest sign of decreased oxygen?

restlessness

A client with cirrhosis develops increasing pedal edema and ascites. What dietary modification is most important for the nurse to teach this client?

restrict salts and fluid intakes

What cardiac complications are associated with Rheumatic Fever?

* Aortic valve stenosis * Mitral valve stenosis

Drug Abuse: Delirium Tremens (DT)

* Appear 12-36 hrs after last drink. * Tachycardia * Diaphoresis * Marked tremors * Hallucinations * Paranoia * Grand mal seizure (possible).

VRE

needs gown and gloves

contraction stress test

negative (normal) positive (abnormal)

ESRD involves deterioration of _________ with progressive loss of renal function.

nephrons

The nurse is admitting a client to the ED who is reporting progressive visual impairment and loss of peripheral vision. These symptoms are consistent with the medical DX of

Close angel glaucoma

Preventative Treatment for Strokes

Health maintenance, Carotid endarterectomy, anticoagulant therapy, antiplatelet therapy, antihypertensive medications

Intimate Partner Violence: S/S

* Delay between time of injury & time of tx. * Anxious when answering questions about injury. * Looks to abuser for answers to questions. * Depression or suicidal. * Feeling of responsibility, provoking event. * Low self esteem * Abrasions, cuts, sprains, black eyes.

Myasthenia gravis: S/S

* Weakness & fatigue. * Difficulty chewing. * Dysphagia (problem swallowing). * Ptosis (drooping eyelid). * Diplopia (double vision). * Weak, hoarse voice. * Difficulty breathing. * Diminished breath sounds. * Respiratory paralysis & failure.

Right-sided Heart Failure: S/S

* Dependent edema (legs & sacrum). * JVD * Abdominal distention * Hepatomegaly * Splenomegaly * Anorexia & nausea * Weight gain * Nocturnal diuresis * Swelling of the fingers & hands. * Increased BP (from fluid volume excess) or decreased BP (from pump failure).

Types of Shock

* Hypovolemic (fluid loss) * Cardiogenic (pump problem) * Distributive (fluid not going to vital organs)

Dobutamine IV

* Stimulate myocardial contractility & produce a positive inotropic effect. * Increases contractility & decreases preload. * Short term tx for HEART FAILURE.

Fluid volume deficit: Interventions

* Strict I & O. * Replacement of fuids, isotonically.

Pt statement that indicates that they are coping with their stoma

"I showed it to my husband last night."

The nurse is giving preoperative instructions to a 14-year-old female client scheduled for surgery to correct a spinal curvature. Which statement by the client best demonstrates learning has taken place?

"Let me show you the method of turning I will use after surgery."

patient going in for surgery, which statement by the patient would make the nurse need to call the doctor

"My dad died during surgery because the room was really hot."

Genitourinary System involves...

* Urinary system * Reproductive system

Bulemia

stained teeth

Intrarenal cause of ARF

Acute tubular necrosis, damage from nephrotoxins, rhabdomyolysis, surgery, crush injuries, electric shock, use of statin drugs, seen in patients "found down"

External radiation

Avoid sunlight

What if a chest tube is pulled out?

Cover the hole immediately

A client who is receiving an ACE inhibitor for hypertension calls the clinic and reports the recent onset of a cough to the nurse. What action should the nurse implement?

Encourage the client to keep taking the drug until seen by the healthcare provider.

The nurse is providing DC instructions to the parents of a patient with a cleft lip. The nurse should instruct the clients too

Encourage the parents to provide the infant rest periods during feedings

Stage 5 of chronic renal failure is called what?

End stage renal disease

Adrenal Medulla secretes what hormones?

Epi and norepi (Fight or Flight response)

Med Surg To avoid a false positive result for fecal occult blood in stool specimen, the nurse should instruct the client to avoid ingestion of which substances prior to collecting a sample?

Fish, Beef, Vitamin C tablets, Ibuprofen. The fecal occult blood test, or guaic test, measures microscopic amounts of blood in the feces. False positive results can occur from food products such as fish, beef, and other red meats, grren vegetables, vitamin C supplements, aspirin, and NSAIDS.

What is patient at risk of for the rest of their life in the recovery phase of acute renal failure?

Fluid and Electrolyte imbalances

Renal Carcinoma Stage 3

In renal vein, rarely have 5 year survival rate

The nurse is working in the labor and delivery room has become aware of the following situations. The nurse should first assess the client who is

In the first phase of labor, FHR is consistently beating 132 BPM

A postoperative client receives a Schedule II opioid analgesic for pain. Which assessment finding requires the most immediate intervention by the nurse?

Respiratory rate of 12 per minute with O2 saturation of 85%

What does a person with Myxedema look like?

Sluggish, heavy, fluid overloaded, sagging eyes

After attending a class on reducing cancer risk factors, a client selects bran flakes with 2% milk and orange slices from a breakfast menu. In evaluating the client's learning, the nurse affirms that the client has made good choices, and makes what additional recommendation?

Switch to skim milk.

Otherwise known as a mini stroke:

TIA (Transient Ischemic Attack)

Fundamaentals When administering an IM injection to an adult client using the ventrogluteal site, which landmarks should the nurse identify to locate the area for injection?

The anterosuperior iliac spine, which are the landmarks used to give an injection in the ventrogluteal site.

A nurse is caring for a 2 month old infant being evaluated for congenital hypothyroidism, The nurse should recognize which of the following findings and being consistent finding of congenital hypothyroidism

The infant sleeps for 6 hours at a time

ABC

airway and breathing Pain management Sleep and rest Definitive therapy First level problems are immediate priorities (airway, breathing, and circulation). Pain Management Maslows hierarchy of basic needs for rest and sleep Drug therapies

What is a patient after thyroidectomy at risk for?

airway problems

Mormons abstain from

alchohol, coffee and tea

Hypothalamus is the what of the endocrine system?

heart

A client diagnosed with chronic kidney disease (CKD) 2 years ago is regularly treated at a community hemodialysis facility. In assessing the client before his scheduled dialysis treatment, which electrolyte imbalance should the nurse anticipate?

hypocalcemia

RA

music therapy

The nurse in the ED is admitting a client who is hallucinating and reports "insects crawling on skin." The client pulse is 124, RR is 10, the nurse notes muscle twitching in the lower extremities. It would be a priority to determine if the client has

recently ingested cocaine

mechanical receptors

receptors located in lungs, upper airways, chest wall, and diaphragm that are stimulated by irritants, muscle stretching, and alveolar wall distortion.

A female client with a nasogastric tube attached to low suction states that she is nauseated. The nurse assesses that there has been no drainage through the nasogastric tube in the last 2 hours. What action should the nurse take first?

reposition the patient on her side

Hemodialysis

requires AV fistula, dialysis catheter, go to a facility to have done

After PET scan patient can

resume normal activity

Is a kidney tumor slow or fast growing?

slow

What is an EMG

small needle electrodes into muscle

H-A1c

target level is below 7%

what to use if pt needs trach suctioning and is on a vent with infection

closed suctioning equipment

epistaxis

nosebleed

How to prevent infection for pts with ARF?

strict sterile technique, avoidance of indwelling cath's

Fluid volume deficit: S/S

* Decreased urine output. * Weight loss. * Decreased skin turgor. * Dry mucous membranes. * Hypotension. * Tachycardia

Kawasaki Disease: Convalescent (last) Phase

* Starts when all s/s are gone & ends when lab values have returned to normal.

he nurse in the ED is assessing a toddler who has swallowed bleach, The toddler is crying. It would be a priority if the patient parents says

"I gave my toddler a tablespoon of Ipecac syrup."

The nurse has provided discharge instructions for a client who has been prescribed digoxin. It would require follow up by the nurse if the client says.

"I will avoid avocados, grapefruit and cantaloupe."

The nurse has provided health teaching for a group of clients who recently DX with HIV. Which statement made by the clients will require further teaching

"I will miss not being able to work in my garden."

The nurse at a health promotion fair has taught a group of parents about car seat safety. Which of the following statement, if made by the parents, would indicate a correct understanding of the information given?

"I will place my newborn infant in a rear facing car seat in the middle of the rear seat."

The nurse plans to help an 18-year-old developmentally disabled female client ambulate on the first postoperative day. When the nurse tells her it is time to get out of bed, the client becomes angry and yells at the nurse, "Get out of here! I'll get up when I'm ready." Which response should the nurse provide?

"I'll be back in 30 minutes to help you get out of bed and walk around the room."

Intimate Partner Violence

* A criminal act of physical, emotional, economic, or sexual abuse between an assailant & a victim who most commonly in an intimate relationship. * Assailant may have family history of abuse & violence. * Person more violent when drinking or when using drugs. * Relationship characterized by extreme jealousy & issues of power & control. * Women in a battering relationship may lack self confidence & feel trapped. * Abuse often begins during pregnancy.

Serum Sodium

* A major cation of extracellular fluid. * Maintains osmotic pressure & acid-base balance, & assists in the transmission of nerve impulses. * Is absorbed from the small intestine & excreted in the urine in amounts dependent on dietary intake. * Minimum daily requirement of sodium is approximately 15 mEq. * Drawing blood samples in the extremity in which an intravenous (IV) solution of sodium chloride is infusing increases the level, producing an inaccurate result.

Cushing's syndrome

* A metabolic disorder resulting from the chronic & excessive production of cortisol by the adrenal cortex. OR * By the administration of glucocorticoids in large doses for several weeks or longer (exogenous or iatrogenic).

Malignant Hypertension

* A severe type of elevated BP that rapidly progresses. * Systolic BP >200 mm Hg. * Diastolic BP >150 mm Hg or >130 mm Hg when there are pre-existing complications. * Unless intervention occurs promptly, a patient with malignant hypertension may experience renal failure, left ventricular failure, or stroke. S/S: * Morning headaches * Blurred vision * Dyspnea * Symptoms of uremia (accumulation in the blood of substances ordinarily eliminated in the urine).

Colostomy Care

* Adhesive backed opening designed to cover the stoma, should provide 1/8 inch clearance from the stoma. * A rubber band or clip is used to secure the bottom of the pouch and prevent leakage. * Pouch is changed every 3-7 days, when bowel is inactive. * Clients should maintain extra supply of pouches so that they never run out. * Pouch should be emptied when 1/3 - 1/2 full. * Wash around stoma with lukewarm water and a mild soap.

Attention Deficit Disorder (ADD) Medication: Interventions

* Administer 1-3 doses daily. * Take at least 6 hrs before bed. * Take w/ or after meals to avoid appetite suppression. * Monitor height & weight to detect growth suppression.

Dobutamine

* Adrenergic agonists. * Increases BP. * Increases myocardial force & cardiac output through stimulation of beta receptors. * Used in clients with heart failure & for clients undergoing cardiopulmonary bypass surgery.

Early signs of shock...

* Agitation * Restlessness resulting from cerebral hypoxia.

Amyotrophic Lateral Sclerosis (ALS)

* Aka Lou Gehrig's disease. * Progressive degenerative disease involving the MOTOR system. * Sensory & Autonomic systems are NOT involved, & mental status changes do not result from the disease. * Cause of the disease may be related to an excess of glutamate, a chemical responsible for relaying messages between the motor neurons. * As the disease progresses, muscle weakness and atrophy develop until a flaccid tetraplegia develops. * Eventually, the respiratory muscles become affected, leading to respiratory compromise, pneumonia, & death. * No cure is known, & the tx is symptomatic.

Myasthenic Crisis

* An acute exacerbation of the disease. * The crisis is caused by a rapid, unrecognized progression of the disease, inadequate amount of medication, infection, fatigue, or stress. S/S: * Increased P, R, & BP. * Dyspnea, anoxia, & cyanosis. * Bowel & bladder incontinence. * Decreased urine output. * Absent cough & swallow reflex. INTERVENTIONS: * Assess for signs of myasthenic crisis. * Increase anticholinesterase medication, as prescribed.

Kawasaki Disease

* An acute systemic vasculitis that can cause damage to vessels, including the coronary arteries that supply blood flow to the heart. * Can cause permanent damage to the main arteries to the heart, resulting in the formation of an aneurysm of the coronary artery. * Cause of disease unknown. * Usually children < 5 yrs old. 3 STAGES: * Acute phase * Subacute phase * Convalescent (last) phase

Allergic contact dermatitis (reaction)

* An allergic response caused by contact with a substance. S/S: * Rash or skin lesion at site (1st sign). * Rash usually confined to area of exposure. * Appears 1-2 days after exposure. * Itching * Redness * Inflammation * Pain TREATMENT: * Corticosteroid cream (inflammation). * Calamine lotion (itching). * Cool oatmeal bath (itching. * Antihistamine (severe cases).

Pulmonary Embolism

* An embolus blocking the pulmonary artery & disrupting blood flow to one or more lobes of the lung. S/S: * Sudden dyspnea. * Sudden sharp chest or upper abdominal pain. * Cyanosis * Tachycardia * A drop in BP. INTERVENTIONS: * Notify the Dr. immediately because pulmonary embolism may be life-threatening & requires emergency action. * Monitor vital signs. * Administer oxygen & medications as prescribed.

Organ Donation

* An individual who is at least 18 years old may indicate a wish to become a donor on his or her driver's license (state-specific) or in an advance directive. * Heart, lungs, & liver, can be obtained ONLY from a person who is on mechanical ventilation & has suffered brain death. * Other organs or tissues can be removed several hours after death. * Donor must be free of infectious disease & cancer. * Requests to the deceased's family for organ donation usually are done by the physician or nurse specially trained for making such requests. * Donation of organs does not delay funeral arrangements, no obvious evidence that the organs were removed from the body shows when the body is dressed, and the family incurs no cost for removal of the organs donated.

Eptifibatide (Integrilin)

* Anti-platelet (anticoagulant). * Used for Acute Coronary syndrome (unstable angina). SIDE EFFECTS: * GI bleeding * Bruising * Hematuria * Tarry stools ADVERSE EFFECTS: * Bleeding * Hypotension * Thrombocytopenia * Acute Toxicity: Decreased muscle tone, dsypnea, & loss of writing reflex.

Nephrotoxic Substances

* Antibiotics * Aminoglycosides (mycin) * Antineoplastics * NSAIDs * Organic solvents * Dyes & Pesticides * Heavy metals & Ions

Fecal impaction may be contraindicated in pt's w/....

* Anticoagulant therapy. * Cardiac problems.

Amphotericin B

* Antifungal medication. * Used to tx sytemic infections: Candidiasis & Cryptococcal Meningitis. * HIGHLY Nephrotoxic!!!!! SIDE EFFECTS: * Flush * Fever * Chills * N/V * HYPOtension * Paresthesias * Thrombophlebitis

Type 2 Diabetes

* Arises because of insulin resistance. * Body fails to use insulin properly. * Can develop HHS.

Gastrointestinal (GI) bleeding: S/S

* Dark, tarry stools. * Coffee ground emesis. * Bright red rectal bleeding. * Fatigue * Pallor * Severe abdominal pain.

Hallucinations: Intervention

* Ask client directly about hallucination. * Avoid reacting to hallucination as if it were real. * Decrease stimuli or move client to another area. * Do not negate client's experience. * Focus on reality-based topics. * Attempt to engage client's attention through a concrete activity. * Respond verbally to anything real that client talks about. * Avoid touching client. * Monitor for signs of increasing anxiety or agitation, which may indicate that hallucinations are increasing.

Rheumatic Fever: Interventions

* Assess vital signs. * Control joint pain & inflammation with massage & alternating hot & cold applications as prescribed. * Bed rest during the acute febrile phase. * Limit physical exercise in a child with carditis. * Initiate seizure precautions if the child is experiencing chorea. * Assist w/ ambulation. * Instruct the parents about the importance of follow-up and the need for antibiotic prophylaxis for dental work, infection, & invasive procedures. MEDICATIONS: * Antibiotics (penicillin or erythromycin). * Salicylates (Aspirin) * Anti-inflammatory agents.

Prostate Cancer: S/S

* Asymptomatic in early stages. * Hard, pea-sized nodule or irregularities palpated on rectal examination. * Gross, painless hematuria. * Late symptoms such as weight loss, urinary obstruction, & bone pain radiating from the lumbosacral area down the leg. * PSA level is elevated in various noncancerous conditions; therefore, it should not be used as a screening test without a digital rectal examination. It is routinely used to monitor response to therapy. * Dx made through biopsy of the prostate gland.

RDS: Lung Surfacant

* Beractant (Survanta) * Calfactant (Infasurf) * Poractant alfa (Curosurf)

Amyotrophic Lateral Sclerosis (ALS): Interventions

* Care is directed toward the treatment of symptoms. * Monitor respiratory status & institute measures to prevent aspiration. * Provide respiratory treatments. * Prepare to initiate respiratory support. * Assess for complications of immobility. * Address advance directives as appropriate * Provide the client & family with support.

Pheochromocytoma

* Catecholamine-producing tumor usually found in the adrenal medulla, but extra-adrenal locations include the chest, bladder, abdomen, & brain; typically is a benign tumor but can be malignant * Excessive amounts of epinephrine & norepinephrine are secreted. DIAGNOSTIC TEST: * 24-hour urine collection for vanillylmandelic acid (VMA), a product of catecholamine metabolism, metanephrine, & catecholamines, all of which are elevated in the presence of pheochromocytoma; * Normal range of urinary catecholamines is up to 14 mcg/100 mL of urine, with higher levels occurring in pheochromocytoma. TREATMENT: * Surgical removal of the adrenal gland is the primary treatment. * Symptomatic treatment is initiated if surgical removal is not possible. COMPLICATIONS: * Hypertensive crisis, including hypertensive retinopathy and nephropathy, cardiac enlargement,dysrhythmias, congestive heart failure, myocardial infarction, increased platelet aggregation, & stroke. * Death can occur from shock, stroke, renal failure, dysrhythmias, or dissecting aortic aneurysm.

Type 1 Diabetes

* Characterized by the destruction of the pancreatic beta cells, which produce insulin; * Absolute insulin deficiency. * Can develop DKA.

Total Parenteral Nutrition (TPN): Interventions

* Check the TPN solution with the physician's prescription to ensure that the prescribed components are contained in the solution. * To prevent infection & solution incompatibility, IV medications & blood are NOT given through the TPN line. * Stored under refrigeration & administered w/ in 24 hrs from the time they are prepared (remove from refrigerator 30 min-1 hr before use). * Cloudy or darkened solutions should NOT be used & should be returned to the pharmacy. * Additions of substances such as nutrients to TPN solutions should be made in the pharmacy & not on the nursing unit. * In severely dehydrated clients, the Albumin level may drop initially after initiating TPN, because the treatment restores hydration. * With severely malnourished clients, monitor for "refeeding syndrome" (a rapid drop in K+, Mg+, & Phosp serum levels). * Abnormal liver function values may indicate intolerance to or an excess of fat emulsion or problems with metabolism with glucose & protein. * Abnormal renal function tests may indicate an excess of amino acids. LABS TO MONITOR: * PTT & PT for clients receiving anticoagulants. * Electrolytes * Albumin * Liver (ALT & AST) * Renal (BUN & Cr)

Pancreatic Cancer Part 1

* Clinical manifestations depend on the site of origin or metastasis. HEAD OF THE PANCREAS: * Most common site. * Tumors are usually small lesions with poorly defined margins. * Jaundice results from tumor compression & obstruction of the common bile duct & from gallbladder dilation, causing the organ to enlarge. BODY & TAIL: * Usually large & invade the entire tail & body. * Tumors may be palpable abdominal masses, especially in the thin patient. * Through metastatic spread via the splenic vein, metastasis to the liver may cause hepatomegaly (enlargement of the liver up to 2-3x its normal size). * Spreads more extensively than pancreatic head carcinomas, with invasion of the retroperitoneum, vertebral column, spleen, adrenal glands, colon, or stomach. * Regardless of where it originates, it spreads rapidly through the lymphatic and venous systems to other organs.

After the urinary catheter is removed in the TURP client, what are the 3 priority nursing actions?

* Continued strict I & O. * Observations for hematuria. * Inform client burning & frequency may last for a week.

Febrile seizures

* Convulsions brought on by a fever in infants or small children. * Children prone to febrile seizures are not considered to have epilepsy, since epilepsy is characterized by recurrent seizures that are not triggered by fever. S/S: * Fever usually > 102' F. * Loss of consciousness. * Shakes * Moving limbs on both sides of the body. * Can last 15 min-15 seconds. MEDICATION: * Acetaminophen & Ibruphofen. INTERVENTIONS: * Seizure precautions.

Betamethasone & Dexamethasone

* Corticosteroids that increase the production of surfactant to accelerate fetal lung maturity & reduce the incidence or severity of RDS. * Used for a client in preterm labor between 28-32 weeks' gestation whose labor can be inhibited for 48 hrs without jeopardizing the mother or fetus. ADVERSE REACTIONS: * May decrease the mother's resistance to infection. * Pulmonary edema secondary to Na+ & fluid retention can occur. * Elevated blood glucose levels can occur in a client with diabetes mellitus. INTERVENTIONS: * Monitor maternal vital signs, lung sounds, & for edema. * Monitor mother for signs of infection. * Monitor WBC. * Monitor blood glucose levels.

Chemistry Values

* Cr 0.6-1.35 * BUN 7-22 * Albumin 3.4-5 * Protein 6.2-8.1 * Glucose 70-110 * Specific Gravity 1.010-1.030

CPR

* Determine responsiveness, if no response call a "code". * Remain w/ client & tell someone to get the crash cart. * Put client on cardiac board or bed in CPR postion. * Start chest compressions, 30 with both hands over the lower half of the sternum at a rate of 100 per minute w/ depth of 2 inches. * After 30 chest compressions, open airway with head tilt-chin maneuver & ventilate by mask or bag over 1 second for 2 breaths.

Attention Deficit Disorder (ADD): Medications

* Dextroamphetamine sulfate (Dexedrine) * Methylphenidate HCI (Ritalin & Concerta) * Pemoline (Cylert) * Lisdexamfetamine (Vyvanse) *Amphetamine/Dextroamphetamin(Adderall) * Dexmethylphenidate (Focalin)

Therapeutic Drug Levels

* Digoxin 0.5-2.0 * Lithium 0.8-1.5 * Dilantin 10-20 * Theophylline 10-20

Fluid volume excess: Interventions

* Diuretics * Fluid restriction. * Strict I & O. * Na+ restricted diet. * Weigh daily. * Serum K + monitored.

S/S of HYPERkalemia

* Dizziness * Weakness * Cardiac irregularities. * Muscle cramps. * Diarrhea * Nausea

ICP ABGs: Vent setting

* Dr. may order passive HYPERventilation on ventilator. * Leads to Respiratory Alkalosis, which causes cerebral vasoconstriction & decreased cerebral blood flow, & decreased ICP.

S/S of HYPOkalemia

* Dry mouth * Thirst * Drowsiness * Lethargy * Muscle weakness * Aches * Tachycardia

Fluid volume overload: S/S

* Dyspnea * Tachycardia * JVD * Peripheral edema * Pulmonary edema * Weight gain

Left-sided Heart Failure: S/S

* Dyspnea * Tachypnea * Crackles in the lungs * Dry, hacking cough. * Pink, frothy sputum. * Paroxysmal nocturnal dyspnea. * Increased BP (from fluid volume excess) or decreased BP (from pump failure).

Dobutamine IV: Side effects

* Dysrhythmias * Hypotension * Thrombocytopenia ADVERSE REACTIONS: * Hepatotoxicity manifested by elevated liver enzyme levels. * Hypersensitivity manifested by wheezing, SOB, pruritus, urticaria, clammy skin, & flushing.

Intimate Partner Violence: Interventions

* Establish trust, use nonjudgemental approach. * Tx physical wounds. * Interview victim when abuser is not present. * Document factual, objective statements about client's physical condition, injuries, & interaction w/ partner ot family. * Determine potential for further violence. * Provide crisis intervention. * Assist w/ referral to shelter. * Contact authorities.

Rheumatic Fever: S/S

* Fever * Rash * Chest pain * SOB * Tachycardia * Large joint pain. * Subcutaneous nodules over bony prominences * Chorea (irregular involuntary movements). * Aschoff bodies (lesions): Found in the heart, blood vessels, brain, & serous surfaces of the joints & pleura. LABS: * Elevated antistreptolysin O titer. * Elevated erythrocyte sedimentation rate. * Elevated C-reactive protein level.

DKA & HHS: Treatment

* Fluid replacement. * Insulin administration. * Correct electrolyte imbalances.

Cushing's disease: S/S

* Generalized muscle wasting & weakness. * Moon face, buffalo hump. * Truncal obesity with thin extremities, supraclavicular fat pads; * Weight gain * Hirsutism (masculine characteristics in female). * Hypertension * Fragile skin that easily bruises. * Reddish-purple striae on the abdomen & upper thighs. * Hyperglycemia, hypernatremia * Hypokalemia, hypocalcemia

Warning signs of Suicide

* Gives away possessions. * Depressed client becomes happy.

Acute Glomerulonephritis (AGN): S/S

* Gross hematuria * Recent strep infection. * HTN * Mild edema * Positive ASO titer.

What can cause Fluid volume excess?

* HF * Renal failure * Cirrhosis, liver failure * Excessive ingestion of salt. * Overhydration w/ Na+ containing fluid. * Poorly controlled IV therapy.

Diabetes Insipidus (DI)

* HYPOsecretion of ADH caused by stroke or trauma. * May be idiopathic. * Kidney tubules fail to reabsorb water. S/S: * Polyuria of 4-24 L/day. * Polydipsia (thirsty) * Dehydration (decreased skin turgor & dry mucous membranes). * Inability to concentrate urine. * Low urinary specific gravity, 1.006 or lower. * Fatigue * Muscle pain & weakness. * Headache * Postural hypotension that may progress to vascular collapse without rehydration. * Tachycardia

Hematology Values

* Hct 12-16 * Hgb 36-50 * RBC 4.5-5 * WBC 5-10 * Plt 150-400

Rule of Nines

* Head 9% * Arms 18% (9% each). * Chest & Back 36% (18% each). * Legs 36% (18% each) * Genitalia 1%

Risk factors for HTN

* Heredity * Race * Age * Alcohol * Increased salt intake. * Obesity * Use of oral contraceptives.

Chronic Kidney Disease: Diet

* High calories * Low (or restrict) protein. * Low sodium. * Low potassium. * Low phosphate.

Kawasaki Disease: Acute Phase

* High fever more that 5 days. * Conjuctival redness. * Strawberry tongue. * Red swollen hands & feet. * Irritability

What is the diet for Nephrosis?

* High protein * Low salt

Diabetic ketoacidosis (DKA): S/S

* Hyperglycemia >250 * Ketonuria in large amounts. * Kussmaul's respirations * Acetone (fruity) breath odor. * Increasing lethargy * Decreasing LOC. * N/V * Dehydration

Hyperosmolar Hyperglycemia Syndrome (HHS):S/S

* Hyperglycemia >600. * Plasma hyperosmality. * Dehydration * Changed mental status. * Absent ketone bodies.

Addison's disease

* Hyposecretion of adrenal cortex hormones (glucocorticoids & mineralocorticoids). * Can be primary or secondary. * The condition is fatal if left untreated. INTERVENTIONS: * Monitor vital signs, particularly BP, weight, and I & O. * Monitor WBC count, blood glucose, K+, Na+, & Ca+ levels. * Administer glucocorticoid or mineralocorticoid medications as prescribed. * Observe for addisonian crisis caused by stress, infection, trauma, or surgery. * Avoid individuals with an infection. * DIET: High protein & high carbohydrate, normal sodium intake. * Avoid strenuous exercise & stressful situations. * Need for lifelong glucocorticoid therapy. * Avoid OTC medications. * Wear a Medic-Alert bracelet.

Chemotherapy: Discharge Teaching

* If diarrhea is a problem, avoid hot foods & high-fiber foods. * Inspect the oral mucosa frequently for erythema & ulcers. * Rinse the mouth after meals, & carry out good oral hygiene. * Use mouth rinses as prescribed for mouth sores if necessary. * Use antifungal agents for mouth sores, if prescribed, for the development of a fungal infection. * Avoid crowds & persons with infections. * Report signs of infection such as a low-grade fever, chills, or sore throat. * Instruct individuals with colds or infections to wear a mask when visiting or to avoid visiting the client. * Use a soft toothbrush & electric razor. * Avoid aspirin-containing products. * Consult the physician before receiving vaccinations (live vaccines should not be administered).

Shock: Interventions

* If shock develops, elevate the legs. * Notify the physician. * Determine & treat the cause of shock. * Administer oxygen, as prescribed. * Monitor LOC. * Monitor vital signs for increased P or decreased BP. * Monitor I & O. * Assess color, temperature, turgor, & moisture of the skin & mucous membranes. * Administer IV fluids, blood, & colloid solutions, as prescribed.

Seizure precautions

* If the client is standing or sitting, place the client on the floor & protect the head & body. * Support the ABCs (airway, breathing, & circulation). * Do not put anything in mouth. * Administer oxygen (if needed). * Turn the client to the side. * Prevent injury. * Remain with the client. * Do not restrain the client. * Loosen restrictive clothing. * Note the type, character, & progression of the movements during the seizure. * Note duration.

Femoral arterial sheath

* In certain procedures during the cardiac catheterization, a femoral sheath is often left in place & removed in several hours by the nurse caring for the patient. * Post-interventional sheaths provide emergency access to the vasculature in the event that the coronary artery occludes & to allow time for anticoagulants to wear off. INTERVENTIONS: * Timing of the removal of the femoral sheath depends on location of the access site, size of the sheath, use of anticoagulants, and agency protocols. * Manual pressure applied to the groin can stimulate the baroreceptors & cause a vasovagal reaction in which the patient becomes bradycardic & hypotensive. * Vasovagal reactions are usually brief & self-limiting. * When applying pressure to the groin after sheath removal, be alert for a vasovagal reaction & be prepared to treat it by lowering the head of the bed to the flat position & giving a bolus of IV fluids. * Have emergency resuscitation equipment available during sheath removal.

Dobutamine IV: Interventions

* Increase in BP is expected outcome. * Should NOT be diluted with dextrose-containing solutions. * For continuous IV infusion, administer with an infusion pump. * Stop the infusion if the client's BP drops or dysrhythmias occur. * Monitor the apical pulse & BP. * Monitor for hypersensitivity. * Assess lung sounds for wheezing & crackles. * Monitor for edema. * Monitor for relief of HF as noted by reduction in edema, lessening of dyspnea, orthopnea, & fatigue. * Monitor electrolyte & liver enzyme levels, platelet count, & renal function studies; * The medications may decrease K+ & increase liver enzyme levels; continuous electrocardiographic monitoring is done during administration.

Chemotherapy: Interventions

* Initiate bleeding precautions if thrombocytopenia occurs. * When the platelet count is less than 50,000 cells/mm3, minor trauma can lead to episodes of prolonged bleeding; when less than 20,000 cells/mm3, spontaneous and uncontrollable bleeding can occur. * Monitor for petechiae, ecchymosis, bleeding of the gums, & nosebleeds. * Avoid IM injections & venipunctures as much as possible to prevent bleeding. * Initiate neutropenic precautions if the white blood cell count decreases. * Monitor for fever, sore throat, unusual bleeding, or signs & symptoms of infection. * Inform the client that loss of appetite also may be the result of taste changes or a bitter taste in the mouth from the medications. * Monitor for nausea & vomiting. * Provide a high-calorie diet with protein supplements. * Administer antiemetics several hours before chemotherapy & for 12 to 48 hours after as prescribed. * Encourage hydration; IV fluids are administered before & during therapy. * Promote a fluid intake of at least 2000 mL/day to maintain adequate renal function. LABS TO MONITOR: * CBC * WBC * Platelet count * Uric acid level * Electrolytes

Assessment of a child with suspected Rheumatic Fever includes...

* Inquiring about a recent sore throat because rheumatic fever manifests 2-6 wks after an untreated or partially treated group A beta-hemolytic streptococcal infection of the upper respiratory tract.

Lung Surfacants: Interventions

* Instill surfactant through the catheter inserted into the newborn's endotracheal tube; * Avoid suctioning for at least 2 hrs after administration. * Monitor for bradycardia & decreased O2 sat during administration. * Monitor respiratory status & lung sounds & for signs of adverse reactions.

Pancreatic Cancer: S/S

* Jaundice • Clay- (light) colored stools. • Dark urine * GI bleeding (esophageal varicies). * Coffee ground emesis. • Abdominal pain: usually vague, dull, or nonspecific that radiates into the back. • Weight loss • Anorexia • N/V * Spleenomegaly * Ascites * Glucose intolerance * Weakness & fatigue * Thrombophlebitis

Cardiac catheterization: Post-Op Interventions

* Keep affected extremity immobilized for 6-8 hrs after removal of catheter. * Lie flat for 6 -12 hrs (& possibly overnight if the catheters are left in the groin). * Encourage patient to drink 1-2 L of fluid after procedure or administer oral & IV fluids as ordered.

Colostomy care (flatus)

* Leaking or inadequately closed pouch is the usual cause of odor, flatus can also contribute to the odor. * Remind the patient that although generally no foods for ostomates are forbidden, certain foods and habits can cause flatus or contribute to odor when the pouch is open. FOODS THAT CAUSE GAS: * Broccoli * Beans * Spicy foods * Onions * Brussels sprouts * Cabbage * Cauliflower * Cucumbers * Mushrooms * Peas * Chewing gum * Smoking * Drinking beer * Skipping meals. FOODS THAT CAUSE ODOR: * Asparagus * Broccoli * Cabbage * Turnips * Eggs * Fish * Garlic FOODS THAT PREVENT GAS: * Crackers * Toast * Yogurt FOODS THAT PREVENT ODOR: * Buttermilk * Cranberry juice * Parsley * Yogurt * Charcoal filters * Pouch deodorizers * Breath mint in the pouch.

Addison's disease: S/S

* Lethargy, fatigue, & muscle weakness. * GI disturbances. * Weight loss. * Menstrual changes in women; impotence in men. * Postural hypotension * Hyperpigmentation of skin (bronzed) with primary disease. * Hypoglycemia, Hyponatremia * Hyperkalemia, Hypercalcemia

Breast Cancer: S/S

* Mass felt during BSE. * Mass usually felt in the upper outer quadrant, beneath the nipple, or in axilla. * A fixed, irregular nonencapsulated mass; typically painless except in the late stages. * Nipple retraction or elevation. * Asymmetry, with the affected breast being higher. * Bloody or clear nipple discharge. * Skin dimpling, retraction, or ulceration. * Skin edema or peau d'orange skin. * Axillary lymphadenopathy. * Lymphedema of the affected arm. * Symptoms of bone or lung metastasis in late stage. * Presence of the lesion on mammography.

The classic manifestations of nephrotic syndrome are....

* Massive proteinuria * Hypoalbuminemia * Edema

Nephrosis: S/S

* Massive proteinuria * Severe edema * Frothy urine * Anorexia * Negative ASO titer.

Attention Deficit Disorder (ADD) Medication: Adverse Reactions

* May interact with MAOI, producing fever & hypertensive crisis. * Nervousness * Insomnia * Dizziness * Tourette's syndrome * Tachycardia, palpitations, angina, & dysrhythmias. * Anorexia * Weight loss * Nausea * Abdominal pain

Warfarin (Coumadin): Pt Teaching

* Monitor PT & INR. * Observe for bleeding gums, bruises, nosebleeds, hematuria, hematemesis, occult blood in the stool, & petechiae. * Instruct the client regarding measures to prevent bleeding. * Soft toothbrush, electric razor. * Limit foods w/ Vitamin K.

Kawasaki Disease: Interventions

* Monitor cardiac status: * Monitor I & O. * Daily weights. * Minimize skin discomfort w/ lotions & cool compresses. * Provide meticulous mouth care. MEDICATIONS: * IV immunoglobulin (IVIG) * Acetaminophen & Aspirin (salicylate therapy)

DIC: Interventions

* Monitor for bleeding. * Monitor vital signs. * Monitor PT/INR. * Prevent injury. * Gentle oral care w/ swabs. * Minimize needle sticks, use smallest gauge. * Turn frequently. * Minimize BP measurements by cuff. * Apply pressure to oozing site. * Administer heparin IV during 1st phase to inhibit coagulation.

Myasthenia gravis: Interventions

* Monitor respiratory status, ability to cough & deep-breathe adequately. * Monitor for respiratory failure. * Maintain suctioning & emergency equipment at the bedside. * Monitor vital signs. * Monitor speech & swallowing abilities to prevent aspiration. * Encourage the client to sit up when eating. * Assess muscle status. * Instruct the client to conserve strength. * Plan short activities that coincide with times of maximal muscle strength. * Monitor for myasthenic & cholinergic crises. * Administer anticholinesterase medications as prescribed. * Instruct the client to avoid stress, infection, fatigue, & over-the counter medications. * Instruct the client to wear a Medic-Alert bracelet. * Anticholinesterase medications: Increase levels of acetylcholine at the myoneural junction.

Pheochromocytoma: Interventions

* Monitor vital signs, particularly BP & HR. * Monitor for hypertensive crisis; monitor for complications that can occur with hypertensive crisis, such as stroke, cardiac dysrhythmias, & myocardial infarction. * Instruct the client not to smoke, drink caffeine-containing beverages, or change position suddenly. * Prepare to administer a β-adrenergic blocking agent as prescribed to control hypertension. * Monitor serum glucose level. * Promote rest & a nonstressful environment. * Provide a diet high in calories, vitamins, & minerals. * Prepare the client for adrenalectomy.

Cushing's disease: Interventions

* Monitor vital signs, particularly BP. * Monitor I & O and weight. * Monitor WBC, serum glucose, Na+, K+, & Ca+ levels. * Provide meticulous skin care. * Allow the client to discuss feelings related to body appearance. * Administer chemotherapeutic agents as prescribed for inoperable adrenal tumors. * Prepare the client for radiation as prescribed if the condition results from a pituitary adenoma. * Prepare the client for removal of pituitary tumor (hypophysectomy, transsphenoidal adenectomy) if the condition results from increased pituitary secretion of ACTH.

Conn's syndrome: Interventions

* Monitor vital signs, particularly BP. * Monitor for signs of hypokalemia & hypernatremia. * Monitor I & O and urine for specific gravity. * Spironolactone (Aldactone) may be prescribed to promote fluid balance & control hypertension; * Administer potassium supplements as prescribed. * Prepare the client for adrenalectomy. * Maintain Na+ restriction, if prescribed, preoperatively. * Administer glucocorticoids preoperatively, as prescribed, to prevent adrenal hypofunction. * Monitor the client for adrenal insufficiency postoperatively. * Instruct the client regarding the need for glucocorticoid therapy following adrenalectomy. * Instruct the client about the need to wear a Medic-Alert bracelet.

Acute Renal Failure (ARF): Interventions

* Monitor vital signs. * I & O hourly in ARF. * Limit fluid intake to 400-500ml/hr. * Assess urine amount, color & characteristics. * Daily weight. * Monitor for altered LOC caused by uremia. * Monitor for signs of infection because the client may not exhibit an elevated temperature or an increased WBC. * Auscultate lungs for wheezes & rhonchi. * Monitor for edema, which can indicate fluid overload. LABS: * BUN * Cr * Electrolyes * Urinalysis for protein level, hematuria, casts, & specific gravity. DIET: * Low-to-moderate protein (to decrease the workload on the kidneys) & High-carbohydrate diet. * Restrict K+ & Na+ intake as prescribed based on the electrolyte level.

Diabetes Insipidus (DI): Interventions

* Monitor vital signs. neuro & cardio status. * Provide a safe environment, particularly for the client with postural hypotension. * Monitor electrolyte values & for signs of dehydration. * Maintain client intake of adequate fluids. * Monitor I & O, weight, serum osmolality, & specific gravity of urine. * Instruct the client to avoid foods or liquids that produce diuresis. MEDICATIONS: * Vasopressin tannate (Pitressin) * Desmopressin acetate (DDAVP, Stimate) * These are used when the ADH deficiency is severe or chronic. * DDAVP may be administered by injection, intranasally, or orally.

Pancreatic cancer: Part 2

* Most are highly malignant, adenocarcinomas. * Symptoms usually do not occur until the tumor is large; therefore, the prognosis is poor. RISK FACTORS: * Increased age. * History of diabetes mellitus. * Alcohol use. * History of previous pancreatitis. * Smoking * Ingestion of a high-fat diet. * Exposure to environmental chemicals. DIAGNOSTIC TEST: * Endoscopic retrograde cholangiopancreatography for visualization of the pancreatic duct and biliary system and collection of tissue and secretions may be done.

Antidotes

* Warfarin (Coumadin)- Vit K * Heparin- Protamine sulfate * Mg+ sulfate- Ca+ gluconate. * Digoxin- Digibind. * Benzos- Flumzaemil * Tylenol- Mucomyst * Opiates- Narcan * Cholinergic meds- Atropine

Rheumatic Fever

* Most common cause of acquired heart disease in children. * Inflammatory autoimmune disease. * Injures the heart, joints, subcutaneous tissues, & blood vessels of the CNS. * Manifests 2-6 wks after an untreated or partially treated group A beta-hemolytic streptococcal infection of the upper respiratory tract.

Hypocalcemia: S/S

* Muscle spasms * Convulsions * Cramps/tetany * (+) Trousseau's * (+) Chvostek's * Prolonged ST interval * Prolonged QT segment

Restraints

* Need Dr.'s rx. * State the type of restraint. * Identify specific client behaviors for which restraints are to be used. * Limited time frame for use. * Safety devices are not to be prescribed PRN. * The reason for the safety device should be given to the client & the family, and their permission should be sought. HOW TO APPLY: * Use a half-bow or safety knot (quick release tie) to secure the device to the bed frame or chair, NOT to the side rails. * Ensure that enough slack is on the straps to allow some movement of the body part. * Assess skin integrity & neurovascular & circulatory status every 30 minutes. * Remove the safety device at least every 2 hours to permit muscle exercise & to promote circulation. * Continually assess & document the need for safety devices.

Myasthenia gravis

* Neuromuscular disease characterized by considerable weakness & abnormal fatigue of the voluntary muscles. * A defect in the transmission of nerve impulses at the myoneural junction occurs. CAUSES: * Insufficient secretion of acetylcholine (ACTH). * Excessive secretion of cholinesterase. * Unresponsiveness of the muscle fibers to acetylcholine.

Prostate VCA Screening

* No risk- 50 yrs.old. * High risk- begin @ 40-45yrs.old.

Fluid volume deficit

* Occurs when the body loses water & electrolytes. S/S: * Weight loss * Decreased skin turgor. * Oliguria (concentrated urine). * Postural hypotension * Weak, rapid pulse.

Fluid volume excess

* Occurs when the body retains water & electrolytes. S/S: * Peripheral edema * Increased bounding pulse * Elevated BP * Distended neck & hand veins. * Dyspnea * Crackles in lungs. * Attention loss, confusion, asphasia. * Altered LOC.

Hypovolemic Shock

* Occurs when too little circulating blood volume causes a MAP decrease, resulting in inadequate total body oxygenation. * R/t external or internal blood or fluid loss. * Most common cause of shock. CAUSES: * Hemorrhage * Dehydration

Shock: Tx

* Oxygenation & Ventilation * Fuid resuscitation * Drug Therapy * Monitoring

How is DIC dx?

* PT & PTT- prolonged. * Platelets- decreased. * FSPs- increased.

Interventions to prevent a thromboembolism w/ a hip fracture?

* Passive ROM exercises. * Elastic stocking. * Elevation of foot of bed 25 degrees to increase venous return. * Low dose Heparin therapy.

Kawasaki Disease: Subacute Phase

* Peeling of hands & feet. * Irritability

What medications are used to treat Rheumatic Fever?

* Pencillin * Erythromycin * Aspirin

Breast self examination (BSE)

* Perform monthly 7-10 days AFTER period. * Women who had a mastectomy should select a specific day of the month & perform BSE monthly on that day. HOW TO PERFORM: * While in the shower or bath, when the skin is slippery with soap & water, examine your breasts. * Use the pads of your second, third, & fourth fingers to press every part of the breast firmly. * Use your right hand to examine your left breast, & use your left hand to examine your right breast. * Using the pads of the fingers on your left hand, examine the entire right breast using small circular motions in a spiral or up-and-down motion so that the entire breast area is examined. * Repeat the procedure using your right hand to examine your left breast. * Repeat the pattern of palpation under the arm. * Check for any lump, hard knot, or thickening of the tissue. * Look at your breasts in a mirror. * Stand with your arms at your side. * Raise your arms overhead & check for any changes in the shape of your breasts, dimpling of the skin, or any changes in the nipple. * Next, place your hands on your hips & press down firmly, tightening the pectoral muscles. * Observe for asymmetry or changes, keeping in mind that your breasts probably do not match exactly. * While lying down, feel your breasts as described in shower. * When examining your right breast, place a folded towel under your right shoulder & put your right hand behind your head. * Repeat the procedure while examining your left breast. * Mark your calendar that you have completed your breast-self-examination; note any changes or unique characteristics you want to check with your health care provider.

Partial or Total Nephrectomy

* Performed for extensive kidney damage, renal infection, severe obstruction from stones or tumors, & prevention of stone recurrence. POST-OP INTERVENTIONS: * The plan of care depends on the incision location and the type of drainage tubes present. * Monitor the incision, particularly if a Penrose drain is in place, because it will drain large amounts of urine. * Protect the skin from urinary drainage, changing dressings frequently if necessary. * Place an ostomy pouch over the Penrose drain to protect the skin if urinary drainage is excessive. * Monitor the nephrostomy tube, which may be attached to a drainage bag, for a continuous flow of urine. * Do not irrigate the nephrostomy or bladder catheters unless specifically prescribed. * Monitor the indwelling bladder catheter for drainage. * Encourage fluid intake to ensure a urine output of 2500 to 3000 mL/day or more. * Measure I & O. * If a stone was removed, determine its composition from laboratory analysis.

Tensilon Test

* Performed to diagnose Myasthenia gravis & to differentiate between myasthenic crisis & cholinergic crisis. * The test places the client at risk for ventricular fibrillation and cardiac arrest; the nurse must be prepared for this possibility. * Have ATROPINE SULFATE available when performing test!!!!!! TO DX MYASTHENIA GRAVIS: * Edrophonium (Tensilon) injection is administered to the client. * POSITIVE- Client shows improvement in muscle strength after the administration of Tensilon. * NEGATIVE- Client shows no improvement in muscle strength, and strength may even deteriorate after injection of Tensilon.

When your Magnesium is HIGH, your...

* Phosphorus (high) * Calcium (low)

Mannitol (Osmitrol)

* Potent Osmotic K + wasting diuretic. * Prevents water reabsorption. * Draws fluid from the extravascular spaces into the plasma. * Increases the osmolality (concentration) & Na+ reabsorption. * Diuresis occurs 1-3 hrs after IV administration. USED FOR: * Prevent kidney failure. * Decrease ICP (in cerebral edema). * Decrease intraocular pressure (IOP) (in glaucoma).

Nephrotic Syndrome: Meds

* Prednisone- steroid * Bethanechol (Urecholine)- cholinergic * Cytoxan (if doesn't respond to Prednisone). * IV Albumin * Diuretic

Conn's syndrome

* Primary hyperaldosteronism. * Hypersecretion of mineralocorticoids (aldosterone) from the adrenal cortex of the adrenal gland. * Most commonly caused by an adenoma.

Diabetic: Preventive Foot Care

* Provide meticulous skin care & proper foot care. * Inspect feet daily & monitor feet for redness, swelling, or break in skin integrity. * Notify the physician if redness or a break in the skin occurs. * Avoid thermal injuries from hot water, heating pads, & baths. * Wash feet with warm (not hot) water & dry thoroughly (avoid foot soaks). * Avoid treating corns, blisters, or ingrown toenails. * Do not cross legs or wear tight garments that may constrict blood flow. * Apply moisturizing lotion to the feet but not between the toes. * Prevent moisture from accumulating between the toes. * Wear loose socks and well-fitting (not tight) shoes, & instruct the client not to go barefoot. * Wear clean cotton socks to keep the feet warm & change the socks daily. * Avoid wearing the same pair of shoes 2 days in a row. * Avoid wearing open-toed shoes or shoes with a strap that goes between the toes. * Check shoes for cracks or tears in the lining and for foreign objects before putting them on. * Break in new shoes gradually. * Cut toenails straight across & smooth nails with an emery board. * Avoid smoking.

Cardiogenic Shock

* R/t ischemia or impairment in tissue perfusion. * Occurs when the actual heart muscle is unhealthy & pumping is directly impaired. CAUSES: * MI * Arrhythmias * HF

Acute Renal Failure

* Rapid loss of kidney function from renal cell damage. * Occurs abruptly & can be reversible. * ARF leads to cell hypoperfusion, cell death, & decompensation of renal function. * Prognosis depends on the cause & the condition of the client. * Near-normal or normal kidney function may resume gradually.

Epistaxis (nosebleed)- Pediatrics: Interventions

* Remain calm & keep the child calm & quiet. * Have the child sit up & lean forward (not lying down). * Apply continuous pressure to the nose with the thumb & forefinger for at least 10 minutes. * Insert cotton or wadded tissue into each nostril, & apply ice or a cold cloth to the bridge of the nose if bleeding persists. * If bleeding cannot be controlled, packing or cauterization of the bleeding vessel may be prescribed. ---------------------------------------------- * If a nosebleed occurs in a child, it is important for the nurse to remain calm; otherwise, the child becomes agitated and it is difficult to get the child to cooperate with the necessary interventions. * The child should be assisted to a sitting up & leaning forward position to prevent aspiration of blood. * The child should NOT be placed in a lying down position. * Nosebleeds usually originate in the anterior part of the nasal septum & can be controlled by applying pressure to the soft lower portion of the nose with the thumb & forefinger for at least 10 minutes. * If bleeding persists, cotton or wadded tissue should be placed into each nostril, & ice or a cold cloth should be applied to the bridge of the nose. * If bleeding does persist, the physician needs to be notified, & the nose may require packing by the physician. * After the nosebleed has been stopped, petroleum or a water-soluble jelly may be inserted into each nostril to prevent crusting of old blood & to lessen the likelihood of the child picking at the crusted lesions & restarting the bleeding. * Repeated bleeding episodes that last longer than 30 minutes may be an indication of the need for evaluation of a bleeding disorder.

Lung Surfacants

* Replenish surfactant & restore surface activity to the lungs to prevent & treat respiratory distress syndrome (RDS). * Used to prevent or treat RDS in premature infants. * Administered by the intratracheal route. * Usually given w/in 8 hrs of birth. ADVERSE REACTIONS: * Transient bradycardia. * Oxygen desaturation. * Pulmonary hemorrhage. * Mucus plugging. * Endotracheal tube reflux. CONTRAINDICATED: * In infants at risk for circulatory overload.

Amyotrophic Lateral Sclerosis (ALS): S/S

* Respiratory difficulty. * Fatigue while talking. * Muscle weakness & atrophy. * Tongue atrophy. * Dysphagia (problem swallowing). * Weakness of the hands and arms. * Fasciculations of the face (muscle twitch). * Nasal quality of speech. * Dysarthria (problem using muscles needed to speak).

Antineoplastic medication causes the rapid destruction of cells....

* Resulting in the release of uric acid. * Allopurinol (Zyloprim) may be prescribed to lower the serum uric acid level.

Epidural hematoma

* Results from arterial bleeding into the space between the dura & the inner skull. * Often caused by a fracture of the temporal bone, which houses the middle meningeal artery. * Symptoms progress very quickly w/ life threatening ICP elevation & structural changes. * NEUROSURGICAL EMERGENCY!!! S/S: * "lucid intervals" that last for minutes during which time the patient is awake & talking. * Followed by momentary unconsciousness that occurs within minutes of the injury & can lead to coma. INTERVENTIONS: * Monitor every 5-10 minutes for changes in neuro status. * If changes occur, notify Rapid Response Team immediately!!

Distributive Shock: Anaphylactic, Neurogenic, & Septic Shock)

* Results from excessive vasodilation & the impaired distribution of blood flow. * Occurs when blood volume is not lost from the body but is distributed to the interstitial tissues where it cannot circulate & deliver oxygen. CAUSES: 1.) NEURAL-INDUCED: •Pain •Anesthesia •Stress •Spinal cord injury •Head trauma 2.) CHEMICAL-INDUCED: •Anaphylaxis •Sepsis •Capillary leak •Burns •Extensive trauma •Liver impairment •Hypoproteinemia

The s/s of acute renal failure are primarily caused by...

* Retention of nitrogenous wastes. * Retention of fluids. * Inability of the kidneys to regulate electrolytes.

MRSA

* Spread through person-to-person contact, through contact with contaminated items, or through infection of a preexisting cut or wound that is not protected by a dressing. * Bacteria found on skin or in nose.

MRSA precautions

* Standard & Contact isolation precautions. * C/S to confirm & determine antiobiotic tx. * Wash hands before & after tx. * Gown & glove before entering. * Keep equipment in room.

Radical Nephrectomy

* Surgical removal of the entire kidney, adjacent adrenal gland, renal artery & vein. * Radiation therapy & possibly chemotherapy may follow radical nephrectomy. * Before surgery, radiation may be used to embolize (occlude) the arteries supplying the kidney to reduce bleeding during nephrectomy. POST-OP INTERVENTIONS: * Monitor vital signs for signs of bleeding (hypotension & tachycardia). * Monitor for abdominal distention, decreases in urinary output, & alterations in LOC as signs of bleeding; check the bed linens under the client for bleeding. * Monitor for signs of adrenal insufficiency, which include a large urinary output followed by hypotension & subsequent oliguria. * Administer fluids & PRBCs intravenously as prescribed. * Monitor I & O. * Daily weight. * Monitor for a urinary output of 30 to 50 mL/hr to ensure adequate renal function. * Monitor urine specific gravity. * Semi-Fowler's position. * Encourage coughing & deep-breathing exercises. * Monitor for passing of flatus & bowel sounds. * Apply antiembolism stockings as prescribed. * If a nephrostomy tube is in place, do not irrigate (unless specifically prescribed) or manipulate the tube. * Administer pain medications as prescribed.

Conn's syndrome: S/S

* Symptoms related to hypokalemia, hypernatremia, & HTN. * Headache, fatigue, muscle weakness, & nocturia. * Polydipsia & polyuria. * Paresthesias * Visual changes * Low urine specific gravity and increased urinary aldosterone level. * Elevated serum aldosterone levels. * Metabolic alkalosis

TPN- Central vein catheter (CVC)

* TPN is administered through a central vein when the client requires a larger concentration of carbohydrates (>10% glucose concentration). (< 4 WEEKS): * Subclavian vein or * Internal jugular vein (> 4 WEEKS): * PICC line or * Implanted vascular device.

Air embolism: S/S

* Tachycardia * Dyspnea * Hypotension * Cyanosis * Decreased level of consciousness.

Shock: S/S

* Tachycardia >100 bpm. * Tachypnea >24 bpm. * Hypotension <80 mm Hg. * Restlessness, hyperalert (early sign) * Decreased alterness, lethargy, coma (late sign) * Cool, clammy skin. * Warm skin (Septic) * Diaphoresis * Paleness * Decreased urine output. * CVP abnormal.

Digital rectal exam (DRE)

* Test to check the prostate. Because the prostate is an internal organ, your doctor cannot look at it directly. * Prostate lies in front of the rectum. * Performed by inserting a gloved, lubricated finger into the rectum.

Diabetes Insipidus

* The HYPOsecretion of antidiuretic hormone from the posterior pituitary gland, resulting in failure of tubular reabsorption of water in the kidneys & diuresis.

Heart Failure (HF)

* The inability of the heart to maintain adequate cardiac output to meet the metabolic needs of the body because of impaired pumping ability. * Diminished cardiac output results in inadequate peripheral tissue perfusion. * Congestion of the lungs & periphery may occur; the client can develop acute pulmonary edema. 2 TYPES: * Right- sided. * Left- sided.

Epistaxis (nosebleed) - Pediatrics

* The nose, especially the septum, is a highly vascular structure, & bleeding usually results from direct trauma, foreign bodies, & nose picking or from mucosal inflammation. * Recurrent epistaxis & severe bleeding may indicate an underlying disease.

Port-A-Cath care

* To prevent an air embolism when a central line is open to air, position client in Trendelenburg position. OR * Have client perform Valsalva maneuver if there is no slide clamp on the line.

Transurethal resection of the prostate gland (TURP)

* Tx for hypertrophy of the prostate. * Prostate is removed by endoscopy. INTERVENTIONS: * Inform client about bladder spasms after procedure. * Maintain patent urinary drainage system (large 3 way indwelling catheter w/ a 30mL balloon) to decrease spasms. * Provide pain relief as prescribed (analgesics, narcotics, & antispasmodics). * Minimize catheter manipulation by taping. * Check drainage for clots. * Irrigate bladder using ONLY sterile saline. * Normal drainage is reddish pink, clearing to light pink w/in 24hrs. Small to medium sized blood clots may be present. * Large amounts of bright, flank blood should be reported. * Monitor vital signs. * Monitor for decreasing Hgb & Hct indicating bleeding. * After catheter is removed, increase fluid intake to 3000mL/day.

Diabetic Ketoacidosis (DKA)

* Type 1 Diabetes. * A life-threatening complication of diabetes mellitus that develops when a severe insulin deficiency occurs. * Hyperglycemia progresses to ketoacidosis over a period of several hours to several days. * Acidosis occurs in clients with type 1 diabetes mellitus, persons with undiagnosed diabetes, & persons who stop prescribed treatment for diabetes.

Hyperosmolar Hyperglycemia Syndrome (HHS)

* Type 2 Diabetes * Extreme hyperglycemia w/o acidosis. * May result in dehydration or vascular collapse but does not include the acidosis component of diabetic ketoacidosis. * Onset is usually slow, taking from hours to days.

Mannitol: Interventions

* Used for short term therapy only!! * Filter urine & watch for crystals. * Crystallization in the vial may occur if drug is exposed to a low temperature. * Warm vial to dissolve the crystals. * Do NOT use for IV infusion if crystals are present & have not been dissolved. CONTRAINDICATED: * Cerebral hemorrhage * If no urine output (anuria). * If output is < 30ml/hr, accumulation can cause pulmonary edema & water intoxication. * Given w/ extreme caution to HF pt. * Discontinue if HF or Renal failure develop.

PICC line

* Used long-term IV therapy. * Basilic, Median cubital & Cephalic veins in the antecubital area can be used. * Catheter tip ends in the Subclavian vein or Superior vena cava. * A small amount of bleeding may occur at the time of insertion & may continue for 24 hrs, but bleeding thereafter is not expected. * Phlebitis is a common complication. * Insertion is below the heart level; therefore air embolism is not common.

What can cause Fluid volume deficit?

* Vomiting * Diarrhea * GI suctioning * Swaeting * Inadequate fluid intake. * Massive edema during 1st stage of major burn. * Ascites * Older adult forgeting to drink.

Nephrotic Syndrome: S/S

* Weight gain. * Periorbital & facial edema most prominent in the morning. * Leg, ankle, labial, or scrotal edema. * Urine output decreases; urine dark & frothy. * Ascites * BP normal or slightly decreased. * Lethargy, anorexia, & pallor. * Massive proteinuria. * Decreased serum protein (hypoproteinemia) & elevated serum lipid levels.

The nurse is caring for a client with AIDS who started Zidovidine ATZ. It would be important to assess

CBC

Adrenal gland disorders

*Addison's disease(adrenal cortex) *Cushing's disease(adrenal cortex) *Conn's syndrome (adrenal cortex) *Pheochromocytoma (adrenal medulla)

Parkinson's disease treatment

COMT inhibitors and Levadopa Carbidopa

An older client is admitted with a diagnosis of bacterial pneumonia. The nurse's assessment of the client will most likely reveal which sign/symptom?

Confusion and tachycardia

foramen ovale

Connects the left and right atria, allowing blood to flow directly from the right to the left side of the heart

ductus venosus

Connects the umbilical vein to the inferior vena cava, closes by day 3 of life and becomes a ligament

lightening

- fetus begins to settle into the pelvic inlet and engagement occurs

34. The patient has an order for each of the following inhalers. Which of the following should the nurse offer to the patient at the onset of an asthma attack? A. Albuterol (Proventil) B. Beclomethasone (Beclovent) C. Ipratropium bromide (Atrovent) D. Salmeterol (Serevent)

A. Albuterol (Proventil) Albuterol is a short-acting bronchodilator that should initially be given when the patient experiences an asthma attack.

What does GFR do in ARF?

Decrease

Heparin

...

Thromolytic Therapy protocol

...

emphysema

an abnormal condition of the pulmonary system, characterized by overinflation and destructive changes in alveolar walls.

Seconal 0.1 gram PRN at bedtime is prescribed for rest. The scored tablets are labeled grain 1.5 per tablet. How many tablets should the nurse plan to administer?

1 tablet

Which change in lab values indicates to the nurse that a client with rheumatoid arthritis may be experiencing an adverse effect of methotrexate (Mexate) therapy?

Decrease in hemoglobin

What is the the action of Anticholinergics?

Decrease oral secretions.

apnea

an absence of spontaneous respirations.

A student nurse is admin Maalox. The nurse should intervene if the student plans to admin the antacid

at the same time as Iron

Wound drainage: Interventions

1. Assess surgical site, drains, & wound dressings (serous drainage may occur from an incision, but if excessive bleeding occurs from the site, notify the physician). 2. Assess the skin for redness, abrasions, or breakdown that may have resulted from surgical positioning. 3. Monitor body temperature & wound for signs of infection. 4. Maintain a dry, intact dressing. 5. Change dressings as prescribed, noting the amount of bleeding or drainage, odor, & intactness of sutures or staples. 6. Wound drains should be patent; prepare to assist with the removal of drains (as prescribed by the physician) when the drainage amount becomes insignificant. 7. An abdominal binder may be prescribed for obese and debilitated individuals to prevent dehiscence of the incision.

Cirrhosis (Liver failure): Interventions

1. Elevate HOB to minimize SOB. 2. If ascites & edema are ABSENT & the client does not exhibit signs of impending coma, a high-protein diet supplemented with vitamins is prescribed. 3. Provide supplemental vitamins (B complex, vitamins A, C, & K, folic acid, and thiamine) as prescribed. 4. Restrict sodium intake & fluid intake as prescribed. 5. Initiate enteral feedings or parenteral nutrition as prescribed. 6. Administer diuretics as prescribed to treat ascites. 7. Monitor I &O and electrolyte balance. 8. Weigh client & measure abdominal girth daily. 9. Monitor LOC; assess for precoma state (tremors, delirium). 10. Monitor for asterixis, a coarse tremor characterized by rapid, nonrhythmic extensions and flexions in the wrist and fingers. 11. Monitor for fetor hepaticus, the fruity, musty breath odor of severe chronic liver disease. 12. Maintain gastric intubation to assess bleeding or esophagogastric balloon tamponade to control bleeding varices if prescribed. 13. Administer blood products as prescribed. 14. Monitor coagulation laboratory results; administer vitamin K if prescribed. 15. Administer antacids as prescribed. 16. Administer lactulose (Constulose, Enulose) as prescribed, which decreases the pH of the bowel, decreases production of ammonia by bacteria in the bowel, and facilitates the excretion of ammonia. 17. Administer antibiotics as prescribed to inhibit protein synthesis in bacteria & decrease the production of ammonia. 18. Avoid medications such as opioids, sedatives, and barbiturates and any hepatotoxic medications or substances. 19. Instruct the client about the importance of abstinence of alcohol intake. 20. Prepare the client for paracentesis to remove abdominal fluid. 21. Prepare the client for surgical shunting procedures if prescribed to divert fluid from ascites into the venous system.

T3 and T4 levels in hypothyroidism are increased or decreased?

Decreased

One day after a Billroth II surgery, a male client suddenly grabs his right chest and becomes pale and diaphoretic. Vital signs are assessed at blood pressure 100/80, pulse 110 beats/min, and respirations 36 breaths/min. What action is most important for the nurse to take?

Apply oxygen at 2 L per nasal cannula.

Lab findings for Fluid volume deficit & Fluid volume excess...

1.) FLUID VOLUME DEFICIT: * Elevated BUN & Cr. * Elevated Hgb & Hct. * Increased serum osmolarity (concentrated). 2.) FLUID VOLUME EXCESS: * Decreased BUN * Decreased Hgb & Hct. * Decreased serum osmolarity. * Decreased urine osmolarity & specific gravity.

EKG reading for HYPOkalemia & HYPERkalemia

1.) HYPOKALEMIA: * Prominent U waves * Depressed ST segment * Flat T waves. 2.) HYPERKALEMIA: * Prolonged PR interval * Wide QRS * Tall T waves

1.) ADDISON'S disease is characterized by the ____ of adrenal cortex hormones glucocorticoids & mineralcorticoids. 2.) CUSHING'S disease is characterized by _____ of glucocorticoids.

1.) Hyposecretion 2.) Hypersecretion

1.) Drugs that INCREASE preload? 2.) Drugs that DECREASE preload?

1.) INCREASE (BP): * Blood products * Crystalloids 2.) DECREASE (BP): * Morphine * Nitrates * Diuretics

1.) Drugs that INCREASE contractility? 2.) Drugs that DECREASE contractility?

1.) INCREASE: * Digoxin * Dobutamine 2.) DECREASE: * Beta-blockers (lol) * Calcium channel blockers

1.) Drugs that INCREASE afterload? 2.) Drugs that DECREASE afterload?

1.) INCREASE: * Vasopressors * Dopamine 2.) DECREASE: * Nitroprusside * ACE inhibitors (pril) * ARB (sartan)

How much medication should be given for an IM & SubQ injection?

1.) INTRAMUSCULAR (IM) * No more than 3 mL. 2.) SUBCUTANEOUS (SUBQ): * No more than 1 mL. *** Larger volumes are difficult for an injection site to absorb & if prescribed, need to be verified.*******

What is the action of Miotics & Mydriatics?

1.) MIOTICS: * Constrict the pupils. 2.) MYDRIATICS: * Dilate the pupils.

2 Types of Hypertension

1.) PRIMARY: * No known cause (idiopathic). 2.) SECONDARY: * Develops in response to an unindentifiable mechanism or another disease.

Maslow' Heirarchy of Needs

1.) Physical 2.) Safety 3.) Love & Belonging 4.) Esteem 5.) Self-actualization

1.) Signs of left ventricular failure are evident in the____? 2.) Signs of right ventricular failure are evident in the ___?

1.) Pulmonary system. 2.) Systemic circulation.

Cirrhosis (Liver failure)

1.) A chronic, progressive disease of the liver characterized by diffuse degeneration and destruction of hepatocytes. 2.) Repeated destruction of hepatic cells causes the formation of scar tissue. COMPLICATIONS: * Portal hypertension * Ascites * Bleeding esophageal varicies. * Coagulation defect. * Jaundice * Portal systemic encephalopathy * Hepatorenal syndrome.

Nephrotic Syndrome

1.) A kidney disorder characterized by: * Massive proteinuria * Hypoalbuminemia * Hypoproteinemia * Edema. 2.) Primary objectives of therapeutic management are to: * Reduce the excretion of urinary protein. * Maintain protein-free urine. * Reduce edema. * Prevent infection. * Minimize complications. 3.) Usually occurs between ages 2-3 yrs.

Classification of Heart Failure (HF)

1.) ACUTE HEART FAILURE: * Occurs suddenly. 2.) CHRONIC HEART FAILURE: * Develops over time; * However, a client with chronic heart failure can develop an acute episode.

What is the difference between Acute Renal Failure & Chronic Renal Failure?

1.) ACUTE RENAL FAILURE: * Often reversible. * Abrupt deterioration of kidney function. 2.) CHRONIC RENAL FAILURE: * Irreversible. * Slow deterioration of kidney function characterized by increasing BUN & Cr. * Eventally dialysis is required.

Differences in Addison's & Cushing's disease?

1.) ADDISON'S: * Increase Na+ intake. * Cortisone preparations. 2.) CUSHING'S: * Restrict Na+ * Watch for s/s of infection.

What is the difference between Delirium & Dementia?

1.) DELIRIUM: * Acute * Reversible 2.) DEMENTIA: * Gradual * Permanent

Evaluation of Suicide Intent

1.) DIRECTLY ASK CLIENT ABOUT INTENT: * " Do you ever think about harming yourself?". 2.) ASK ABOUT PLAN: * " Do you have a plan for harming yourself?". 3.) IDENTIFY METHOD CHOSEN: * The more lethal the higher probability. 4.) DETERMINE AVAILABILITY

Suicidal: Interventions

1.) EXPRESS CONCERN FOR CLIENT: * "I am very concerned that you want to hurt yourself." 2.) BE HONEST: * "I need to share this info with the staff so we can provide safety until your feeling better." 3.) OFFER HOPE: * "Your feeling bad at this moment, but these feelings will pass. We have medications & tx that can help you." 4.) STAY WITH CLIENT: * Never leave a suicidal client alone.

Renal lithiasis: Surgical Tx

1.) EXTRACORPOREAL SHOCK WAVE LITHOTRIPSY (ESWL): * Uses shock waves to break the stones into tiny pieces that are then passed in urine. * X-rays or ultrasound are used to help determine the position of the stone as well as to monitor the status of the stone during treatment. 2.) PERCUTANEOUS NEPHROLITHOTOMY: * When ESWL isn't effective, or the stone is very large, the kidney stone will be removed through a small incision in your back using an instrument called a nephroscope. 3.) URETEROSCOPIC STONE REMOVAL: * Used to remove a stone lodged in a ureter. * The stone is snared with a small instrument (ureteroscope) that's passed into the ureter through your bladder. * Ultrasound or laser energy can also be directed through the scope to shatter the stone. * These methods work especially well on stones in the lower part of the ureter. 4.) PARATHYROID SURGERY: * Some calcium stones are caused by overactive parathyroid glands, which are located on the four corners of your thyroid gland, just below your Adam's apple. * When these glands produce too much parathyroid hormone, your body's level of calcium can become too high, resulting in excessive excretion of calcium in your urine.

Normal FHR & Respirations

1.) FHR: * 120-160 2.) RESPIRATIONS * 30-60

Age baby can perform imitative speech

10-12 mths

Intraocular Pressure Range

10-21

Oliguria?

100 - 400ml/24 hrs

The nurse in a well baby clinic has assigned several children today. It would be appropriate to suggest follow up for the child who is

18 months old and needs support to ambulate

The community health nurse is caring for the following clients. It would be a priority for the nurse to initiate a multidisciplinary conference for the client who is

17 years old, has DM type 1, unemployed, and recent A1C is 13%

The nurse at a health clinic is screening male clients to testicular cancer. It would be priority for the nurse to teach self exams for what age group

17-35

umbilical cord contains?

2 arteries and 1 vein

The following pediatric unit has been informed the following clients are being admitted. The nurse should first plan to assess the client who is

2 year old, 100.8 temp, BP 68/44

TSH levels in hyperthyroidism are increased or decreased?

Decreased (body is trying to compensate)

If ARF event happens and a pt does not recover what can happen?

CRF

Needle size for administering Heparin

26-27 guage, 5/8 to 1/2 inch

pt who needs further evaluation

3 month old who doesn't smile when played with

Recovery phase of acute renal failure

3-12 months long, mild abnormalities, decreased concentration of urine

Client census is often used to determine staffing needs. Which method of obtaining census determination for a particular unit provides the best formula for determining long-range staffing patterns?

Average daily census

Surgical management for stroke

Carotid Endarterectomy - removing a clot from the carotids; craniotomy - remove the aneurysm

What is the correct procedure for performing an ophthalmoscopic examination on a client's right eye?

From a distance of 8 to 12 inches and slightly to the side, shine the light into the client's pupil.

The nurse is caring for a mechanically ventilated client who was declared brain dead. An Advance Directive is not documented in the medical record. It would be most appropriate to obtain consent for organ donation from the

Clients closest family member

rubella vaccine in pregnancy

not given! Live attenuated virus!!

The nurse is admitting a 20 year old with anorexia. The nurse should assess the client for

Lanugo type hair on body

Comprehensive assessment for an older adult

Functional abilities. The focus of a geriatric assessment is to determine the older clients functional abilities, so appropriate interventions can be planned and implemented to maintain and enhance independence.

GTPAL

G= total number of times the women has been pregnant T=number carried to term 37 weeks P+ pre term birth A=abortions/miscariages L=children living

Pathophysiology of a Hemorrhagic Stroke

ICP increases due to blood in the subarachnoid space - All about the pressure

What disease is associated with Hyperthyroidism?

Grave's disease

What levels of Calcium indicate a hypercalcemic crisis?

Greater than 14

The nurse is caring for a 7 year old with thrombocytopenia and is on protective precaution. Which would b a appropriate toy for the nurse to provide

A board game

Air embolism

A bolus of air enters the vein through an inadequately primed IV line, from a loose connection, during tubing change, or during removal of the IV. INTERVENTIONS: * Prime tubing with fluid before use, & monitor for any air bubbles in the tubing. * Secure all connections. * Replace the IV fluid before the bag or bottle is empty. * Monitor for signs of air embolism; if suspected, clamp the tubing, turn the client on the left side with the head of the bed lowered (Trendelenburg's position) to trap the air in the right atrium, & notify the physician.

rhabdomyolysis

A condition in which damaged skeletal muscle tissue breaks down rapidly. Breakdown products of damaged muscle cells are released into the bloodstream; some of these, such as the protein myoglobin, are harmful to the kidneys and may lead to kidney failure

During change of shift report, the charge nurse reviews the infusions being received by clients on the oncology unit. The client receiving which infusion should be assessed first?

A continuous epidural infusion of morphine

What is the 3rd leading cause of death and the primary cerebrovascular disorder?

A stroke

placenta

A structure that allows an embryo to be nourished with the mother's blood supply

Fat embolism

A syndrome in which fat globules migrate into the bloodstream & combine with platelets to form emboli. * Greatest risk in first 36 hrs. after fracture. S/S: * Confusion due to hypoxemia. * Respiratory distress. * Irriability * Fever * Petechiae INTERVENTIONS: * Notify Dr. STAT!! * Give O2. * Draw blood gases. * Assist w/ endotracheal intubation.

The nurse is preparing a teaching plan for healthy adults. Which individual is most likely to maintain optimum health?

A teacher whose blood glucose levels average 126 daily with oral antidiabetic drugs

Life is sacred

A terminally ill and depressed client with cancer. A nurse who believes

Reversible Ischemic Neurologic Deficit (RND)

A type of cerebral infarct whose clinical course lasts longer than 24 hours but less than 72 hours; brain imaging usually reveals an infarct. In between TIA and a big stroke. Harder to pinpoint a cerebral infarct.

The nurse is performing hourly neurologic checks for a client with a head injury. Which new assessment finding warrants the most immediate intervention by the nurse?

A unilateral pupil that is dilated and nonreactive to light

Home care for the stroke patient

Health promotion, prevention of future strokes, follow up care, medication teaching, safety measures, adaptive strategies, how to prevent contractures, how to prevent skin breakdown

57. When admitting a patient with the diagnosis of asthma exacerbation, the nurse will assess for which of the following potential triggers? (Select all that apply.) A. Exercise B. Allergies C. Emotional stress D. Decreased humidity

A,B,C Although the exact mechanism of asthma is unknown, there are several triggers that may precipitate an attack. These include allergens, exercise, air pollutants, respiratory infections, drug and food additives, psychologic factors, and GERD.

55. During admission of a patient diagnosed with non-small cell carcinoma of the lung, the nurse questions the patient related to a history of which of the following risk factors for this type of cancer? (Select all that apply.) A. Asbestos exposure B. Cigarette smoking C. Exposure to uranium D. Chronic interstitial fibrosis

A,B,C Non-small carcinoma is associated with cigarette smoking and exposure to environmental carcinogens, including asbestos and uranium. Chronic interstitial fibrosis is associated with the development of adenocarcinoma of the lung.

52. When assessing a patient's sleep-rest pattern related to respiratory health, the nurse would ask if the patient: (Select all that apply.) A. Has trouble falling asleep B. Awakens abruptly during the night C. Sleeps more than 8 hours per night D. Has to sleep with the head elevated

A,B,D The patient with sleep apnea may have insomnia and/or abrupt awakenings. Patients with cardiovascular disease (e.g., heart failure that may affect respiratory health) may need to sleep with the head elevated on several pillows (orthopnea). Sleeping more than 8 hours per night is not indicative of impaired respiratory health.

56. When admitting a 45-year-old female with a diagnosis of pulmonary embolism, the nurse will assess the patient for which of the following risk factors? (Select all that apply.) A. Obesity B. Pneumonia C. Hypertension D. Cigarette smoking

A,C,D Research has demonstrated an increased risk of pulmonary embolism in women associated with obesity, heavy cigarette smoking, and hypertension. Other risk factors include immobilization, surgery within the last 3 months, stroke, history of DVT, and malignancy.

54. To promote airway clearance in a patient with pneumonia, the nurse instructs the patient to do which of the following? (Select all that apply.) A. Splint the chest when coughing B. Maintain a semi-Fowler's position C. Maintain adequate fluid intake D. Instruct patient to cough at end of exhalation

A,C,D The nurse should instruct the patient to splint the chest while coughing. This will reduce discomfort and allow for a more effective cough. Maintaining adequate fluid intake liquefies secretions, allowing easier expectoration. Coughing at the end of exhalation promotes a more effective cough. The patient should be positioned in an upright sitting position (high-Fowler's) with head slightly flexed.

Which content about self-care should the nurse include in the teaching plan of a client who has genital herpes? (Select all that apply.)

A. Encourage annual physical and Pap smear. B. Take antiviral medication as prescribed. C. Use condoms to avoid transmission to others. D. Warm sitz baths may relieve itching.

29. The physician has prescribed salmeterol (Serevent) for a patient with asthma. In reviewing the use of dry powder inhalers (DPIs) with the patient, the nurse should provide which of the following instructions? A. "Close lips tightly around the mouthpiece and breathe in deeply and quickly." B. "To administer a DPI, you must use a spacer that holds the medicine so that you can inhale it." C. "Hold the inhaler several inches in front of your mouth and breathe in slowly, holding the medicine as long as possible." D. "You will know you have correctly used the DPI when you taste or sense the medicine going into your lungs."

A. "Close lips tightly around the mouthpiece and breathe in deeply and quickly." Dry powder inhalers do not require spacer devices. The patient should be instructed to breathe in deeply and quickly to ensure medicine moves down deeply into lungs. The patient may not taste or sense the medicine going into the lungs.

29. Which of the following instructions are most appropriate in the home management of a patient who has undergone surgery for oral cancer? A. "You should drink plenty of fluids and eat foods you enjoy." B. "It is normal to have some leakage of saliva from the suture line." C. "Lying in a prone position helps decrease swelling at the suture line." D. "You should avoid foods high in protein while your suture line is healing."

A. "You should drink plenty of fluids and eat foods you enjoy." For patients who have undergone treatment for head and neck cancers, maintaining adequate nutrition is a challenge. The nurse encourages the patient to increase fluids to prevent dehydration and liquefy secretions. These patients are more likely to eat foods that they enjoy and can tolerate.

4. When caring for a patient who is 3 hours postoperative laryngectomy, the nurse's highest priority assessment would be: A. Airway patency B. Patient comfort C. Incisional drainage D. Blood pressure and heart rate

A. Airway patency Remember ABCs with prioritization. Airway patency is always the highest priority and is essential for a patient undergoing surgery surrounding the upper respiratory system.

32. Which of the following test results identify that a patient with an asthma attack is responding to treatment? A. A decreased exhaled nitric oxide B. An increase in CO2 levels C. A decrease in white blood cell count D. An increase in serum bicarbonate levels

A. A decreased exhaled nitric oxide. Nitric oxide levels are increased in the breath of people with asthma. A decrease in the exhaled nitric oxide concentration suggests that the treatment may be decreasing the lung inflammation associated with asthma.

23. A patient is having inspiratory stridor (crowing respiration) and the nurse suspects he is experiencing a laryngospasm. Which of the following would be most appropriate to implement for a patient experiencing a laryngospasm? A. Administer 100% oxygen. B. Position the patient in high Fowler's position. C. Insert a 16-gauge (large-bore) IV needle. D. Activate the emergency response team (code blue team) to the patient's room.

A. Administer 100% oxygen.A nurse should immediately administer 100% oxygen to the patient until the airway is fully reestablished, the larynx relaxes, and the spasms stop. Activating the emergency response team is not an immediate nursing action at this time because the nurse can administer the oxygen without the assistance of others. Positioning the patient in high Fowler's will not address the patient's need for immediate reoxygenation because of the patient's compromised respiratory state. Insertion of an IV device is not the first priority response but should be implemented after the nurse has assessed that the airway is stable.

21. A 45-year-old man with asthma is brought to the emergency department by automobile. He is short of breath and appears frightened. During the initial nursing assessment, which of the following clinical manifestations might be present as an early symptom during an exacerbation of asthma? A. Anxiety B. Cyanosis C. Hypercapnia D. Bradycardia

A. Anxiety An early symptom during an asthma attack is anxiety because he is acutely aware of the inability to get sufficient air to breathe. He will be hypoxic early on with decreased PaCO2 and increased pH as he is hyperventilating.

3. Following a patient's bone marrow aspiration, which of the following nursing interventions should a nurse anticipate? A. Application of firm pressure to the site B. Positioning the patient in a prone position C. Positioning the patient in a supine position D. Application of a warm, moist compress to the site

A. Application of firm pressure to the site After a bone marrow aspiration procedure, a nurse should apply pressure to the aspiration site until bleeding stops. Application of a warm, moist compress will not alter the potential for bleeding. Positioning the patient to assume a supine or prone position also will not address the need to control bleeding from the aspiration site.

5. When initially teaching a patient the supraglottic swallow following a radical neck dissection, with which of the following foods should the nurse begin? A. Cola B. Applesauce C. French fries D. White grape juice

A. ColaWhen learning the supraglottic swallow, it may be helpful to start with carbonated beverages because the effervescence provides clues about the liquid's position. Thin, watery fluids should be avoided because they are difficult to swallow and increase the risk of aspiration. Nonpourable pureed foods, such as applesauce, would decrease the risk of aspiration, but carbonated beverages are the better choice to start with.

3. When caring for a patient with metastatic cancer, the nurse notes a hemoglobin level of 8.7 g/dl and hematocrit of 26%. The nurse would place highest priority on initiating interventions that will reduce which of the following? A. Fatigue B. Thirst C. Headache D. Abdominal pain

A. Fatigue The patient with a low hemoglobin and hematocrit (normal values approximately 13.5% to 17% and 40% to 54%, respectively) is anemic and would be most likely to experience fatigue. This symptom develops because of the lowered oxygen-carrying capacity that leads to reduced tissue oxygenation to carry out cellular functions.

6. The nurse is caring for a patient admitted to the hospital with pneumonia. Upon assessment, the nurse notes a temperature of 101.4° F, a productive cough with yellow sputum and a respiratory rate of 20. Which of the following nursing diagnosis is most appropriate based upon this assessment? A. Hyperthermia related to infectious illness B. Ineffective thermoregulation related to chilling C. Ineffective breathing pattern related to pneumonia D. Ineffective airway clearance related to thick secretions

A. Hyperthermia related to infectious illness Because the patient has spiked a temperature and has a diagnosis of pneumonia, the logical nursing diagnosis is hyperthermia related to infectious illness. There is no evidence of a chill, and her breathing pattern is within normal limits at 20 breaths per minute. There is no evidence of ineffective airway clearance from the information given because the patient is expectorating sputum.

49. The nurse evaluates that a patient is experiencing the expected beneficial effects of ipratropium (Atrovent) after noting which of the following assessment findings? A. Increased peak flow readings B. Increased level of consciousness C. Decreased sputum production D. Increased respiratory rate

A. Increased peak flow readings. Ipratropium is a bronchodilator that should lead to increased PEFRs.

1. When assessing a patient's nutritional-metabolic pattern related to hematologic health, the nurse would: A. Inspect the skin for petechiae. B. Ask the patient about joint pain. C. Assess for vitamin C deficiency. D. Determine if the patient can perform ADLs.

A. Inspect the skin for petechiae. Any changes in the skin's texture or color should be explored when assessing the patient's nutritional-metabolic pattern related to hematologic health. The presences of petechiae or ecchymotic areas could be indicative of hematologic deficiencies related to poor nutritional intake or related causes.

35. The nurse who has administered a first dose of oral prednisone (Deltasone) to the patient with asthma writes on the care plan to begin monitoring which of the following patient parameters? A. Intake and output B. Bowel sounds C. Apical pulse D. Deep tendon reflexes

A. Intake and output Corticosteroids such as prednisone can lead to fluid retention. For this reason, it is important to monitor the patient's intake and output.

8. While obtaining the admission assessment data, which of the following characteristics would a nurse expect a patient with anemia to report? A. Palpitations B. Blurred vision C. Increased appetite D. Feeling of warm flushing sensation

A. Palpitations Patients experiencing moderate anemia (hemoglobin [Hb] 6 to 10 g/dL) may experience dyspnea (shortness of breath), palpitations, diaphoresis (profound perspiration) with exertion, and chronic fatigue. Blurred vision is associated in patients experiencing profound anemia states. Anorexia is common in patients with severe anemia, as well. Patients with anemia often appear pale and complain of feeling cold because of compensatory vasoconstriction of the subcutaneous capillaries.

45. The nurse reviews pursed lip breathing with a patient newly diagnosed with emphysema. The nurse reinforces that this technique will assist respiration by which of the following mechanisms? A. Preventing bronchial collapse and air trapping in the lungs during exhalation B. Increasing the respiratory rate and giving the patient control of respiratory patterns C. Loosening secretions so that they may be coughed up more easily D. Promoting maximal inhalation for better oxygenation of the lungs

A. Preventing bronchial collapse and air trapping in the lungs during exhalation The focus of pursed lip breathing is to slow down the exhalation phase of respiration, which decreases bronchial collapse and subsequent air trapping in the lungs during exhalation.

10. If a nurse is caring for an 80-year-old patient with a temperature of 100.4° F, crackles at the right lung base, pain with deep inspiration, and dyspnea, which of the following orders is the nurse's priority? A. Sputum specimen for culture and sensitivity B. Codeine 15 mg orally every 6 hours as needed C. Incentive spirometer every 2 hours while awake D. Amoxicillin (Amoxil) 500 mg orally 4 times a day

A. Sputum specimen for culture and sensitivity The patient presents with signs of a respiratory infection. To initiate the most effective therapy, the health care prescriber must know the pathogen causing the infection. Therefore, the sputum specimen is the nurse's priority. If the antibiotic is administered before the specimen is obtained, the results of the culture might not be as accurate and could impair the effectiveness of therapy. After the specimen is obtained, the nurse can administer codeine for coughing and begin the incentive spirometry to mobilize secretions and improve the patient's ability to expectorate the secretions.

9. Before beginning a transfusion of RBCs, which of the following actions by the nurse would be of highest priority to avoid an error during this procedure? A. Check the identifying information on the unit of blood against the patient's ID bracelet. B. Select new primary IV tubing primed with lactated Ringer's solution to use for the transfusion. C. Add the blood transfusion as a secondary line to the existing IV and used the IV controller to maintain correct flow. D. Remain with the patient for 60 minutes after beginning the transfusion to watch for signs of a transfusion reaction. The patient's identifying information (name, date of birth, medical record number) on the identification bracelet should exactly match the information on the blood bank tag that has been placed on the unit of blood. If any information does not match, the transfusions should not be hung because of possible error and risk to the patient.

A. The patient's identifying information (name, date of birth, medical record number) on the identification bracelet should exactly match the information on the blood bank tag that has been placed on the unit of blood. If any information does not match, the transfusions should not be hung because of possible error and risk to the patient.

19. The nurse is caring for a postoperative patient with sudden onset of respiratory distress. The physician orders a STAT ventilation-perfusion scan. Which of the following explanations should the nurse provide to the patient about the procedure? A. This test involves injection of a radioisotope to outline the blood vessels in the lungs, followed by inhalation of a radioisotope gas. B. This test will use special technology to examine cross sections of the chest with use of a contrast dye. C. This test will use magnetic fields to produce images of the lungs and chest. D. This test involves injecting contrast dye into a blood vessel to outline the blood vessels of the lungs.

A. This test involves injection of a radioisotope to outline the blood vessels in the lungs, followed by inhalation of a radioisotope gas.A ventilation-perfusion scan has two parts. In the perfusion portion, a radioisotope is injected into the blood and the pulmonary vasculature is outlined. In the ventilation part, the patient inhales a radioactive gas that outlines the alveoli.

27. A patient with acute exacerbation of COPD needs to receive precise amounts of oxygen. Which of the following types of equipment should the nurse prepare to use? A. Venturi mask B. Partial non-rebreather mask C. Oxygen tent D. Nasal cannula

A. Venturi mask The Venturi mask delivers precise concentrations of oxygen and should be selected whenever this is a priority concern. The other methods are less precise in terms of amount of oxygen delivered.

6. A nurse is working on a respiratory care unit where many of the patients are affected by asthma. Which of the following actions by the nurse would most likely increase respiratory difficulty for the patients? A. Wearing perfume to work B. Encouraging patients to ambulate daily C. Allowing the patients to eat green leafy vegetables D. Withholding antibiotic therapy until cultures are obtained

A. Wearing perfume to work People with asthma should avoid extrinsic allergens and irritants (e.g., dust, pollen, smoke, certain foods, colognes and perfumes, certain types of medications) because their airways become inflamed, producing shortness of breath, chest tightness, and wheezing. Many green leafy vegetables are rich in vitamins, minerals, and proteins, which incorporate healthy lifestyle patterns into the patients' daily living routines. Routine exercise is a part of a prudent lifestyle, and for patients with asthma the physical and psychosocial effects of ambulation can incorporate feelings of well-being, strength, and enhancement of physical endurance. Antibiotic therapy is always initiated after cultures are obtained so that the sensitivity to the organism can be readily identified.

24. The nurse is caring for a patient with an acute exacerbation of asthma. Following initial treatment, which of the following findings indicates to the nurse that the patient's respiratory status is improving? A. Wheezing becomes louder B. Vesicular breath sounds decrease C. Aerosol bronchodilators stimulate coughing D. The cough remains nonproductive

A. Wheezing becomes louder The primary problem during an exacerbation of asthma is narrowing of the airway and subsequent diminished air exchange. As the airways begin to dilate, wheezing gets louder because of better air exchange.

28. To find the infection site associated with acute lymphangitis, the nurse should look _____ to the inflammation. A. distal B. anterior C. proximal D. contralateral

A. distal The nurse should assess distal to swelling to locate the initial site of infection. Examining proximal, contralateral, or anterior to the inflammation does not describe swelling associated with infection.

12. A nurse is providing care to an adult female patient and observes that the Hb laboratory analysis result is 9 g/dl. Based on this finding, the nurse should expect to observe A. dyspnea. B. bradycardia. C. warm, dry skin. D. activity tolerance without complaint of fatigue.

A. dyspnea.Hb levels are used to determine the severity of anemia. Patients with moderate anemia (Hb 6 to 10 g/dL) may suffer from dyspnea, palpitations, diaphoresis with exertion, and chronic fatigue. Patients who are anemic usually have cool skin related to compensatory mechanism of mild vasoconstriction. Patients who are anemic experience tachycardia because of increased demands placed on the heart to meet overall metabolic requirements. Activity tolerance without complaint is not correct because patients with anemic conditions fatigue readily.

35. A patient is admitted to the hospital with fever, chills, a productive cough with rusty sputum, and pleuritic chest pain. Pneumococcal pneumonia is suspected. An appropriate nursing diagnosis for the patient based on the patient's manifestations is A. hyperthermia related to acute infectious process. B. chronic pain related to ineffective pain management. C. risk for injury related to disorientation and confusion. D. ineffective airway clearance related to retained secretions.

A. hyperthermia related to acute infectious process. The patient with pneumococcal pneumonia is acutely ill with fever and the systemic manifestations of fever, such as chills, thirst, headache, and malaise. Interventions that monitor temperature and aid in lowering body temperature are appropriate. Ineffective airway clearance would be manifested by adventitious breath sounds and difficulty producing secretions. Disorientation and confusion are not noted in this patient and are not typical unless the patient is very hypoxemic. Pleuritic pain is an acute pain that is due to inflammation of the pleura.

16. To ensure the correct amount of oxygen delivery for a patient receiving 35% oxygen via a Venturi mask, it is most important that the nurse A. keep the air-entrainment ports clean and unobstructed. B. apply an adaptor to increase humidification of the oxygen. C. drain moisture condensation from the oxygen tubing every hour. D. keep the flow rate high enough to keep the bag from collapsing during inspiration.

A. keep the air-entrainment ports clean and unobstructed. Oxygen is delivered to a small jet in the center of a wide-based cone. Air is entrained (pulled through) openings in the cone as oxygen flows through the small jet. The degree of restriction or narrowness of the jet determines the amount of entrainment and the dilution of pure oxygen with room air and thus the concentration of oxygen. Although applying an adaptor can increase the humidification with the Venturi mask, it is not the best answer, because an open port is essential to proper functioning. Draining moisture condensation from the oxygen tubing is performed as often as needed, not on an hourly schedule. A plastic face mask with a reservoir bag needs to have sufficient flow rate to keep the bag inflated.

Pt to assess first

ALS that begins using assessor muscles to breath

20. Using light pressure with the index and middle fingers, the nurse cannot palpate any of the patient's superficial lymph nodes. The nurse A. records this finding as normal. B. should reassess the lymph nodes using deeper pressure. C. asks the patient about any history of any radiation therapy. D. notifies the health care provider that x-rays of the nodes will be necessary.

A. records this finding as normal. Superficial lymph nodes are evaluated by light palpation, but they are not normally palpable. It may be normal to find small (<1.0 cm), mobile, firm, nontender nodes. Deep lymph nodes are detected radiographically.

17. While caring for a patient with respiratory disease, a nurse observes that the oxygen saturation drops from 94% to 85% when the patient ambulates. The nurse should determine that A. supplemental oxygen should be used when the patient exercises. B. ABG determinations should be done to verify the oxygen saturation reading. C. this finding is a normal response to activity and that the patient should continue to be monitored. D. the oximetry probe should be moved from the finger to the earlobe for an accurate oxygen saturation measurement during activity.

A. supplemental oxygen should be used when the patient exercises.An oxygen saturation lower than 90% indicates inadequate oxygenation. If the drop is related to activity of some type, supplemental oxygen is indicated.

3. The nurse notices clear nasal drainage in a patient newly admitted with facial trauma, including a nasal fracture. The nurse should: A. test the drainage for the presence of glucose. B. suction the nose to maintain airway clearance. C. document the findings and continue monitoring. D. apply a drip pad and reassure the patient this is normal.

A. test the drainage for the presence of glucose. Clear nasal drainage suggests leakage of cerebrospinal fluid (CSF). The drainage should be tested for the presence of glucose, which would indicate the presence of CSF.

A male client who has never smoked but has had COPD for the past 5 years is now being assessed for cancer of the lung. The nurse knows that he is most likely to develop which type of lung cancer?

Adenocarcinoma

43. Select all that apply. Which of the following is included in a comprehensive respiratory assessment? A. Pulse oximetry B. Chest auscultation C. Apical radial pulse D. Nail-bed assessment E. Evaluation of respiratory effort F. Rate and character of respirations

ABDEF The total assessment of the respiratory system includes pulse oximetry; auscultation; skin and nail-bed assessment for the detection of cyanosis; and rate, character, and degree of effort of respirations. The apical radial pulse is a cardiac assessment.

42. Select all that apply. Which of the following nursing actions can help clear tracheobronchial secretions in a patient with cystic fibrosis? A. Postural drainage B. Suppressing the cough C. Ensuring adequate hydration D. Administering mucolytic aerosols E. Encouraging the patient to lie flat F. Administering water-soluble vitamins

ACD Postural drainage, adequate hydration, and administration of mucolytic aerosols all encourage coughing and the clearing of secretions. A patient with cystic fibrosis will be more comfortable sitting upright.

41. Select all that apply. During initial assessment, a nurse should record which of the following manifestations of respiratory distress? A. Tachypnea B. Nasal flaring C. Thready pulse D. Panting or grunting E. Use of intercostal muscles F. An inspiratory-to-expiratory ratio of 1:2

AD Manifestations of respiratory distress include tachypnea, grunting and panting on respiration, central cyanosis, use of accessory muscles, and flaring nares.

Nursing considerations when admin heparin

ADMIN in ABD, do not massage, protamine sulfate is antidote

What does Acute Respiratory Failure lead to if untreated?

ARDS

What will be common with dialysis, especially peritoneal dialysis?

Abdominal pain

Peptic Ulcer Disease (PUD)

Abdominal pain relief that happens when foods are eaten on an empty stomach

The nurse witnesses a baseball player receive a blunt trauma to the back of the head with a softball. What assessment data should the nurse collect immediately?

Ability to spontaneously open the eyes before any tactile stimuli are given

What is Acute Renal Failure?

Abrupt loss of kidney function

Uremia

Accumulation of waste products from protein metabolism.

What disease is progressive and sudden?

Acute Respiratory Failure

Nursing concern for pleurisy?

Acute pain

A 74-year-old male client is admitted to the ICU with a diagnosis of respiratory failure secondary to pneumonia. Currently, he is ventilator-dependent with settings of tidal volume (VT) 750 ml and intermittent mandatory ventilation (IMV) rate of 10. ABG results are pH 7.48; PaCO2 30; PaO2 64; HCO3 25; and FiO2 0.80. Which intervention should the nurse implement first?

Add 5 cm positive end-expiratory pressure (PEEP).

MAP

Add SBP to twice the DBP and divide by 3

hospital-acquired pneumonia

pneumonia occurring 48 hours or longer after hospital admission and not incubating at the time of hospitalization.

A central venous catheter has been inserted via a jugular vein and a radiograph has confirmed placement of the catheter. A prescription has been received for a stat medication, but IV fluids have not yet been started. What action should the nurse take prior to administering the prescribed medication?

Administer a bolus of normal saline solution

A client with hypertension has been receiving ramipril (Altace) 5 mg PO daily for 2 weeks and is scheduled to receive a dose at 0900. At 0830 the client's blood pressure is 120/70. What action should the nurse take?

Administer the dose as prescribed.

Why does a patient need to be weened from steroids?

Adrenal insufficiency

Addisons Disease

Adrenal insufficiency is an autoimmune dysfunction that results from WBC damaging the adrenal cortex

A 58-year-old female client tells the nurse that she feels a sense of loss since she has stopped having menstrual periods. She then states, "At least I will no longer have to suffer through those horrible Pap smear tests every year." Which action should the nurse implement?

Advise the client that Pap smear tests should be continued.

When are paradoxical chest movements seen?

After a chest trauma, the 2 sides won't move together, this is an emergency

NGT

After abdominal surgery, ptency of the NGT should be maintained to avoid the need to reinsert the tube, which could possibility perforate the surgical repair site, so irrigation of the NGT should be implemented to promote gastric drainage and decompression.

Non modifiable risk factors for strokes

Age, Sex, Race

In taking health histories of the following individuals, which client would have the greatest potential for development of head and neck cancer?

An alcoholic, tobacco-chewing auctioneer

surfactant

An amphipathic molecule secreted by cells in the alveoli (type 2 alveolar cells) tha reducs surface tension on the inside of the alveolar walls. This prevents the alveoli from collapsing upon exhale and sticking together, thus reducing the effort required for inspiration.

The nurse is reviewing the routine medications taken by a client with chronic angle closure glaucoma. Which medication prescription should the nurse question?

An anticholinergic with a side effect of pupillary dilation

A client schedule for a femoro-popliteal bypass surgery

An elective surgical procedure can be rescheduled for a later date

When assigning clients on a medical-surgical floor to a registered nurse (RN) and a practical nurse (PN), it is best for the charge nurse to assign which client to the PN?

An older adult client with pneumonia and viral meningitis

What is a side effect of Synthroid in the elderly?

Angina

The nurse notes that a client who is scheduled for surgery the next morning has an elevated blood urea nitrogen (BUN) level. Which client condition is most likely to have contributed to this finding?

Anorexia and vomiting for the past 2 days

Ondansetron (Zofran)

Antiemetic Drug * Given for N/V from chemotherapy, radiation, & surgery. SIDE EFFECTS: * Drowsiness * Dizziness * Bradycardia * Hypotension * Vertigo INTERVENTIONS: * Change positions slowly. * Protect from injury.

The nurse is caring for a client who has bumetanide (Bumex) prescribed. The nurse should suggest that the client include which of the following foods in the diet

Apricots

The nurse is preparing a 45-year-old female client for discharge from a cancer center following ileostomy surgery for colon cancer. Which discharge goal should the nurse include in this client's discharge plan?

Attend an ostomy support group within 2 weeks.

An older male client comes to the geriatric screening clinic complaining of pain in his left calf. The nurse notices a reddened area on the calf of his right leg that is warm to the touch and the nurse suspects that the client may have thrombophlebitis. Which additional assessment is most important for the nurse to perform?

Auscultate the client's breath sounds.

What is PEEP?

It opens the alveoli and keeps them from collapsing and improves gas exchange to the capillary bed

For the client with Pheochromocytoma...

Avoid stimuli that can precipitate a hypertensive crisis such as: * Increased abdominal pressure. * Vigorous abdominal palpation.

Client teaching for Hypothyroidism?

Avoiding sedatives

Client teaching for Hyperthyroidism?

Avoiding the use of stimulants such as caffeine containing foods and drugs, avoid bulky foods/fiber

Hospice

Comfort and pain management using an effective analgesic-sedative such as morphine is the most important standard of care therapy in hospice home care to ensure comfort and enhance the quality of life for a client with a life expectancy of less than 6 months.

Fundamentals The nurse is providing comfort and pallative care for a terminally ill client who is experiencing nausea and vomiting. Which action is best for the nurse to take to promote the clients comfort?

B) Offer high protein foods; Measures to manage nausea and vomiting include the use of antiemetics and avoiding foods and liquids that increase stomach acidity, such as coffee, milk, and citrus acid juices. For some clients, an empty stomach exacerbates the nausea, so offering frequent, small amounts of foods that appeal to the client, such as dry cracker or bland, high protein foods, help maintain nutritional status. A &C) Increase fluid intake and provide a high residue diet may help prevent constipation or diarrhea, the best action is to meet the clients basic needs for hydration and nutrition. D) Give prompt mouth care is a comfort measure that minimizes nausea, the presence of protein in the stomach may be more effective.

Psychiatric Mental Health; What action should the hopice nurse implement to assist a client maintain self worth during the end of life process?

B) Plan regular visits with the client thoughout the day. Planning regular visits helps the client maintain a sense of self worth because it demostrates that the client is worthy of others and the nurse time and attention. A,D) a grief counselor and a spiritiual advisor may be requested but do not help maintain a clients sense of self worth because it may imply that the nurse is too busy to be present for the client. Although some clients may choose to finalize personal affairs alone, a hospice client may experience feelings of abandoment and isolation if left alone during the end of life process and should have frequent contact with the nurse and or family members.

Maternity: The neonatologist requests a mother to provide breast milk for her 32 week gestational premature newborn. The nurse provides instructions about pumping, storing, and transporting the brest milk. Which additional information should the nurse include to ensure the mother understands the request?

B) Providing breast milk ensures the premature newborn can easily digest absorb the nutrients Breast milk, rather than formula, provides antibodies and nutrition that is easily digested and readily absorbed by an immature newborn. Breast milk can be frozen and used if the mother is unable to provide breast milk every day so

Brain Stem

Breathing, Heart Rate, Blood Pressure, Movement & sensation, for head, neck, eyes, hearing, relays messages for other movements and sensations

53. A patient is being discharged from the emergency department after being treated for epistaxis. In teaching the family first aid measures in the event the epistaxis would recur, which of the following measures would the nurse suggest? (Select all that apply.) A. Tilt patients head backwards B. Apply ice compresses to the nose C. Pinch the entire soft lower portion of the nose D. Partially insert a small gauze pad into the bleeding nostril

B,C,D First aid measures to control epistaxis includes placing the patient in a sitting position, leaning forward. Tilting the head back does not stop the bleeding, but rather allows the blood to enter the nasopharynx, which could result in aspiration or nausea/vomiting from swallowing blood. All of the other options are appropriate first aid treatment of epistaxis.

The nurse is planning the care for a client who is admitted with the syndrome of inappropriate antidiuretic hormone secretion (SIADH). Which interventions should the nurse include in this client's plan of care? (Select all that apply.)

B. Quiet environment C. Deep tendon reflex assessments D. Neurologic checks E. Daily weights

50. The nurse is teaching a patient how to self-administer ipratropium (Atrovent) via a metered dose inhaler. Which of the following instructions given by the nurse is most appropriate to help the patient learn proper inhalation technique? A. "Avoid shaking the inhaler before use." B. "Breathe out slowly before positioning the inhaler." C. "After taking a puff, hold the breath for 30 seconds before exhaling." D. "Using a spacer should be avoided for this type of medication."

B. "Breathe out slowly before positioning the inhaler." It is important to breathe out slowly before positioning the inhaler. This allows the patient to take a deeper breath while inhaling the medication thus enhancing the effectiveness of the dose.

47. A patient has been receiving oxygen per nasal cannula while hospitalized for COPD. The patient asks the nurse whether oxygen use will be needed at home. Which of the following would be the most appropriate response by the nurse? A. "Long-term home oxygen therapy should be used to prevent respiratory failure." B. "Oxygen will be needed when your oxygen saturation drops to 88% and you have symptoms of hypoxia. C. "Long-term home oxygen therapy should be used to prevent heart problems related to emphysema." D. "Oxygen will not be needed until or unless you are in the terminal stages of this disease."

B. "Oxygen will be needed when your oxygen saturation drops to 88% and you have symptoms of hypoxia.Long-term oxygen therapy in the home should be considered when the oxygen saturation is 88% or less and the patient has signs of tissue hypoxia, such as cor pulmonale, erythrocytosis, or impaired mental status.

9. If a health care provider is planning to transfuse a patient with a unit of packed red blood cells, which of the following solutions should the health care provider hang with the transfusion? A. 5% dextrose in water B. 0.9% sodium chloride C. 5% dextrose in 0.9% sodium chloride D. 5% dextrose in lactated Ringer's solution

B. 0.9% sodium chloride The only solution appropriate for administration with whole blood or blood products is 0.9% sodium chloride. The other options are not appropriate for use with blood products.

expressive aphasia

Broca's

7. The nurse notes a physician's order written at 10:00 AM for 2 units of packed red blood cells to be administered to a patient who is anemic secondary to chronic blood loss. If the transfusion is picked up at 11:30, the nurse should plan to hang the unit no later than which of the following times? A. 11:45 AM B. 12:00 noon C. 12:30 PM D. 3:30 PM

B. 12:00 noon The nurse must hang the unit of packed red blood cells within 30 minutes of signing them out from the blood bank

40. The patient has an order for albuterol 5 mg via nebulizer. Available is a solution containing 2 mg/ml. How many milliliters should the nurse use to prepare the patient's dose? A. 0.2 B. 2.5 C. 3.75 D. 5.0

B. 2.5

2. The nurse is caring for a patient with COPD and pneumonia who has an order for arterial blood gases to be drawn. Which of the following is the minimum length of time the nurse should plan to hold pressure on the puncture site? A. 2 minutes B. 5 minutes C. 10 minutes D. 15 minutes

B. 5 minutes Following obtaining an arterial blood gas, the nurse should hold pressure on the puncture site for 5 minutes by the clock to be sure that bleeding has stopped. An artery is an elastic vessel under higher pressure than veins, and significant blood loss or hematoma formation could occur if the time is insufficient.

4. The nurse is caring for a patient who is to receive a transfusion of two units of packed red blood cells. After obtaining the first unit from the blood bank, the nurse would ask which of the following health team members in the nurses' station to assist in checking the unit before administration? A. Unit secretary B. Another registered nurse C. A physician's assistant D. A phlebotomist

B. Another registered nurseBefore hanging a transfusion, the registered nurse must check the unit with another RN or with a licensed practical (vocational) nurse, depending on agency policy.

24. A nurse is preparing to establish oxygen therapy for a patient with COPD, and the physician's prescription reads "oxygen per nasal cannula at 5 L per minute." Which of the following actions should the nurse take? A. Administer the oxygen as prescribed. B. Call the physician and question the correct flow rate of the oxygen. C. Establish the oxygen as prescribed and obtain an ABG. D. Change the delivery device from a nasal cannula to a simple oxygen mask.

B. Call the physician and question the correct flow rate of the oxygen. The nurse should call the physician immediately and question the flow rate for delivery of the oxygen before implementation. Oxygen is used cautiously in patients with COPD because of longstanding hypoxemia serving as the respiratory drive mechanism. If high levels of oxygen are administered, the respiratory drive can be obliterated. Changing the device to a simple oxygen mask may alter the oxygen concentration being delivered to the patient and will further enhance the obliteration of the patient's respiratory drive. Obtaining an ABG sample is not a priority at this time, and the action does not address the validity of the prescribed oxygen dosing for the patient.

20. If a patient states, "It's hard for me to breathe and I feel short-winded all the time," what is the most appropriate terminology to be applied in documenting this assessment by a nurse? A. Apnea B. Dyspnea C. Tachypnea D. Respiratory fatigue

B. Dyspnea Dyspnea is a subjective description reflective of the patient's statement indicating difficulty in breathing. Apnea refers to absence of breath or breathing. Tachypnea refers to an increased rate of breathing, usually greater than 20 breaths per minute. Respiratory fatigue is subjective and usually refers to the patient exhibiting signs and symptoms associated with a comprehensive respiratory assessment including laborious breathing, use of accessory muscles, and slowing of respirations.

9. Which of the following nursing interventions is of the highest priority in helping a patient expectorate thick secretions related to pneumonia? A. Humidify the oxygen as able B. Increase fluid intake to 3L/day if tolerated. C. Administer cough suppressant q4hr. D. Teach patient to splint the affected area.

B. Increase fluid intake to 3L/day if tolerated. Although several interventions may help the patient expectorate mucus, the highest priority should be on increasing fluid intake, which will liquefy the secretions so that the patient can expectorate them more easily. Humidifying the oxygen is also helpful, but is not the primary intervention. Teaching the patient to splint the affected area may also be helpful, but does not liquefy the secretions so that they can be removed.

8. The nurse receives a physician's order to transfuse fresh frozen plasma to a patient suffering from an acute blood loss. Which of the following procedures is most appropriate for infusing this blood product? A. Hand the fresh frozen plasma as a piggyback to a new bag of primary IV solution without KCl. B. Infuse the fresh frozen plasma as rapidly as the patient will tolerate. C. Hang the fresh frozen plasma as a piggyback to the primary IV solution. D. Infuse the fresh frozen plasma as a piggyback to a primary solution of normal saline.

B. Infuse the fresh frozen plasma as rapidly as the patient will tolerate. The fresh frozen plasma should be administered as rapidly as possible and should be used within 2 hours of thawing. Fresh frozen plasma is infused using any straight-line infusion set. Any existing IV should be interrupted while the fresh frozen plasma is infused, unless a second IV line has been started for the transfusion.

2. Which of the following is a factor significant in the development of anemia in men? A. Condom use B. Large hemorrhoids C. A diet high in cholesterol D. Smoking one pack of cigarettes daily

B. Large hemorrhoids Gastrointestinal (GI) tract bleeding is a common etiologic factor in men and may result from peptic ulcers, hiatal hernia, gastritis, cancer, hemorrhoids, diverticula, ulcerative colitis, or salicylate poisoning.

17. The nurse is caring for a 73-year-old patient who underwent a left total knee arthroplasty. On the third postoperative day, the patient complains of shortness of breath, slight chest pain, and that "something is wrong." Temperature is 98.4o F, blood pressure 130/88, respirations 36, and oxygen saturation 91% on room air. Which of the following should the nurse first suspect as the etiology of this episode? A. Septic embolus from the knee joint B. Pulmonary embolus from deep vein thrombosis C. New onset of angina pectoris D. Pleural effusion related to positioning in the operating room

B. Pulmonary embolus from deep vein thrombosis The patient presents the classic symptoms of pulmonary embolus: acute onset of symptoms, tachypnea, shortness of breath, and chest pain.

37. The nurse is scheduled to give a dose of salmeterol by metered dose inhaler (MDI). The nurse would administer the right drug by selecting the inhaler with which of the following trade names? A. Vanceril B. Serevent C. AeroBid D. Atrovent

B. Serevent The trade or brand name for salmeterol, an adrenergic bronchodilator, is Serevent.

26. The nurse is assigned to care for a patient in the emergency department admitted with an exacerbation of asthma. The patient has received a β-adrenergic bronchodilator and supplemental oxygen. If the patient's condition does not improve, the nurse should anticipate which of the following is likely to be the next step in treatment? A. Pulmonary function testing B. Systemic corticosteroids C. Biofeedback therapy D. Intravenous fluids

B. Systemic corticosteroids Systemic corticosteroids speed the resolution of asthma exacerbations and are indicated if the initial response to the β-adrenergic bronchodilator is insufficient.

48. Before discharge, the nurse discusses activity levels with a 61-year-old patient with COPD and pneumonia. Which of the following exercise goals is most appropriate once the patient is fully recovered from this episode of illness? A. Slightly increase activity over the current level. B. Walk for 20 minutes a day, keeping the pulse rate less than 130 beats per minute. C. Limit exercise to activities of daily living to conserve energy. D. Swim for 10 min/day, gradually increasing to 30 min/day.

B. Walk for 20 minutes a day, keeping the pulse rate less than 130 beats per minute. The patient will benefit from mild aerobic exercise that does not stress the cardiorespiratory system. The patient should be encouraged to walk for 20 min/day, keeping the pulse rate less than 75% to 80% of maximum heart rate (220 minus patient's age).

13. If a patient with an uncuffed tracheostomy tube coughs violently during suctioning and dislodges the tracheostomy tube, a nurse should first A. call the physician. B. attempt to reinsert the tracheostomy tube. C. position the patient in a lateral position with the neck extended. D. cover the stoma with a sterile dressing and ventilate the patient with a manual bag-mask until the physician arrives.

B. attempt to reinsert the tracheostomy tube.Retention sutures may be grasped (if present) and the tracheostomy opening spread, or a hemostat may be used to spread the opening. The obturator is inserted into the replacement tube (one size smaller than the original tube), lubricated with saline solution, and inserted into the stoma at a 45-degree angle to the neck. If the attempt is successful, the obturator tube should immediately be removed.

What religion might want a priest present at death?

Buddist

What does a person with Grave's disease look like?

Bug eyed, hyeractive

1. A nurse is reviewing the hematologic test results for a patient in whom the hematocrit (Hct) is reported at a reading of 30%. Based on this result, the nurse should interpret that the patient A. is susceptible to bleeding disorders. B. has fewer red blood cells than normal. C. is experiencing an inflammatory response. D. is experiencing an acute hemolytic crisis.

B. has fewer red blood cells than normal.The Hct is the measure of the volume of red blood cells in whole blood expressed as a percentage. This test is useful in the diagnosis of anemia, polycythemia, and abnormal hydration states. Patients who are susceptible to bleeding disorders likely will have a low platelet count. The inflammatory response may best be evaluated by examination of results that include the white blood cell count with differential analysis. Acute hemolytic crisis develops in patients receiving blood components in which incompatibility occurs or in patients with bleeding disorders or conditions that promote cellular damage, such as damage associated with shock.

18. Nursing interventions for the patient with aplastic anemia are directed toward the prevention of the complications of A. fatigue and dyspnea. B. hemorrhage and infection. C. thromboemboli and gangrene. D. cardiac arrhythmias and heart failure.

B. hemorrhage and infection. Hemorrhage from thrombocytopenia and infection from neutropenia are the greatest risks for the patient with aplastic anemia. The patient will experience fatigue from anemia, but bleeding and infection are the major causes of death in aplastic anemia.

3. In preparing the preoperative teaching plan for a patient who is to undergo a total laryngectomy, a nurse should give highest priority to the A. tracheostomy being in place for 2 to 3 days. B. patient's not being able to speak normally again. C. insertion of a gastrostomy feeding tube during surgery. D. patient's not being able to perform deep-breathing exercises.

B. patient's not being able to speak normally again. Patients who have a total laryngectomy have a permanent tracheostomy and will need to learn how to speak using alternative methods, such as an artificial larynx. The tracheostomy will be permanent to allow normal breathing patterns and air exchange. After surgery, the patient's nutrition is supplemented with enteral feedings, and when the patient can swallow secretions, oral feedings can begin. Deep-breathing exercises should be performed with the patient at least every 2 hours to prevent further pulmonary complications.

19. A patient with hemophilia is hospitalized with acute knee pain and swelling. An appropriate nursing intervention for the patient includes A. wrapping the knee with an elastic bandage. B. placing the patient on bed rest and applying ice to the joint. C. gently performing range-of-motion (ROM) exercises to the knee to prevent adhesions. D. administering nonsteroidal anti-inflammatory drugs (NSAIDs) as needed for pain.

B. placing the patient on bed rest and applying ice to the joint. During an acute bleeding episode in a joint, it is important to totally rest the involved joint and slow bleeding with application of ice. Drugs that decrease platelet aggregation, such as aspirin or NSAIDs, should not be used for pain. As soon as bleeding stops, mobilization of the affected area is encouraged with range-of-motion (ROM) exercises and physical therapy.

36. The resurgence in TB resulting from the emergence of multidrug-resistant strains of Mycobacterium tuberculosis is primarily the result of A. a lack of effective means to diagnose TB. B. poor compliance with drug therapy in patients with TB. C. the increased population of immunosuppressed individuals with AIDS. D. indiscriminate use of antitubercular drugs in treatment of other infections.

B. poor compliance with drug therapy in patients with TB. Drug-resistant strains of TB have developed because TB patients' compliance to drug therapy has been poor and there has been general decreased vigilance in monitoring and follow-up of TB treatment. Antitubercular drugs are almost exclusively used for TB infections. TB can be effectively diagnosed with sputum cultures. The incidence of TB is at epidemic proportions in patients with HIV, but this does not account for drug-resistant strains of TB.

2. A patient admitted to the emergency department with tension pneumothorax and mediastinal shift following an automobile crash is most likely to exhibit A. bradycardia. B. severe hypotension. C. mediastinal flutter. D. a sucking chest wound.

B. severe hypotension. Mediastinal shift may cause compression of the lung in the direction of the shift and compression, traction, torsion, or kinking of the great vessels. Blood return to the heart is dangerously impaired and causes a subsequent decrease in cardiac output and blood pressure. Tachycardia is a clinical manifestation of tension pneumothorax. An uncovered opened pneumothorax is associated with a sucking chest wound and mediastinal flutter.

25. A 75-year-old obese patient who is snoring loudly and having periods of apnea several times each night is most likely experiencing A. narcolepsy. B. sleep apnea. C. sleep deprivation. D. paroxysmal nocturnal dyspnea.

B. sleep apnea. Sleep apnea is most common in obese patients. Typical symptoms include snoring and periods of apnea. Narcolepsy is when a patient falls asleep unexpectedly. Sleep deprivation could result from sleep apnea. Paroxysmal nocturnal dyspnea occurs when a patient has shortness of breath during the night.

Good vitamin after gastrectomy

B12

40. Select all that apply. Atelectasis can be caused by A. long-term smoking. B. inadequate surfactant. C. localized airway obstruction. D. an increase in lung expansion. E. an increase in elastic recoil.

BCE The collapse of lung tissue has several causes, including reduced lung expansion, localized airway obstruction, inadequate surfactant, and an increase in elastic recoil. Smoking, although harmful, does not in itself cause atelectasis.

21. Select all that apply. Which of the following are significant risk factors for leukemia? A. Being a longtime smoker B. Employment in an oil refinery C. History of hemophilia in parent D. Having Down syndrome E. Having a twin brother with leukemia F. Treatment with an alkylating agent = 3 years ago

BDEF Exposure to chemical agents, treatment with alkylating cancer drugs, leukemia in a sibling, and the patient's having Down syndrome are all risk factors for leukemia.

Hypertension (HTN)

BP equal to or greater than 140/90 mm Hg on 2 separate occasions.

Vesicoureteral Reflux

Backward movement of urine from the lower to upper urinary tracts

Cerebellum

Balance, Coordination

A 45 year old client who was recently DX with terminal cancer says to the nurse "If God could only let me live long enough to put my daughter through college, I wouldn't mind dealing with the illness." The nurse caring for this client recognizes this statement as reflecting of what stage of grieving

Bargaining

Diagnostic Procedures for strokes

Baseline coagulation studies (PTT, PT-INR - how thick or thin the blood is), Brain imaging - CT scans or MRI's.

Why does a hypothyroidism patient receive cardiac medications?

Because of their tachycardia

Lab value that indicates jaundice is getting better

Bilirubin

The nurse is caring for a critically ill client with cirrhosis of the liver who has a nasogastric tube draining bright red blood. The nurse notes that the client's serum hemoglobin and hematocrit are decreased. What additional change in lab data should the nurse expect?

Decreased serum ammonia

Pediatrics: A mother brings her 4 year old boy to the clinic because he spends his day in constant motion, talks excessively, and is eaisly distracted from playing with his toys. His preschool teacher is unable to keep him focused in the classroom and suggested he unfergo a mental health evaluation. Which nursing diagnosis should the nurse formulate?

C) Impaired social Interaction Attention deficit hyperactivity disorder (ADHD) is a behavioral disorder of cjildren with significant problems in attention and concentratio, impulse control, and overactivity. The nursing diagnosis, Impaired social interaction addresses the childs hyperverbalism, shortened attention span, and increased need for mobility that created his classroom dificulties. A) Risk for injury B) Compromised family coping D) Dificient knowledge Should be included in the plan of care, communication between the family and the school should address ways to accommodate the child's behaviors.

39. Select all that apply. Which of the following are clinical manifestations of tension pneumothorax? A. Midline trachea B. Severe hypertension C. Progressive cyanosis D. A loud bruit on affected side E. Asymmetrical chest wall movement F. Subcutaneous emphysema in the neck

C,E, F The indicators of tension pneumothorax are asymmetrical chest wall movement, severe hypotension, subcutaneous emphysema in the neck and upper chest, and progressive cyanosis.

51. Which of the following statements made by a patient with COPD indicates a need for further education regarding the use of an ipratropium inhaler? A. "I should rinse my mouth following the two puffs to get rid of the bad taste." B. "I should wait at least 1 to 2 minutes between each puff of the inhaler." C. "If my breathing gets worse, I should keep taking extra puffs of the inhaler until I can breathe more easily." D. "Because this medication is not fast-acting, I cannot use it in an emergency if my breathing gets worse.

C. "If my breathing gets worse, I should keep taking extra puffs of the inhaler until I can breathe more easily." The patient should not take extra puffs of the inhaler at will to make breathing easier. Excessive treatment could trigger paradoxical bronchospasm, which would worsen the patient's respiratory status.

19. Which of the following statements made by a nurse would indicate proper teaching principles regarding feeding and tracheostomies? A. "Follow each spoon of food consumed with a drink of fluid." B. "Thin your foods to a liquid consistency whenever possible." C. "Tilt your chin forward toward the chest when swallowing your food." D. "Make sure your cuff is overinflated before eating if you have swallowing problems."

C. "Tilt your chin forward toward the chest when swallowing your food." A nurse should instruct a patient to tilt the chin toward the chest, which will close the glottis and allow food to enter the normal passageway. Ideally, foods should be of a thick consistency to enable effective swallowing and reduce the risk of aspiration. Overinflation of the cuff causes swallowing difficulties. Fluids should be consumed in small amounts after swallowing to prevent the risk of aspiration.

6. When preparing to administer an ordered blood transfusion, the nurse selects which of the following intravenous solutions to use when priming the blood tubing? A. 5% dextrose in water B. Lactated Ringer's C. 0.9% sodium chloride D. 0.45% sodium chloride

C. 0.9% sodium chloride The blood set should be primed before the transfusion with 0.9% sodium chloride, also known as normal saline. It is also used to flush the blood tubing after the infusion is complete to ensure the patient receives blood that is left in the tubing when the bag is empty.

43. The nurse evaluates that nursing interventions to promote airway clearance in a patient admitted with COPD are successful based on which of the following findings? A. Absence of dyspnea B. Improved mental status C. Effective and productive coughing D. PaO2 within normal range for the patient

C. Effective and productive coughing The issue of the question is airway clearance, which is most directly evaluated as successful if the patient can engage in effective and productive coughing.

46. Nursing assessment findings of jugular vein distention and pedal edema would be indicative of which of the following complications of emphysema? A. Acute respiratory failure B. Pulmonary edema caused by left-sided heart failure C. Fluid volume excess secondary to cor pulmonale D. Secondary respiratory infection

C. Fluid volume excess secondary to cor pulmonale Cor pulmonale is a right-sided heart failure caused by resistance to right ventricular outflow due to lung disease. With failure of the right ventricle, the blood emptying into the right atrium and ventricle would be slowed, leading to jugular venous distention and pedal edema.

23. Which of the following positions is most appropriate for the nurse to place a patient experiencing an asthma exacerbation? A. Supine B. Lithotomy C. High-Fowler's D. Reverse Trendelenburg

C. High-Fowler'sThe patient experiencing an asthma attack should be placed in high-Fowler's position to allow for optimal chest expansion and enlist the aid of gravity during inspiration.

8. Which of the following clinical manifestations would the nurse expect to find during assessment of a patient admitted with pneumococcal pneumonia? A. Hyperresonance on percussion B. Fine crackles in all lobes on auscultation C. Increased vocal fremitus on palpation D. Vesicular breath sounds in all lobes

C. Increased vocal fremitus on palpation. A typical physical examination finding for a patient with pneumonia is increased vocal fremitus on palpation. Other signs of pulmonary consolidation include dullness to percussion, bronchial breath sounds, and crackles in the affected area.

28. While teaching a patient with asthma about the appropriate use of a peak flow meter, the nurse instructs the patient to do which of the following? A. Use the flow meter each morning after taking medications to evaluate their effectiveness. B. Empty the lungs and then inhale quickly through the mouthpiece to measure how fast air can be inhaled. C. Keep a record of the peak flow meter numbers if symptoms of asthma are getting worse. D. Increase the doses of the long-term control medication if the peak flow numbers decrease.

C. Keep a record of the peak flow meter numbers if symptoms of asthma are getting worse. It is important to keep track of peak flow readings daily and when the patient's symptoms are getting worse. The patient should have specific directions as to when to call the physician based on personal peak flow numbers. Peak flow is measured by exhaling into the meters and should be assessed before and after medications to evaluate their effectiveness.

6. What type of anemia is associated with folate deficiency? A. Microcytic B. Pernicious C. Megaloblastic D. Iron deficiency

C. Megaloblastic Megaloblastic anemia is nutritional anemia; large immature red blood cells with a decreased oxygen-carrying capacity can occur as a result of impaired DNA synthesis. Folic acid is used in the synthesis of DNA and helps convert B12 to coenzyme form. Folic acid is needed for growth and development of red blood cells. Microcytic anemia is anemia with abnormally small erythrocytes (red blood cells) in Hb. This anemia is associated with vitamin B6 (pyridoxine) deficiency. Pernicious anemia is caused by a deficiency of vitamin B12. Iron-deficiency anemia results from loss of blood or deficient intake of iron foods or disease states in which the body does not absorb or utilize iron as it should.

20. During assessment of a 45-year-old patient with asthma, the nurse notes wheezing and dyspnea. The nurse interprets that these symptoms are related to which of the following pathophysiologic changes? A. Laryngospasm B. Overdistention of the alveoli C. Narrowing of the airway D. Pulmonary edema

C. Narrowing of the airwayNarrowing of the airway leads to reduced airflow, making it difficult for the patient to breathe and producing the characteristic wheezing.

Management: The nurse gives a client a narcotic for pain and must now leave the unit. To whom should the nurse delegate the task of evaluating the client's response to the pain medication? A. Unit clerk B. Student nurse C. Nurse manager D. CNA

C. Nurse manager Pain assessment and evaluation of analgesia are components in the scope of nursing practice and are determined by a licensed nurse, so the nurse manager should assume this responsibilty.

16. While ambulating a patient with metastatic lung cancer, the nurse observes a drop in oxygen saturation from 93% to 86%. Which of the following nursing interventions is most appropriate based upon these findings? A. Continue with ambulation as this is a normal response to activity. B. Move the oximetry probe from the finger to the earlobe for more accurate monitoring during activity. C. Obtain a physician's order for supplemental oxygen to be used during ambulation and other activity. D. Obtain a physician's order for arterial blood gas determinations to verify the oxygen saturation.

C. Obtain a physician's order for supplemental oxygen to be used during ambulation and other activity. An oxygen saturation level that drops below 90% with activity indicates that the patient is not tolerating the exercise and needs to have supplemental oxygen applied.

11. When assessing a patient's respiratory status, which of the following nonrespiratory data are most important for the nurse to obtain? A. Height and weight B. Neck circumference C. Occupation and hobbies D. Usual daily fluid intake

C. Occupation and hobbiesMany respiratory problems occur as a result of chronic exposure to inhalation irritants. Common occupational sources of inhalation irritants include mines, granaries, farms, lawn care companies, paint, plastics and rubber manufacture, and building remodeling. Hobbies associated with inhalation irritants include woodworking, metal finishing, furniture refinishing, painting, and ceramics. Daily fluids, height, and weight are more related to respiratory problems secondary to cardiac issues.

30. The nurse determines that a patient is experiencing common adverse effects from the inhaled corticosteroid beclomethasone (Beclovent) after noting which of the following? A. Adrenocortical dysfunction and hyperglycemia B. Elevation of blood glucose and calcium levels C. Oropharyngeal candidiasis and hoarseness D. Hypertension and pulmonary edema

C. Oropharyngeal candidiasis and hoarseness Oropharyngeal candidiasis and hoarseness are common adverse effects from the use of inhaled corticosteroids because the medication can lead to overgrowth of organisms and local irritation if the patient does not rinse the mouth following each dose.

15. When reviewing the results of a 83-year-old patient's blood tests, which of the following findings would be of most concern to the nurse? A. Platelets of 150,000/µl B. Serum iron of 50 mcg/dl C. Partial thromboplastin time (PTT) of 60 seconds D. Erythrocyte sedimentation rate (ESR) of 35 mm in 1 hour

C. Partial thromboplastin time (PTT) of 60 seconds In aging, the partial thromboplastin time (PTT) is normally decreased, so an abnormally high PTT of 60 seconds is an indication that bleeding could readily occur. Platelets are unaffected by aging, and 150,000 is a normal count. Serum iron levels are decreased and the erythrocyte sedimentation rate (ESR) is significantly increased with aging, as are reflected in these values.

30. Which of the following conditions or factors in a 64-year-old patient diagnosed with head and neck cancer most likely contributed to this health problem? A. Patient's hobby is oil painting. B. Patient's father also had head and neck cancer. C. Patient uses chewing tobacco and drinks beer daily. D. Patient quit school at age 16 and has worked in a butcher shop for more than 40 years.

C. Patient uses chewing tobacco and drinks beer daily. Many environmental risk factors contribute to the development of head and neck cancer, although the actual cause is unknown. There does not appear to be a genetic predisposition to this type of cancer. The two most important risk factors are tobacco and alcohol use, especially in combination. Other risk factors include chewing tobacco, pipe smoking, marijuana use, voice abuse, chronic laryngitis, exposure to industrial chemicals or hardwood dust, and poor oral hygiene.

14. A 71-year-old patient is admitted with acute respiratory distress related to cor pulmonale. Which of the following nursing interventions is most appropriate during admission of this patient? A. Delay any physical assessment of the patient and review with the family the patient's history of respiratory problems. B. Perform a comprehensive health history with the patient to review prior respiratory problems. C. Perform a physical assessment of the respiratory system and ask specific questions related to this episode of respiratory distress. D. Complete a full physical examination to determine the effect of the respiratory distress on other body functions.

C. Perform a physical assessment of the respiratory system and ask specific questions related to this episode of respiratory distress.Because the patient is having respiratory difficulty, the nurse should ask specific questions about this episode and perform a physical assessment of this system. Further history taking and physical examination of other body systems can proceed once the patient's acute respiratory distress is being managed.

10. During discharge teaching for a 65-year-old patient with emphysema and pneumonia, which of the following vaccines should the nurse recommend the patient receive? A. S. aureus B. H. influenzae C. Pneumococcal D. Bacille Calmette-Guérin (BCG)

C. Pneumococcal The pneumococcal vaccine is important for patients with a history of heart or lung disease, recovering from a severe illness, age 65 or over, or living in a long-term care facility.

26. Which of the following conditions is manifested by unexplained shortness of breath and a high mortality rate? A. Bleeding ulcer B. Transient ischemia C. Pulmonary embolism D. MI

C. Pulmonary embolism A high mortality rate is associated with a pulmonary embolism. A pulmonary embolism is an obstruction of the pulmonary artery caused by an embolus. It presents with hypoxia, anxiety, restlessness, and shortness of breath. Bleeding ulcers, MI, and transient ischemia are not associated with such a high mortality rate.

12. If a nurse is assessing a patient whose recent blood gas determination indicated a pH of 7.32 and respirations are measured at 32 breaths/min, which of the following is the most appropriate nursing assessment? A. The rapid breathing is causing the low pH. B. The nurse should sedate the patient to slow down respirations. C. The rapid breathing is an attempt to compensate for the low pH. D. The nurse should give the patient a paper bag to breathe into to correct the low pH.

C. The rapid breathing is an attempt to compensate for the low pH. The respiratory system influences pH (acidity) through control of carbon dioxide exhalation. Thus, rapid breathing increases the pH. Breathing into a paper bag aids a patient who is hyperventilating; in respiratory alkalosis, it aids in lowering the pH. The use of sedation can cause respiratory depression and hypoventilation, resulting in an even lower pH.

33. A patient with an acute pharyngitis is seen at the clinic with fever and severe throat pain that affects swallowing. On inspection the throat is reddened and edematous with patchy yellow exudates. The nurse anticipates that collaborative management will include A. treatment with antibiotics. B. treatment with antifungal agents. C. a throat culture or rapid strep antigen test. D. treatment with medication only if the pharyngitis does not resolve in 3 to 4 days.

C. a throat culture or rapid strep antigen test. Although inadequately treated β-hemolytic streptococcal infections may lead to rheumatic heart disease or glomerulonephritis, antibiotic treatment is not recommended until strep infections are definitely diagnosed with culture or antigen tests. The manifestations of viral and bacterial infections are similar, and appearance is not diagnostic except when candidiasis is present.

The nurse is caring for a client with a fractured right elbow. Which assessment finding has the highest priority and requires immediate intervention?

Deep, unrelenting pain in the right arm

32. An excess of carbon dioxide in the blood causes an increased respiratory rate and volume because CO2 A. displaces oxygen on hemoglobin, leading to a decreased PaO2. B. causes an increase in the amount of hydrogen ions available in the body. C. combines with water to form carbonic acid, lowering the pH of cerebrospinal fluid. D. directly stimulates chemoreceptors in the medulla to increase respiratory rate and volume.

C. combines with water to form carbonic acid, lowering the pH of cerebrospinal fluid. A combination of excess CO2 and H2O results in carbonic acid, which lowers the pH of the cerebrospinal fluid and stimulates an increase in the respiratory rate. Peripheral chemoreceptors in the carotid and aortic bodies also respond to increases in PaCO2 to stimulate the respiratory center. Excess CO2 does not increase the amount of hydrogen ions available in the body but does combine with the hydrogen of water to form an acid.

34. Following a supraglottic laryngectomy, the patient is taught how to use the supraglottic swallow to minimize the risk of aspiration. In teaching the patient about this technique, the nurse instructs the patient to A. perform Valsalva maneuver immediately after swallowing. B. breathe between each Valsalva maneuver and cough sequence. C. cough after swallowing to remove food from the top of the vocal cords. D. practice swallowing thin, watery fluids before attempting to swallow solid foods.

C. cough after swallowing to remove food from the top of the vocal cords. A supraglottic laryngectomy involves removal of the epiglottis and false vocal cords, and the removal of the epiglottis allows food to enter the trachea. Supraglottic swallowing requires performance of the Valsalva maneuver before placing food in the mouth and swallowing. The patient then coughs to remove food from the top of the vocal cords, swallows again, and then breathes after the food has been removed from the vocal cords.

11. If a patient has pernicious anemia, the nurse should provide information regarding A. frequent bouts of dyspnea. B. risks relative to dehydration. C. deficiency of intrinsic factor. D. lack of any effective treatment for this condition.

C. deficiency of intrinsic factor. Pernicious anemia is a type of anemia caused by failure of absorption of vitamin B12 (cobalamin). The most common cause is lack of intrinsic factor, a glucoprotein produced by the parietal cells of the gastric lining.

18. A nurse establishes the presence of a tension pneumothorax when assessment findings reveal a(n) A. absence of lung sounds on the affected side. B. inability to auscultate tracheal breath sounds. C. deviation of the trachea toward the side opposite the pneumothorax. D. shift of the point of maximal impulse (PMI) to the left, with bounding pulses.

C. deviation of the trachea toward the side opposite the pneumothorax. Tension pneumothorax is caused by rapid accumulation of air in the pleural space, causing severely high intrapleural pressure. This results in collapse of the lung, and the mediastinum shifts toward the unaffected side, which is subsequently compressed.

37. The chronic inflammation of the bronchi characteristic of chronic obstructive pulmonary disease (COPD) results in A. collapse of small bronchioles on expiration. B. permanent, abnormal dilation of the bronchi. C. hyperplasia of mucus-secreting cells and bronchial edema. D. destruction of the elastic and muscular structures of the bronchial wall.

C. hyperplasia of mucus-secreting cells and bronchial edema. Chronic bronchitis is characterized by chronic inflammation of the bronchial lining, with edema and increased mucus production. Collapse of small bronchioles on expiration is common in emphysema, and abnormal dilation of the bronchi because of destruction of the elastic and muscular structures is characteristic of bronchiectasis.

8. A person complains of fatigue and malaise and has a slight temperature elevation for 2 days before symptoms of influenza (fever, chest congestion, and productive cough) become noticeable. During the time immediately before the illness is diagnosed, the patient A. could avoid contracting the disease if treatment is begun with antibiotics. B. is unable to spread the disease because it is still in the incubation period. C. is in the prodromal stage and is highly contagious and able to spread the disease. D. has a nosocomial infection, which affects approximately two million individuals a year.

C. is in the prodromal stage and is highly contagious and able to spread the disease. The prodromal stage is a short period of time (hours to several days) immediately preceding the onset of an illness during which the patient is very contagious. Antibiotics are not effective against viral illnesses. The incubation period is the time from entry of the organism to the onset of symptoms and, in some viral illnesses, may be contagious. Nosocomial infections are those acquired in a hospital, and this scenario does not suggest the source of the infection.

7. During care of a patient with multiple myeloma, an important nursing intervention is A. limiting activity to prevent pathologic fractures. B. assessing for changes in size and characteristics of lymph nodes. C. maintaining a fluid intake of 3 to 4 L/day to dilute calcium load. D. administering narcotic analgesics continuously to control bone pain.

C. maintaining a fluid intake of 3 to 4 L/day to dilute calcium load. Adequate hydration must be maintained to minimize problems from hypercalcemia. The goal of a urinary output of 1.5 to 2 L/day requires an intake of 3 to 4 L/day.

5. Absorption of vitamin B12 may be decreased in older adults because of decreased A. intestinal motility. B. production of bile by the liver. C. production of intrinsic factor by the stomach. D. synthesis of cobalamin (vitamin B12) by intestinal bacteria.

C. production of intrinsic factor by the stomach. Older persons are at risk for deficiency of cobalamin (pernicious anemia) because of a naturally occurring reduction of the intrinsic factor by the stomach mucosa. Absorption of cobalamin relies on intrinsic factor. Both must be present for absorption. Megaloblastic anemia is related to folate dysfunction. Intestinal motility (peristalsis) is the motion that moves food down the GI tract. The rhythmic contractions of muscles cause wave-like motions. Lack of peristalsis is called "paralytic ileus." Bile is produced in the liver, is stored and concentrated in the gallbladder, and is released into the duodenum when fat is eaten. Bile emulsifies fats and prepares them for enzyme digestion in order for the nutrient to be absorbed into lymph and eventually into blood vessels to the liver. Vitamin K (the blood-clotting vitamin) is synthesized by intestinal bacteria.

9. In older adults, infection after exposure to respiratory illness is most likely to A. result in similar rates of infection as in the younger adult. B. be easily prevented with the use of antibiotics after being exposed. C. result in serious lower respiratory infection related to weakened respiratory muscles and fewer cilia. D. be less serious because the older adult has less contact with younger children who are most likely to carry serious infections.

C. result in serious lower respiratory infection related to weakened respiratory muscles and fewer cilia. Changes in the older adult respiratory system make older adults more susceptible to infections that can be very serious and life threatening. Use of antibiotics to "prevent" lung infections is not recommended and is ineffective for viral infections.

38. In teaching the patient with COPD about the need for physical exercise, the nurse informs the patient that A. all patients with COPD should be able to increase walking gradually up to 20 min/day. B. a bronchodilator inhaler should be used to relieve exercise-induced dyspnea immediately after exercise. C. shortness of breath is expected during exercise but should return to baseline within 5 minutes after the exercise. D. monitoring the heart rate before and after exercise is the best way to determine how much exercise can be tolerated.

C. shortness of breath is expected during exercise but should return to baseline within 5 minutes after the exercise.Shortness of breath usually increases during exercise, but the activity is not being overdone if breathing returns to baseline within 5 minutes after stopping. Bronchodilators can be administered 10 minutes before exercise but should not be administered for at least 5 minutes after activity to allow recovery. Patients are encouraged to walk 15 to 20 minutes a day with gradual increases, but actual patterns will depend on patient tolerance. Dyspnea most frequently limits exercise and is a better indication of exercise tolerance than is heart rate in the patient with COPD.

Amphotericin B: Interventions

CAN CAUSE: * Nephrotoxicity * Electrolyte imbalance. * Hypokalemia * Hypomagnesemia * Urinary output, BUN, & serum Cr levels need to be closely monitored.

16. If a patient with blood type O Rh- is given AB Rh- blood, the nurse would expect A. the patient's Rh factor to react with the RBCs of the donor blood. B. no adverse reaction because the patient has no antibodies against the donor blood. C. the anti-A and anti-B antibodies in the patient's blood to hemolyze the donor blood. D. the anti-A and anti-B antibodies in the donor blood to hemolyze the patient's blood.

C. the anti-A and anti-B antibodies in the patient's blood to hemolyze the donor blood. A patient with O Rh+ blood has no A or B antigens on the red cell but does have anti-A and anti-B antibodies in the blood and has an Rh antigen. AB Rh- blood has both A and B antigens on the red cell but no Rh antigen and no anti-A or anti-B antibodies. If the AB Rh- blood is given to the patient with O Rh+ blood, the antibodies in the patient's blood will react with the antigens in the donor blood, causing hemolysis of the donor cells. There will be no Rh reaction because the donor blood has no Rh antigen.

14. Upon entering the room of a patient who has just returned from surgery for total laryngectomy and radical neck dissection, a nurse should recognize a need for intervention when finding A. a gastrostomy tube that is clamped. B. the patient coughing blood-tinged secretions from the tracheostomy. C. the patient positioned in a lateral position with the head of the bed flat. D. 200 ml of serosanguineous drainage in the patient's portable drainage device.

C. the patient positioned in a lateral position with the head of the bed flat. After total laryngectomy and radical neck dissection, a patient should be placed in a semi-Fowler's position to decrease edema and limit tension on the suture line.

Lab results for the older adult affected by aging

Calcium Potassium Sodium

Vit D is needed for absorption of what?

Calcium and Phosphorus

What does the parathyroid gland regulate?

Calcium and phosphorus

Pt starts screaming when the nurse moves the cast leg onto the pillow.. What should the nurse do

Call DR RT suspected compartment syndrome

The staff member of an out patient clinic have successfully assisted the clients to safety during a fire in the waiting area. What would be the next plan of action

Call for additional help

The nurse notes that the only ECG for a 55-year-old male client scheduled for surgery in 2 hours is dated 2 years ago. The client reports that he has a history of "heart trouble," but has no problems at present. Hospital protocol requires that those over 50 years of age have a recent ECG prior to surgery. What nursing action would be best for the nurse to implement?

Call for an ECG to be performed immediately.

A client with chronic asthma is admitted to postanesthesia complaining of pain at a level of 8 of 10, with a blood pressure of 124/78, pulse of 88 beats/min, and respirations of 20 breaths/min. The postanesthesia recovery prescription is, "Morphine 2 to 4 mg IV push while in recovery for pain level over 5." What intervention should the nurse implement?

Call the anesthesia provider for a different medication for pain.

Client teaching for Addison's Disease?

Carry injectable dexamethasone (Decadron), will have to take this medication for life

What does the Thyroid control?

Cellular Metabolism

What will happen of a DKA patient's blood sugar is dropped too rapidly?

Cerebral edema

The nurse initiates neuro checks for a client who is at risk for neurologic compromise. Which manifestation typically provides the first indication of altered neuro function?

Change in level of consciousness

A client is ready for discharge following creation of an ileostomy. Which instruction should the nurse include in discharge teaching?

Change the bag when the seal is broken.

What is the RN's responsibility to ensure the ET tube is in the correct position?

Check bilateral breath sounds

Hiatal Hernia

Chest pain after swallowing that subsides when the food bolus moves into stomach

The nurse has attended a staff development conference on a STD. Which of the following statements, if made would require nurse to follow up

Chlamydia infection are strongly linked with cervical cancer

A client is admitted with a diagnosis of chronic obstructive pulmonary disease (COPD). What is the nurse's rationale for keeping the client's oxygen administration level at 3 L/min or less?

Chronic hypoxemia creates the urge to breathe in COPD.

What sign is elicited by tapping over the facial nerve and results in a spasm?

Chvostek sign

Urine speciman

Clamp the drainage bag label the urine specimen Place in a biohazard bag Document procedure

A nurse is observing a new hire nurse provide care for assigned clients. The nurse should intervene if the new hire nurse is observed

Cleansing the wound from the outer surface to the inner surface for the client how wound is infected with MRSA

During the shift report, the charge nurse informs a nurse that she has been assigned to another unit for the day. The nurse begins to sigh deeply and tosses about her belongings as she prepares to leave, making it known that she is very unhappy about being "floated" to the other unit. What is the best immediate action for the charge nurse to take?

Continue with shift report and talk to the nurse about the incident at a later time.

Ulcerative colitis treatments

Corticosteriods, metronidazole (flagyl) and ciprofloxacin (cipro) for exaserbations

Dosages of this drug may need to be decreased if a client is also taking levothyroxine (Synthroid)?

Coumadin

What is the more specific lab test for renal function other than BUN?

Creatinine

Med Surg On the secound day after admission, a client with a fractures pelvis develops chest pain, tachypnea, and tachycardia. Which additional finding should the nurse identify that is most likely related to a fat embolism?

D) Petechaie of the anterior chest wall The pathophysiologic process of fat embolism syndrome (FES) after fracture is related to the release of bone marrow fat globules into the venous circulation followed with platelet aggregation. Fat emboli lodge in the pulmonary vasculature, result in tissue hypoxia, and manifest as petechiae on the neck, anterior chest wall, axilla, buccal membrane, and conjunctiva of the eye. The client with FES experiences acute onset of chest pain, tachypnea, tachycardia, and elevated blood pressure. Although hypoxia, restlessness, and confusion occur, petechiae provides the most differentiating data for FES venus other complications.

11. The nurse evaluates that discharge teaching for a patient hospitalized with pneumonia has been most effective when the patient states which of the following measures to prevent a relapse? A. "I will increase my food intake to 2400 calories a day to keep my immune system well." B. "I must use home oxygen therapy for 3 months and then will have a chest x-ray to reevaluate." C. "I will seek immediate medical treatment for any upper respiratory infections." D. "I should continue to do deep-breathing and coughing exercises for at least 6 weeks."

D. "I should continue to do deep-breathing and coughing exercises for at least 6 weeks." It is important for the patient to continue with coughing and deep breathing exercises for 6 to 8 weeks until all of the infection has cleared from the lungs. A patient should seek medical treatment for upper respiratory infections that persist for more than 7 days. Increased fluid intake, not caloric intake, is required to liquefy secretions. Home O2 is not a requirement unless the patient's oxygenation saturation is below normal.

17. The nurse evaluates that teaching for the patient with iron deficiency anemia has been effective when the patient states A. "I will need to take the iron supplements the rest of my life." B. "I will increase my dietary intake of milk and milk products." C. "I should increase my activity to increase my aerobic capacity." D. "I should take the iron for several months after my blood is normal."

D. "I should take the iron for several months after my blood is normal." To replace the body's iron stores, iron supplements should be continued for 2 to 3 months after the Hb level returns to normal, but if the cause of the iron deficiency is corrected, the supplements do not need to be taken for a lifetime. Milk and milk products are poor sources of dietary iron. Activity should be gradually increased as Hb levels return to normal because aerobic capacity can be increased when adequate Hb is available.

31. The nurse determines that the patient understood medication instructions about the use of a spacer device when taking inhaled medications after hearing the patient state which of the following as the primary benefit? A. "Now I will not need to breathe in as deeply when taking the inhaler medications." B. "This device will make it so much easier and faster to take my inhaled medications." C. "I will pay less for medication because it will last longer." D. "More of the medication will get down into my lungs to help my breathing."

D. "More of the medication will get down into my lungs to help my breathing." A spacer assists more medication to reach the lungs, with less being deposited in the mouth and the back of the throat.

4. The nurse is teaching a patient who is to undergo bone marrow aspiration. Which of the following statements made by the nurse would indicate correct instruction regarding the site for the aspiration procedure? A. "The health care provider will perform the aspiration by needle to the femur." B. "The health care provider will perform the aspiration by needle to the scapula." C. "The health care provider will perform the aspiration by needle to the antecubital fossa." D. "The health care provider will perform the aspiration by needle to the posterior iliac crest."

D. "The health care provider will perform the aspiration by needle to the posterior iliac crest." Bone marrow samples are commonly taken from the posterior iliac crest or, as an alternative, the sternum may be aspirated. These sites provide relative ease in accessing the bone marrow via the biopsy needle. The antecubital fossa, femur, and scapula do not allow access to bone marrow while also providing reduced risk of harm to the patient.

5. Before starting a transfusion of packed red blood cells for an anemic patient, the nurse would arrange for a peer to monitor his or her other assigned patients for how many minutes when the nurse begins the transfusion? A. 60 B. 5 C. 30 D. 15

D. 15 As part of standard procedure, the nurse remains with the patient for the first 15 minutes after hanging a blood transfusion. Patients who are likely to have a transfusion reaction will more often exhibit signs within the first 15 minutes that the blood is infusing.

42. The patient has an order for albuterol 5 mg via nebulizer. Available is a solution containing 1 mg/ml. How many milliliters should the nurse use to prepare the patient's dose? A. 0.2 B. 2.5 C. 3.75 D. 5.0

D. 5.0

1. The arterial blood gas (ABG) readings that indicate compensated respiratory acidosis are a PaCO2 of A. 30 mm Hg and bicarbonate level of 24 mEq/L. B. 30 mm Hg and bicarbonate level of 30 mEq/L. C. 50 mm Hg and bicarbonate level of 20 mEq/L. D. 50 mm Hg and bicarbonate level of 30 mEq/L.

D. 50 mm Hg and bicarbonate level of 30 mEq/L. If compensation is present, carbon dioxide and bicarbonate are abnormal (or nearly so) in opposite directions (e.g., one is acidotic and the other alkalotic).

27. A patient with COPD is receiving oxygen at 2 L/min. While in the supine position for a bath, the patient complains of shortness of breath. What is the most appropriate first nursing action? A. Increase the flow of oxygen. B. Perform tracheal suctioning. C. Report this to the physician. D. Assist the patient to Fowler's position.

D. Assist the patient to Fowler's position. Breathing is easier in Fowler's position because it permits greater expansion of the chest cavity. If repositioning does not improve the situation, then oxygenation and physician reporting might be appropriate. The patient would not benefit from tracheal suctioning.

7. Which of the following physical assessment findings in a patient with pneumonia best supports the nursing diagnosis of ineffective airway clearance? A. Oxygen saturation of 85% B. Respiratory rate of 28 C. Presence of greenish sputum D. Basilar crackles

D. Basilar crackles The presence of adventitious breath sounds indicates that there is accumulation of secretions in the lower airways. This would be consistent with a nursing diagnosis of ineffective airway clearance because the patient is retaining secretions.

22. Respiratory acidosis is at highest risk in a patient with A. hypokalemia. B. pulmonary fibrosis. C. salicylate overdose. D. COPD.

D. COPD. Chronic respiratory acidosis is most commonly caused by COPD. Pulmonary fibrosis, hypokalemia, and salicylate overdose do not predispose a patient to respiratory acidosis. Hypokalemia can lead to cardiac dysrhythmias. Salicylate overdose results in central nervous system changes, and pulmonary fibrosis can result in respiratory arrest.

14. Which of the following foods is high in iron? A. Citrus fruits B. Milk products C. Yellow vegetables D. Green leafy vegetables

D. Green leafy vegetables Green leafy vegetables are high in iron. Foods cooked in iron pots and foods such as liver (the richest source), oysters, lean meats, kidney beans, whole wheat bread, kale, spinach, egg yolk, turnip tops, beet greens, carrots, apricots, and raisins are also high in iron.

22. The nurse is assigned to care for a patient who has anxiety and an exacerbation of asthma. Which of the following is the primary reason for the nurse to carefully inspect the chest wall of this patient? A. Observe for signs of diaphoresis B. Allow time to calm the patient C. Monitor the patient for bilateral chest expansion D. Evaluate the use of intercostal muscles

D. Evaluate the use of intercostal muscles The nurse physically inspects the chest wall to evaluate the use of intercostal (accessory) muscles, which gives an indication of the degree of respiratory distress experienced by the patient.

41. When planning patient teaching about emphysema, the nurse understands that the symptoms of emphysema are caused by which of the following? A. Hypertrophy and hyperplasia of goblet cells in the bronchi B. Collapse and hypoventilation of the terminal respiratory unit C. An overproduction of the antiprotease alpha1-antitrypsin D. Hyperinflation of alveoli and destruction of alveolar walls

D. Hyperinflation of alveoli and destruction of alveolar walls In emphysema, there are structural changes that include hyperinflation of alveoli, destruction of alveolar walls, destruction of alveolar capillary walls, narrowing of small airways, and loss of lung elasticity.

5. A nurse is performing assessment for a patient diagnosed with chronic obstructive pulmonary disease (COPD). Which of the following findings should the nurse expect to observe? A. Nonproductive cough B. Prolonged inspiration C. Vesicular breath sounds D. Increased anterior-posterior chest diameter

D. Increased anterior-posterior chest diameter An increased anterior-posterior diameter is a compensatory mechanism experienced by patients with COPD and is caused by air-trapping. Patients with COPD have a productive cough, often expectorating copious amounts of sputum. Because of air-trapping, patients with COPD experience a prolonged expiration because the rate of gas on exhalation takes longer to escape. Chest auscultation for patients with COPD often reveals wheezing, crackles, and other adventitious breath sounds.

2. When assessing lab values on a patient admitted with septicemia, the nurse would expect to find: A. Increased platelets B. Decreased red blood cells C. Decreased erythrocyte sedimentation rate (ESR) D. Increased bands in the WBC differential (shift to the left)

D. Increased bands in the WBC differential (shift to the left) When infections are severe, such as in septicemia, more granulocytes are released from the bone marrow as a compensatory mechanism. To meet the increased demand, many young, immature polymorphonuclear neutrophils (bands) are released into circulation. WBCs are usually reported in order of maturity, with the less mature forms on the left side of a written report. Hence, the term "shift to the left" is used to denote an increase in the number of bands.

10. The blood bank notifies the nurse that the two units of blood ordered for an anemic patient are ready for pick up. The nurse should take which of the following actions to prevent an adverse effect during this procedure? A. Immediately pick up both units of blood from the blood bank. B. Regulate the flow rate so that each unit takes at least 4 hours to transfuse. C. Set up the Y-tubing of the blood set with dextrose in water as the flush solution. D. Infuse the blood slowly for the first 15 minutes of the transfusion.

D. Infuse the blood slowly for the first 15 minutes of the transfusion. Because a transfusion reaction is more likely to occur at the beginning of a transfusion, the nurse should initially infuse the blood at a rate no faster than 2 ml/min and remain with the patient for the first 15 minutes after hanging a unit of blood.

38. The nurse is evaluating whether a patient understands how to safely determine whether a metered dose inhaler is empty. The nurse interprets that the patient understands this important information to prevent medication underdosing when the patient describes which method to check the inhaler? A. Place it in water to see if it floats. B. Shake the canister while holding it next to the ear. C. Check the indicator line on the side of the canister. D. Keep track of the number of inhalations used.

D. Keep track of the number of inhalations used. It is no longer appropriate to see if a canister floats in water or not as research has demonstrated this is not accurate. The best method to determine when to replace an inhaler is by knowing the maximum puffs available per MDI and then replacing when those inhalations have been used.

15. When planning appropriate nursing interventions for a patient with metastatic lung cancer and a 60-pack-year history of cigarette smoking, the nurse recognizes that the smoking has most likely decreased the patient's underlying respiratory defenses because of impairment of which of the following? A. Reflex bronchoconstriction B. Ability to filter particles from the air C. Cough reflex D. Mucociliary clearance

D. Mucociliary clearance Smoking decreases the ciliary action in the tracheobronchial tree, resulting in impaired clearance of respiratory secretions, chronic cough, and frequent respiratory infections.

13. If a patient with arthritis develops iron-deficiency anemia, a nurse should ask about the patient's use of A. alcoholic beverages. B. stool softeners and laxatives. C. caffeinated foods and beverages. D. NSAIDs.

D. NSAIDs. NSAIDs decrease the level of vitamin C, which aids in the absorption of iron. These drugs also compete with folate and vitamin K and may cause gastritis. Excessive alcoholic beverage consumption can cause stomach irritation; alcohol would not be directly related to iron-deficiency anemia unless bleeding ulcers or gastritis were to occur. NSAID consumption, not stool softeners and laxative use, would be suspect for iron-deficiency anemia. Caffeinated foods and beverages can cause gastric irritation and discomfort but are not associated with iron-deficiency anemia.

44. When caring for a patient with COPD, the nurse identifies a nursing diagnosis of imbalanced nutrition less than body requirements after noting a weight loss of 30 lb. Which of the following would be an appropriate intervention to add to the plan of care for this patient? A. Teach the patient to use frozen meals at home that can be microwaved. B. Provide a high-calorie, high-carbohydrate, nonirritating, frequent feeding diet. C. Order fruits and fruit juices to be offered between meals. D. Order a high-calorie, high-protein diet with six small meals a day.

D. Order a high-calorie, high-protein diet with six small meals a day.Because the patient with COPD needs to use greater energy to breathe, there is often decreased oral intake because of dyspnea. A full stomach also impairs the ability of the diaphragm to descend during inspiration, interfering with the work of breathing. Finally, the metabolism of a high carbohydrate diet yields large amounts of CO2, which may lead to acidosis in patients with pulmonary disease. For these reasons, the patient with emphysema should take in a high-calorie, high-protein diet, eating six small meals per day.

13. Which of the following nursing interventions is most appropriate to enhance oxygenation in a patient with unilateral malignant lung disease? A. Positioning patient on right side. B. Maintaining adequate fluid intake C. Performing postural drainage every 4 hours D. Positioning patient with "good lung down"

D. Positioning patient with "good lung down" Therapeutic positioning identifies the best position for the patient assuring stable oxygenation status. Research indicates that positioning the patient with the unaffected lung (good lung) dependent best promotes oxygenation in patients with unilateral lung disease. For bilateral lung disease, the right lung down has best ventilation and perfusion. Increasing fluid intake and performing postural drainage will facilitate airway clearance, but positioning is most appropriate to enhance oxygenation.

33. The nurse determines that the patient is not experiencing adverse effects of albuterol (Proventil) after noting which of the following patient vital signs? A. Oxygen saturation 96% B. Respiratory rate of 18 C. Temperature of 98.4° F D. Pulse rate of 76

D. Pulse rate of 76 Albuterol is a β2-agonist that can sometimes cause adverse cardiovascular effects. These would include tachycardia and angina. A pulse rate of 76 indicates that the patient did not experience tachycardia as an adverse effect.

36. The nurse is assisting a patient to learn self-administration of beclomethasone two puffs inhalation q6hr. The nurse explains that the best way to prevent oral infection while taking this medication is to do which of the following as part of the self-administration techniques? A. Chew a hard candy before the first puff of medication. B. Ask for a breath mint following the second puff of medication. C. Rinse the mouth with water before each puff of medication. D. Rinse the mouth with water following the second puff of medication.

D. Rinse the mouth with water following the second puff of medication. The patient should rinse the mouth with water following the second puff of medication to reduce the risk of fungal overgrowth and oral infection.

18. In the case of pulmonary embolus from deep vein thrombosis, which of the following actions should the nurse take first? A. Notify the physician. B. Administer a nitroglycerin tablet sublingually. C. Conduct a thorough assessment of the chest pain. D. Sit the patient up in bed as tolerated and apply oxygen.

D. Sit the patient up in bed as tolerated and apply oxygen.The patient's clinical picture is consistent with pulmonary embolus, and the first action the nurse takes should be to assist the patient. For this reason, the nurse should sit the patient up as tolerated and apply oxygen before notifying the physician.

12. After admitting a patient to the medical unit with a diagnosis of pneumonia, the nurse will verify that which of the following physician orders have been completed before administering a dose of cefotetan (Cefotan) to the patient? A. Serum laboratory studies ordered for AM B. Pulmonary function evaluation C. Orthostatic blood pressures D. Sputum culture and sensitivity

D. Sputum culture and sensitivityThe nurse should ensure that the sputum for culture and sensitivity was sent to the laboratory before administering the cefotetan. It is important that the organisms are correctly identified (by the culture) before their numbers are affected by the antibiotic; the test will also determine whether the proper antibiotic has been ordered (sensitivity testing). Although antibiotic administration should not be unduly delayed while waiting for the patient to expectorate sputum, all of the other options will not be affected by the administration of antibiotics.

3 causes of anemia?

DIET LACKING: * Iron * Folate * Vitamin B12

The nurse is reviewing lab data of the following clients. It would be a priority for the nurse follow up with the primary health care provider if a client with

DM 1 and A1C level of 12%

The nurse assesses a patient with shortness of breath for evidence of long-standing hypoxemia by inspecting: A. Chest excursion B. Spinal curvatures C. The respiratory pattern D. The fingernail and its base

D. The fingernail and its base Clubbing, a sign of long-standing hypoxemia, is evidenced by an increase in the angle between the base of the nail and the fingernail to 180 degrees or more, usually accompanied by an increase in the depth, bulk, and sponginess of the end of the finger.

25. The nurse identifies the nursing diagnosis of activity intolerance for a patient with asthma. The nurse assesses for which of the following etiologic factor for this nursing diagnosis in patients with asthma? A. Anxiety and restlessness B. Effects of medications C. Fear of suffocation D. Work of breathing

D. Work of breathingWhen the patient does not have sufficient gas exchange to engage in activity, the etiologic factor is often the work of breathing. When patients with asthma do not have effective respirations, they use all available energy to breathe and have little left over for purposeful activity.

21. To prevent atelectasis in an 82-year-old patient with a hip fracture, a nurse should A. supply oxygen. B. suction the upper airway. C. ambulate the patient frequently. D. assist the patient with aggressive coughing and deep breathing.

D. assist the patient with aggressive coughing and deep breathing. Decreased mobility after surgery in older adults creates the possibility of fluid buildup and retention in lung tissue. One of the primary goals of nursing intervention is to prevent atelectasis in a high-risk patient. Aggressive coughing and deep breathing can prevent atelectasis in the postoperative patient.

What should you assess for in a patient diagnosed with SIADH?

Daily weights and I&O

4. After a posterior nasal pack is inserted by a physician, the patient is very anxious and states, "I don't feel like I'm breathing right." The immediate intervention the nurse should initiate is to A. monitor ABGs. B. reassure the patient that this is normal discomfort. C. cut the pack strings and pull the packing out with a hemostat. D. direct a flashlight into the patient's mouth and inspect the oral cavity.

D. direct a flashlight into the patient's mouth and inspect the oral cavity. The nurse should inspect the oral cavity for the presence of blood, soft palate necrosis, and proper placement of the posterior plug. If the posterior plug is visible, the physician should be notified for readjustment of the packing. Reassurance, cutting the strings, and ABGs are not top priority interventions. The nurse needs further data before intervening.

10. Anticoagulant therapy is used in the treatment of thromboembolic disease because anticoagulants can A. dissolve the thrombi. B. decrease blood viscosity. C. prevent absorption of vitamin K. D. inhibit the synthesis of clotting factors.

D. inhibit the synthesis of clotting factors. Anticoagulant therapy is based on the premise that the initiation or extension of thrombi can be prevented by inhibiting the synthesis of clotting factors or by accelerating their inactivation. The anticoagulants heparin and warfarin do not induce thrombolysis but effectively prevent clot extension.

Following TURP, hematuria should subside by what postoperative day?

Day 4.

31. A patient's ABGs include a PaO2 of 88 mm Hg and a PaCO2 of 38 mm Hg and mixed venous blood gases include a PvO2 of 40 mm Hg and PvCO2 of 46 mm Hg. These findings indicate that the patient has A. impaired cardiac output. B. unstable hemodynamics. C. inadequate delivery of oxygen to the tissues. D. normal capillary oxygen-carbon dioxide exchange.

D. normal capillary oxygen-carbon dioxide exchange. Normal venous blood gas values reflect the normal uptake of oxygen from arterial blood and the release of carbon dioxide from cells into the blood, resulting in a much lower PaO2 and an increased PaCO2. The pH is also decreased in mixed venous blood gases because of the higher PvCO2. Normal mixed venous blood gases also have much lower PvO2 and SvO2 than arterial blood bases. Mixed venous blood gases are used when patients are hemodynamically unstable to evaluate the amount of oxygen delivered to the tissue and the amount of oxygen consumed by the tissues.

7. The most appropriate position to assist a patient with chronic obstructive pulmonary disease (COPD) who is having difficulty breathing would be a A. high Fowler's position without a pillow behind the head. B. semi-Fowler's position with a single pillow behind the head. C. right side-lying position with the head of the bed at 45 degrees' elevation. D. sitting upright and forward position with arms supported on an over-the-bed table.

D. sitting upright and forward position with arms supported on an over-the-bed table.Sitting upright and leaning forward with arms supported on an over-the-bed table would be of most help to this patient, because it allows for expansion of the thoracic cage in all four directions (front, back, and two sides).

15. When administering oxygen to a patient with COPD with the potential for carbon dioxide narcosis, the nurse should A. never administer oxygen at a rate of more than 2 L/min. B. monitor the patient's use of oxygen to detect oxygen dependency. C. monitor the patient for symptoms of oxygen toxicity, such as paresthesias. D. use ABGs as a guide to determine what FIO2 level meets the patient's needs.

D. use ABGs as a guide to determine what FIO2 level meets the patient's needs. It is critical to start oxygen at low flow rates and then use ABGs as a guide to determine what FIO2 level is sufficient and can be tolerated.

Med surg

Defib at 200 joules. V tach converting to V fib After confirming ventricular fibrillation, rapid defibrillation is critical in reestablishing cardiac output and preserving vital organ function. After CPR is initiated and defibrillation attempted, airway intubation and IV access are indicated for successful resuscitation.

What signs and symptoms mimic stroke signs and symptoms when doing a health history?

Dementia

Duchene Muscular dystrophy

Develop an active ROM exercise. DMD is a genetic degenerative muscular disease that results in muscle wasting, immobility, and eventual death. The initial plan of care should include active exercises to maintain muscle strength, which can prolong independence

#1 cause of CRF?

Diabetes (more than 30% of dialysis patients are diabetic

Disorder of the pituitary gland that causes a deficiency of ADH hormone?

Diabetes Insipidus

Clinical Management for strokes

Diagnostic procedures, medication therapy, surgical, collaborative care, client education

Hydroureter

Dialation of the renal pelvis

Hydronephrosis

Dialation or enlargement of the renal pelvis and calyces

A 77-year-old female client is admitted to the hospital. She is confused and has had no appetite for several days. She has been nauseated and vomited several times prior to admission. She is currently complaining of a headache. Her pulse rate is 43 beats/min. The nurse is most concerned about the client's history related to what medication?

Digitalis (Lanoxin)

empyema

an accumulation of purulent exudates in a body cavity, especially the pleural space, as a result of bacterial infection, such as pleurisy or tuberculosis.

An adult resident in a long-term care facility is diagnosed with hepatitis B. Which intervention should the nurse implement with the staff caring for this client?

Discuss the importance of all employees starting the hepatitis B vaccine series.

Health History for Stroke patients

Do you have hx of strokes? Medications? Smoker? Diabetes? Hypertension? Exercise? Obesity? Heart disease?

The nurse is caring for a client who has a new colostomy. The colostomy stoma is red moist and raised. The nurse should

Document as normal finding

The home health nurse is assessing a male client being treated for Parkinson disease with levodopa-carbidopa (Sinemet). The nurse observes that he does not demonstrate any apparent emotion when speaking and rarely blinks. Which intervention should the nurse implement?

Document the presence of these assessment findings.

What med is given for BP drops in dialysis?

Dopamine

Medical Management for Acute phase of a stroke

Elevate head of bed, ABC's (airway clearance), continuous cardio monitoring and neurologic assessment, I&O, renal function, pt will be in ICU, watch and wait and hope little damage is done - use holistic nursing procedure

If a patient experiences sudden shortness of breath, what should the nurse do?

Elevate the head of the bed to open the airway

A 55-year-old male client is admitted to the coronary care unit having suffered an acute myocardial infarction (MI). Within 24 hours of the occurrence, the nurse can expect to find which systemic sign?

Elevated CM-MB level

Pediatrics

Encourage family members to bring foods from home. Encourageing the parents to bring familiar foods from home that the child likes should increase the childs likelihood to eat

A risk factor for multiple fractures, long bone fracture, & pelvis fracture?

Fat embolism

Maternity An infant who is delivered at 32 weeks gestation arrives in the nursery intubated. After the infant is placed under a radiant warmer with prescribed ventilator settings, the nurse applies a cardiorespiratory monitor and pulse oximeter, which indicates an O2 sat of 80%. What action should the nurse implement first?

Ensure patency of the endotracheal tube. The infants oxygenation is crucial, so airway patency should be implemented first to ensure the prescribed rate of oxygen administration to maintain the infants saturation above 92% a. Further assessment and intervention are warranted to determine the infants status before notifying the parents. B. Before notifying the Dr. The endotracheal tube should be suctioned to ensure patency. C. Continue with the admission assessment; To prevent futher oxygen de saturation, other tasks of the admission assessment should be delayed until the infant is stabilized.

The nurse is providing care for a client who has had a cervical cord injury. Following reduction of the cervical fracture, a halo vest is placed to maintain realignment of the spinal canal. What intervention is needed to ensure client safety while the halo vest is in place?

Ensure that a set of wrenches are kept in close proximity

Hep A

Ensure that all enrolled children have been immunized for Hep A. The CDC recommended immunization schedule for children includes Hep A (HAV), so follow up of enrolled childrens immunization status with HAV or human immune gamma globulin should be implemented.

Free radicals and destruction of a cell membrane

Enzyme release from lysosomes. Oxidative damage to cells is thought to be a causative factor in disease and aging. If free radicals bind to polyunsaturated fatty acids found in lysosome membrane, the lysosome, nicknamed suicide bags, leaks its protein catalytic enzymes intracellularly and the cell is destroyed.

Which assessment finding would confirm the specific location of an enteral feeding tube?

Examination of portable radiograph taken after the tube was inserted

S/S of Hypothyroidism

Fatigue, Mental sluggishness, Hypothermia, Dry flaky skin and thinning nails, cold intolerance, weight gane

Which assessment finding in a client with an acute small bowel obstruction requires the most immediate intervention by the nurse?

Fever of 102° F

Symptoms of Hemorrhagic Stroke

Exploding headache, decreased level of consciousness, numbness/weakness on one side, vomitting, stiff neck (nochal rigidity - which is also associated with Meningitis)

EOEM

Extraocular eye movement

FHR that is bad

FHR that is consistent 132 no variables

True or False: Synthroid can be given at different times each day.

False

A client is being discharged following radioactive seed implantation for prostate cancer. What is the most important information for the nurse to provide this client's family?

Follow exposure precautions.

Physical Assessment for Strokes

General appearance, Vital signs, Neurological Assessment (level of consciousness, sluggish pupilary reactions, motor and sensory dysfunction, cranial nerve deficits, speech difficulties and visual disturbance, headache and nuchal rigidity or other neuro deficits)

NG feed what action needs intervention by the nurse

Give patient all meds at the same time and flushing with 5 ML of water

Adrenal Cortex secretes what hormones?

Glucocorticoids, aldosterone (increase blood sugar, stimulate digestion, regulate NA and K)

In assessing a client for complications of total parenteral nutrition, it is most important for the nurse to monitor which lab value regularly?

Glucose

#2 cause of CRF?

HTN

Modifiable risk factors for strokes

HTN (primary risk factor), cardiovascular disease (pts on anticoagulants), obesity, Diabetes, Smoking (makes blood vessels weak), ETOH, illicit drug use, hyperlipidemia (plaque in vessel walls can break off), physical inactivity (sedentary lifestyle), oral contraceptive use (increases risk for clots), A-fib (irregular heartbeat)

The nurse has become aware of the following client situations. The nurse should first assess the client who

Had an ABD hysterectomy yesterday and is reporting calf pain

The nurse on an orthopedic unit has become aware of the following situations. It would be a priority for the nurse to follow up if a client who

Had an ORIF of a fracture femur 12 hours ago has developed a small rash on the cheat and neck.

The nurse is made aware for the following situations. The nurse should first check the client who

Has a history of partial seizures and is sitting in the bed picking at the clothing and smacking lips

Change in sleep patterns that occur in the older adult

Has decline in stage 4 sllep

The nurse is admitting a client with major depression. It would be a priority for the nurse to

Have the clients possessions searched for sharps

A male client with arterial peripheral vascular disease (PVD) complains of pain in his feet. Which instruction should the nurse give to the unlicensed assistive personnel (UAP) to quickly relieve the client's pain?

Help the client to dangle his legs.

An 81-year-old male client has emphysema. He lives at home with his cat and manages self-care with no difficulty. When making a home visit, the nurse notices that this client's tongue is somewhat cracked and his eyeballs appear sunken into his head. What nursing intervention is indicated?

Help the client to determine ways to increase his fluid intake.

What is the cardinal sign of renal trauma?

Hematuria, seen in 80% of cases

S/S of Thyroid storm?

High Fever, tachycardia, nausea, vomiting, tremors, agitation, psychosis

What kind of diet do you want for ARF?

High calorie, low protein

The nurse is administering a nystatin suspension (Mycostatin) for stomatitis. Which instruction will the nurse provide to the client when administering this medication?

Hold the medication in the mouth for a few minutes before swallowing it.

Hyperglycemia

Hot & dry, sugar is high.

Collateral Blood Flow

How hard the smaller blood vessels are working. This will help lessen the damage because it will provide perfustion.

A female client with diabetes mellitus type 2 has a plantar foot ulcer. When developing a teaching plan regarding foot care, what information should the nurse obtain first from this client?

How the client examines her feet

What does the Creatinine Clearance show?

How well the kidneys are functioning, the most accurate renal test

What is most important in the diuretic phase of acute renal failure?

Hydration! Patients can die in the this phase if they aren't hydrated

Post Renal obstruction

Hydroureter, vesicoureteral reflux, and hydronephrosis result from post renal obstruction which can consequently result in chronic pyelonephritis and renal atrophy.

What is Pheochromocytoma?

Hyperactivity of the adrenal medulla - or always in fight or flight because of too much adrenaline

What does a positive Chvostek's sign indicate?

Hypocalcemia

Beta blockers promote what in patients with diabetes?

Hypoglycemia (especially in elderly)

Delusional

I don't talk about things like that anymore. When the client states she doesn't want to talk about things like that anymore she is likely less delusional, because when a client begins to question the delusional belief or stops talking about it, the client is becoming less delusional.

Evidence base practice

Identification of practice problem Review of published research Implemetation of data gathering methods and data evaluation Identify and develop plan for application of research findings.

NGT

If head and neck injuries are suspected, a client with a GCS of 6 who demonstrates motor flexion in response to painful stimuli requires airway maintanance without risk of compromise to spinal cord function. Nasal intubation using a NGT is the airway of choice for a client with suspected spinal cord injury because less cervical spine manipulation is needed during insertion.

Before administering ampicillin check what?

If the child has ever taken before and had an allergic reaction

The nurse is receiving report from PACU about a client with a Penrose drain who is to be admitted to the surgical nursing unit. Before choosing a room for this client, which information is most important for the nurse to obtain?

If the client's wound is infected

A client is placed on a mechanical ventilator following a cerebral hemorrhage, and vecuronium bromide (Norcuron) 0.04 mg/kg every 12 hours IV is prescribed. What is the priority nursing diagnosis for this client?

Impaired communication related to paralysis of skeletal muscles

Goals for planning care of patient with hemorrhagic stroke

Improved cerebral tissue perfusion, relief of sensory and perceptual deprivation, relief of anxiety, absence of complications

Nursing goals for patients after strokes

Improved mobility, avoidance of shoulder pain, achievement of self care, relief of sensory and perceptual deprivation, prevention of aspiration, continence of bowel and bladder

Renal Carcinoma Stage 1

In capsule, 65% survival rate

Renal Carcinoma Stage 2

In fat, 40% survival rate

Renal Carcinoma Stage 4

In lung, heart, liver, bone, rarely have 5 year survival rate

TSH levels in hypothyroidism are increased or decreased?

Increased (body is trying to compensate)

Which assessment finding indicates that the expected outcome of administering donepezil (Aricept) to a client with Alzheimer disease has been accomplished?

Increased ability to solve simple problems

The nurse is assessing a 75-year-old male client for symptoms of hyperglycemia. Which symptom of hyperglycemia is an older adult most likely to exhibit?

Infection

Which condition should the nurse anticipate as a potential problem in a female client with a neurogenic bladder?

Infection

A family member was taught to suction a client's tracheostomy prior to the client's discharge from the hospital. Which observation by the nurse indicates that the family member is capable of correctly performing the suctioning technique?

Inserts the catheter until resistance or coughing occurs

The nurse is precepting a new hire nurse who is caring for a client receiving a continuous NG feeding. The nurse should intervene if

Installs 30cc of NS into the feeding tube while auscultating the stomach for bowel sounds

Parietal Lobe

Intellect, Sense or touch, differentiation of size, shape and color, spatial perception, visual perception, muscle tone, strength & sensation

Psy

Interacting with a client with alzheimers disease. Encourage verbal and nonverbal communication Maintain a calm demeanor during all interactions. Alzhemiers causes the client to experience cognitive deficits and memory impairment, so frequent communication and a calm affect should maintained with the client.

What is the most common cause of ARF?

Intrarenal or Acute tubular necrosis

The nurse from the pediatric unit has been temp assigned to the ED. It would be most appropriate to assign the nurse to the client who

Is HIV + reporting nausea and vomiting

The nurse has become aware of the following client situations. It would be a priority for the nurse to intervene if a client

Is being transported to have a MRI test and has an attach pulse oximeter

Four clients recently returned to the unit following invasive diagnostic testing. The nurse should immediately intervene if one of the clients:

Is observed flexing and extending the legs two hours post cardiac cath

The nurse has become aware of the following client situations. It would be priority for the nurse to follow up if a client who

Is scheduled for a myleogram in 4 hours and states "I can not drink and liquids until after the procedure is finished."

Negative Pressure Room

Isolation room used for clients with TB

Where is a transplanted kidney placed?

It is attached to the iliac artery

What religion might want mirrors covered after death of a patient?

Jewish

What is the correct location for placement of the hands for manual chest compressions during cardiopulmonary resuscitation (CPR) on the adult client?

Just above the xiphoid process on the lower third of the sternum

A client with ESRD is schedule for hemodialysis in one hour, the nurse should notify physician is the client has

K of 6.8

Instructions for a patient with a chest tube who has orders for ambulation?

Keep the chest tube system low, below the chest

S/S of DKA

Ketones in urine, nausea & vomiting, fruity breath, dehydration

HHNS

Lack of effective insulin blood sugar above 1000, no ketones are spilled

Nephrotic Syndrome: Interventions

MONITOR: * Vital signs * I & O * Daily weights. * Urine for specific gravity & protein. * Edema. * Abdominal girth daily. NUTRITION * A regular-protein diet, WITHOUT added salt may be prescribed if the child is in remission. * Sodium is restricted during periods of massive edema (fluids may also be restricted).

How to differeniate between Myasthenic Crisis & Cholinergic Crisis?

MYASTHENIC CRISIS: * Tensilon is administered & strength improves, the client needs more medication. CHOLINERGIC CRISIS: * Tensilon is administered & weakness is more severe, the client is overmedicated; * Administer atropine sulfate, the antidote, as prescribed.

A 43-year-old homeless, malnourished female client with a history of alcoholism is transferred to the ICU. She is placed on telemetry, and the rhythm strip shown is obtained. The nurse palpates a heart rate of 160 beats/min, and the client's blood pressure is 90/54. Based on these findings, which IV medication should the nurse administer?

Magnesium sulfate

A client in the emergency department is bleeding profusely from a gunshot wound to the abdomen. In what position should the nurse immediately place the client to promote maintenance of the client's blood pressure above a systolic pressure of 90 mm Hg?

Maintain the client in a supine position to reduce diaphragmatic pressure and visualize the wound

An antacid (Maalox) is prescribed for a client with peptic ulcer disease. What is the therapeutic action of this medication which is effective in treating the client's ulcer?

Maintenance of a gastric pH of 3.5 or above

What gender does renal cancer affect more?

Males

Use caution when giving positive inotropes because of the potential risk for increasing myocardial oxygen consumption and further decreasing cardiac output.....

Monitor the patient frequently, paying particular attention to the development of chest pain.

Neuroprotectants

Medications hat protect the brain from secondary injury

Temporal Lobe

Memory, Hearing, Understanding Language

Slef administer a sub q injection. Help ensure sterility of the procedure.

Method used to aspirate medication from a vial. To maintain sterility of the procedure, the most important factor to include in the teaching plan is how to manipulate the syringe parts so that the medication maintains sterility during the preparation and administration.

Which abnormal lab finding indicates that a client with diabetes needs further evaluation for diabetic nephropathy?

Microalbuminuria

Transient Ischemic Attack

Mini Stroke, the results will wear off in 24 hours, it is a temporary impairment of blood flow. It needs to be addressed to prevent a major stroke.

Cushing syndrome results from a hypersecretion of glucocorticoids in the adrenal cortex. Based on the clinical manifestations of Cushing syndrome, which nursing intervention would be appropriate for a client who is newly diagnosed with Cushing syndrome?

Monitor blood glucose levels daily.

Nursing management for renal transplant?

Monitor for graft rejection, cardiopulmonary complications

During report, the nurse learns that a client with tumor lysis syndrome is receiving an IV infusion containing insulin. Which assessment should the nurse complete first?

Monitor the client's serum potassium and blood glucose.

A severe thunderstorm has moved into a small community, and the tornado warning alarm has been activated at the local hospital. Which action should the charge nurse in the surgical department implement first?

Move clients and visitors into the hallways and close all doors to client rooms.

Mucomyst

Mucomyst is the antidote for acute Tylenol poisoning and is the treatment of choice for an overdose

What religion may have the body faced to the east at death?

Muslim

What disease is associated with Hypothyroidism?

Myxedema

The nurse is caring for a client who has been DX with RA. The nurse should anticipate the PT to be prescribed

NSAID

Is BUN increase specific for renal?

No

O2 sat

Normal 96-100

Hemoglobin

Normal for males 14-18

Bronchospasm

Notify resp. therapy immediately for PRN bronchodilator treatment is indicated for a bronchospasm, which typically produces wheezing or musical adventitious lung sounds.

In assessing a client with an arteriovenous shunt who is scheduled for dialysis today, the nurse notes the absence of either a thrill or a bruit at the shunt site. What action should the nurse take?

Notify the healthcare provider of the findings.

During a health fair, a male client with emphysema tells the nurse that he fatigues easily. Assessment reveals marked clubbing of the fingernails and an increased anteroposterior chest diameter. Which instruction is best to provide the client?

Pace your activities and schedule rest periods

Symptoms of Ischemic stroke

Numbness of face, arm, leg (especially on one side). Confusion (ask family about changes), speech, seeing double or blurry, visual disturbances (have pt. read something), trouble walking, perceptual problems (have pt. reach for something)

In assessing an older client with dementia for sundowning syndrome, what assessment technique is best for the nurse to use?

Observe for tiredness at the end of the day

The nurse assesses a postoperative client. Oxygen is being administered at 2 L/min and a saline lock is in place. Assessment shows cool, pale, moist skin. The client is very restless and has scant urine in the urinary drainage bag. What intervention should the nurse implement first?

Obtain IV fluids for infusion per protocol.

The nurse should intervene if the nurse suspects a staff member

Obtain a client consent prior to receiving Ativan

If a patient has dyspnea, cyanosis, tachycardia and hypotension what is the priority nursing intervention?

Open the airway, prepare patient for intubation

The nurse is observing an unlicensed assistive personnel (UAP) who is performing morning care for a bedfast client with Huntington disease. Which care measure is most important for the nurse to supervise?

Oral care

A 62-year-old woman who lives alone tripped on a rug in her home and fractured her hip. Which predisposing factor most likely contributed to the fracture in the proximal end of her femur?

Osteoporosis resulting from declining hormone levels

The nurse is preparing a class for parents of children DX with sickle cell. The nurse should instruct the parents that children should avoid

Other children who are sick

ABG levels

PH 7.35 - 745 PCO2 45-35 HCO3 22-28

Left Hemispheric

Paralysis or weakness on right side of the body, right visual field deficit, right side neglect, aphasia (expressive, receptive, or global), altered intellectual ability, slow, cautious behavior, Memory deficits

A 63-year-old client with type 2 diabetes mellitus is admitted for treatment of an ulcer on the heel of the left foot that has not healed with conventional wound care. The nurse observes that the entire left foot is darker in color than the right foot. Which additional symptom should the nurse expect to find?

Pedal pulses will be weak or absent in the left foot.

A client with alcohol-related liver disease is admitted to the unit. Which prescription should the nurse question as possibly inappropriate for this client?

Pentobarbital (Nembutal sodium) 50 mg at bedtime for rest

A client who is being monitored with telemetry has a pattern of uncontrolled atrial fibrillation with a rapid ventricular response. Based on this finding, the nurse anticipates administering which treatment?

Perform synchronized cardioversion

The Nurse and the nurse assistant are caring for a group of clients. Which of the following client care activity should the nurse assign to the nursing assistant

Placing a AKA patient 24 hours post op in prone position Assisting a client who had a colon resection 36 hours ago to ambulate

In assessing a client diagnosed with primary aldosteronism, the nurse expects the laboratory test results to indicate a decreased serum level of which substance?

Potassium

S/S of Pheochromocytoma?

Pounding headache and elevated BP

During assessment of a client in the intensive care unit, the nurse notes that the client's breath sounds are clear upon auscultation, but jugular vein distention and muffled heart sounds are present. Which intervention should the nurse implement?

Prepare the client for a pericardial tap.

Causes of ARF?

Prerenal, Intrarenal and Postrenal

The nurse knows that clients taking diuretics must be assessed for the development of hypokalemia, and that hypokalemia will create changes in the client's normal ECG tracing. Which ECG change would be an expected finding in the client with hypokalemia?

Presence of a U wave

The nurse is planning care for a client with diabetes mellitus who has gangrene of the toes to the mid-foot. Which goal should be included in this client's plan of care?

Prevent infection

Medical management of ARF

Prevention, hydration, adequate diuresis, watch labwork, I&O, weights, maintain fluid and electrolyte balance, replace renal function (dialysis), prevent infection

Chronic Renal failure

Progressive and irreversible reduction of functional renal tissue

Adjunct Therapy

Promotes a sense of well being including prednisone and provides adequate oxygenation (albuterol and Oxygen) work to supplement morphine in easing a clients discomfort, associated with SOB, and anxiety during terminal illness.

What position can the patient be placed in to help ventilation in ARDS?

Prone

Post renal cause of ARF?

Prostate disease, obstruction (stones), pelvic trauma, surgical accidents, spinal cord injury

Which consideration is most important when the nurse is assigning a room for a client being admitted with progressive systemic sclerosis (scleroderma)?

Provide a room that can be kept warm.

The nurse is preparing a staff presentation on legal and ethical issues in nursing. The nurse would be correct to include which of the following examples.

Putting a client in a geriatric chair with the lap tray in front of the client in the day room to watch TV... Example of false imprisonment

The nurse is developing a care plan for a client who is the manic phase of bipolar disorder. Which intervention should the nurse include in the plan of care

Provide the client with finger foods

One side neglect

Pt completely forgets that one side is even there, may startle pt. when they wake up ans see that side is even there.

What does ARDS lead to if not treated properly?

Pulmonary Edema

The nurse is caring for several clients who have been prescribed diuretics. The nurse should teach about increasing the consumption of citrus fruit and bananas to the client taking

Torsemide (demedex)

Patient that has ALS with dyspnea and SOB what do u do first

Raise HOB 45 degrees

Oral care for unconscious client

Raise bed to a comfortable working height Lower the side rail between the nurse and the client Position the client in a flat side lying position Place an emesis basin under the clients chin

A female practical nurse (PN) who is a valued employee tells the charge nurse in a long-term facility that she does not want to be assigned to one particular resident. She reports that the male client keeps insisting that she is his daughter and begs her to stay in his room. What is the best managerial decision?

Reassign the PN until the resident can be assessed more completely for reality orientation.

Client teaching for Diabetes Insipidus?

Recognizing the signs and symptoms of dehydration

Client teaching for Cushing's Syndrome?

Recognizing the signs and symptoms of infection and fluid retention

Post op

Record these findings in the clients record. wound edges are close together, there is no sign of redness, and there is a slight amount of bright red blood oozing from the incision.

A client with non-insulin-dependent diabetes mellitus (NIDDM) takes metformin (Glucophage) daily. The client is scheduled for major surgery requiring general anesthesia the next day. The nurse anticipates the use of which approach to best manage the client's diabetes while the client is NPO during the perioperative period?

Regular insulin subcutaneously per sliding scale

Interventions for stroke patients

Rehab is crucial, okay to reality orient (they are not dementia/Alzheimer's patients), education for patient and family, seizure precautions, keep sensor stimulation to a minimum, focus on the whole person, provide support and encouragement, listen to the patient

The nurse is caring for a postpartum clients who had vaginal deliveries with in the last eight hours. The nurse should first assess the client who

Reports swelling in her right calf

BP readings

Retake the BP in the same arm, deflating the cuff slowly. The nurse should first retake the BP in the right arm, deflating the cuff more slowly, because a low SBP and hig DBP measurement is often the result of deflating the cuff too rapidly.

The nurse is interviewing a client who is taking interferon-alfa-2a (Roferon-A) and ribavirin (Virazole) combination therapy for hepatitis C. The client reports experiencing overwhelming feelings of depression. What action should the nurse implement first?

Review the medication actions and interactions.

Materinity

Rh incompatibility occurs when a Rh negative mother responds to previous exposure to Rh positive erythrocytes, and these antibodies cross the placentral barrier to agglutinate erythrocytes of a Rh positive fetus. If a mother is Rh negative and a father is Rh positive and homozygous for the Rh factor, each Rh positive fetus is at risk for Rh incompatibility.

The most serious complication of Rheumatic Fever is....

Rheumatic Heart Disease. AFFECTS: * Aortic & Mitral valves.

Client teaching for Diabetes Mellitus?

Self monitoring for infection, skin breakdown, changes in peripheral circulation, poor wound healing, numbness in extremities, foot care

What position is a good position for a patient following a thyroidectomy?

Semi fowlers to promote good breathing

The nurse is assessing a client who presents with jaundice. Which assessment finding is the most significant indication that further follow-up is needed?

Serum amylase and lipase levels are twice their normal values

Pleuritis

Sharp, stabbing, thoracic pain that occurs during deep inspiration or coughing

Confrontation Technique

Sit facing the client and while look directly at the clients face, move an object inward from the periphery. Confrontation technique during a vision exam is used to determine peripheral vision, so allows the client to state when the object enters the nasal, temporal, superior, and inferior fields of vision.

Cushing's disease priority intervention?

Skin breakdown

The nurse is developing a plan of care for a client DX with fibromyalgia. Which nursing DX should the nurse include

Sleep pattern disturbance

hegars sign

Softening and compressibility of lower uterus

Nutrition

Special diets - thickened liquids, consult with speech therapist or nutritionist, have pt sit upright to eat, use chin tuck method

Biological practices are federally regulated for healthcare workers.

Standard precautions N-95 tuberculosis standard Blood-borne pathogen standard Resource Conservation and Recovery Act - requires labeling, storage, transportation, and disposal of biological waste according to fedral standards.

The nurse observes ventricular fibrillation on telemetry and upon entering the client's bathroom finds the client unconscious on the floor. What intervention should the nurse implement first?

Start cardiopulmonary resuscitation

Med surg

Start the first transfusion of blood. The hemoglobin of 6 (normal 14-18 in males) and the 82% O2 sat (normal 96-100%) indicates the client is hypoxic, so the first transfusion of blood should be started. H&H should be obtained after the client is transfused to evaluate its effectiveness

Renal lithiasis

Stone formation (calculi) in the kidney. CAUSES: * Family history. * Limited fluid intake. * Certain diseases (gout, UTI, & hyperparathyroid) * Certain medications * Limited activity * High protein & sodium diet. S/S: * Intense, colicky pain starting in your back or side, radiating to groin. * Bloody, cloudy or foul-smelling urine. * N/V * Persistent urge to urinate. * Fever & chills if an infection is present.

Ileoanal Reservoir (J pouch)

Stool is eventually expelled through the rectum. An J pouch is a surgically created ileoanal reservoir in the anal canal that preseves the rectal sphincter muscle, so that stool is eventually expelled through the rectum is the eventual result. To promote healing of the anastomosed parts of the colon, a temporary loop ostomy is created.

What are some expected findings when caring for a client with a cyanotic heart defect

Tachycardia, increased respiration rate, delayed growth and development

Name one factor that most commonly triggers a thyroid storm?

Stress

Mechanically ventilated.

Suction the clients endotracheal tube and ausculate following suction. Coarse, snoring sounds (rhonchi) heard over the upper airways are frequently produced by secretions partially blocking air passages and usually disappear after suctioning

Which description of symptoms is characteristic of a client diagnosed with trigeminal neuralgia (tic douloureux)?

Sudden, stabbing, severe pain over the lip and chin

A client diagnosed with angina pectoris complains of chest pain while ambulating in the hallway. Which action should the nurse implement first?

Support the client to a sitting position.

What can be given directly down into the ET tube to help with the exchange of gases?

Surfactant replacement

Teaching Plan

Teach the information that the client wants to learn first.

When preparing a male client who has had a total laryngectomy for discharge, what instruction would be most important for the nurse to include in the discharge teaching?

Tell the client to carry a medic alert card that explains his condition.

The nurse is assessing a 3 year old during a well baby visit. During the visit the boy states to his mother, "Mommy, I love you. I am going to marry you." The nurse should

Tell the mother that the statement is appropriate for his stage of development.

What might a patient with hypoparathyroidism present with?

Tetany and seizures

The nurse has attended a staff development conference on preparing clients for neurological diagnosis test. Which of the following statements, if made by the nurse would require a follow up

The EEG will require the client to be NPO for 12 hours before the test

The charge nurse is observing a LPN preforming patient care. Follow up would be required if

The LPN dons clean gloves on the dominant hand before trach suctioning

A 55-year-old male client has been admitted to the hospital with a medical diagnosis of chronic obstructive pulmonary disease (COPD). Which risk factor is the most significant in the development of this client's COPD?

The client smokes 1 to 2 packs of cigarettes per day.

The nurse is developing a teaching plan for a client with pulmonary TB who has been prescribed Rifampin, Isoniazid INH, pyrazinamide Tebrazid, and ethambutol Myambutol. The nurse should include in the teaching plan that

The combination of drugs prescribed is necessary to decrease the risk of drug resistance

During the oliguric phase of renal failure, why should protein be severely restricted?

Toxic metabolites that accumulate in the blood (Urea & Cr) are derived mainly from protein catabolism.

Collaborative Problems/Potential Complications of stroke

Vasospasm, seizures, hydrocephalus, rebleeding, hyponatremia

Med Surg

Venous Thrombosis s/s warm and reddened streaks along veins in the legs.

A 58-year-old client, who has no health problems, asks the nurse about taking the pneumococcal vaccine (Pneumovax). Which statement given by the nurse would offer the client accurate information about this vaccine?

The immunization is administered once to older adults or persons with a history of chronic illness."

The nurse has attended a staff development conference on cultural consideration for clients receiving hospice care. Which of the following statement id made by the nurse would require follow up

The family of a client of the Hindu faith may request that the client body be bathed after the clients death

Med Surg

The nurse instills an atropine phthalmic solution into both eyes for a client who is having a routine eye examination. Which side effects should the nurse tell the client to anticipate? A) Blurred vision Atropine opthalmic solution is used during eye exams to dialate the pupil (mydriasis) and paralyze the ciliary muscle (cycloplegic refraction) which prevents accomodation and causes blurred vision.

A client with cirrhosis states that his disease was caused by a blood transfusion. What information should the nurse obtain first to provide effective client teaching?

The year the blood transfusion was received

If a diabetic patient is taking a betablocker, what might need to be adjusted?

Their insulin

If a low pressure alarm goes off on a mechanical ventilator what needs to be done by the nurse?

There is a disconnection somewhere and the nurse needs to bag the patient with oxygen

A home health nurse knows that a 70-year-old male client who is convalescing at home following a hip replacement is at risk for developing decubitus ulcers. Which physical characteristic of aging contributes to such a risk?

Thinning of the skin with loss of elasticity

A risk factor for a hip fracture?

Thromboembolism

What gland is calcitonin (which keeps calcium in the bones) secreted by?

Thyroid

The nurse is caring for a client with a chest tube to water seal drainage that was inserted 10 days ago because of a ruptured bullae and pneumothorax. Which finding should the nurse report to the healthcare provider before the chest tube is removed?

Tidaling of water in water seal chamber

What is a nurses goal when treating a patient who had a stroke?

To re-perfuse quickly

HEP A

Transmitted via fecal oral route

What other sign other than Chvostek's sign indicated hypocalcemia?

Trousseau's sign

The nurse is caring for a client with bipolar disorder who has lithium prescribe. The nurse should suggest that the client have which of the following snacks

Tuna fish on saltine crackers (sodium)

A hospitalized male client is receiving nasogastric tube feedings via a small-bore tube and a continuous pump infusion. He begins to cough and produces a moderate amount of white sputum. Which action should the nurse take first?

Turn off the continuous feeding pump.

In evaluating the effects of lactulose (Cephulac), which outcome would indicate that the drug is performing as intended?

Two or three soft stools per day

High Risk for CKD secondary to DM

Type 1 DM and retinopathy and mild vision loss. Diabetic retinopathy and nephropathy are related to prolonged hyperglycemia and HTN which damage the microvasculature of the eyes and kidneys, so a client with DM1 and retinopathy is most likely to develop nephropathy and CKD

Cholelithiasis

Upper right abdominal pain that occurs after meals and radiates to the back or right shoulder

A nurse is assisting an 82-year-old client with ambulation and is concerned that the client may fall. What area contains the older person's center of gravity?

Upper torso

The nurse knows that normal lab values expected for an adult may vary in an older client. Which data would the nurse expect to find when reviewing laboratory values of an 80-year-old man who is in good health overall?

Urinalysis reveals slight protein in the urine and bacteriuria with pyuria.

What is Diabetes Insipidus treated with?

Vasopressin

receptive aphasia

Wernike's

Fecal impaction

When stool blocks the intestinal lumen. S/S: * Constipation * Rectal discomfort * Anorexia * N/V * Abdominal pain * Distention * Diarrhea (leaking around the impacted stool). * Urinary frequency INTERVENTION: * Enema * Digital removal.

What is refractory hypoxemia?

When the hypoxia does not respond to Oxygen therapy

Fundamentals

Which individual may legally sign an informed consent? B) a 16 year old mother for her newborn A mother who is younger than 18 years is an emancipated minor who may legally sign an informed consent for the care of her child. If a client questions a proposed treatment plan that requires informed consent, the nurse should notify the Dr. who is responsible for explaining the treatment plan before the client signs a consent form.

The nurse is assessing a male client with acute pancreatitis. Which finding requires the most immediate intervention by the nurse?

While the nurse is taking the client's blood pressure, he has a carpal spasm.

The nurse has become aware of the following client situations. It would be a priority for the nurse to intervene if a client

Who had transsphenoidal hypophysectomy twelve hours ago is drinking fluids through a straw

The nurse in the ED is caring for clients admitted following a rescue from the burning bus. You would first see the client

Who has soot on face and the nares and is coughing

Shock

Widespread, serious reduction of tissue perfusion (lack of o2 & nutrients) that if prolonged, leads to generalized impairment of cellular functioning.

surfactant

a lipoprotein that lowers the surface tension in the alveoli, reduces the amount of pressure needed to inflate the alveoli, and decreases the tendency of the alveoli to collapse.

community-acquired pneumonia

a lower respiratory tract infection of the lung parenchyma with onset in the community or during the first 2 days of hospitalization.

compliance

a measure of the ease of expansion of the lungs and thorax.

esophageal speech

a method of swallowing air, trapping it in the esophagus, and releasing it to create sound.

The nurse is reviewing medical prescription for the newly admitted clients. It would be a PRIORITY for the nurse to flow up with the physician if a client with

a K level of 4.0 and has kayexalate prescribed

pt with radium implant nurse should intervene if

a child is sitting on pt lap

asthma

a chronic inflammatory lung disease that results in airflow obstruction; characterized by recurring episodes of paroxysmal dyspnea, wheezing on expiration and/or inspiration caused by constriction of the bronchi, coughing, and viscous mucoid bronchial secretions.

pneumothorax

a collection of air or gas in the pleural space causing the lung to collapse.

pancreatic insufficiency

a condition characterized by inadequate production and secretion of pancreatic hormones or enzymes.

obstructive sleep apnea

a condition characterized by partial or complete upper airway obstruction during sleep, causing apnea and hypopnea.

chylothorax

a condition marked by lymphatic fluid in the pleural space caused by a leak in the thoracic duct.

O2 toxicity

a condition of oxygen overdosage caused by prolonged exposure to a high levels of oxygen; may inactivate pulmonary surfactant and lead to development of acute respiratory distress syndrome.

deviated septum

a deflection of the normally straight nasal septum.

wheezes

a form of rhonchus characterized by continuous high-pitched squeaking sound caused by rapid vibration of bronchial walls.

pneumoconiosis

a general term for lung diseases caused by inhalation and retention of dust particles.

tension pneumothorax

a pneumothorax with rapid accumulation of air in the pleural space causing severely high intrapleural pressures with resultant tension on the heart and great vessels.

lung abscess

a pus-containing lesion of the lung parenchyma that results in a cavity formed by necrosis of lung tissue.

alpha 1-antitrypsin

a serum protein produced by the liver normally found in the lungs that inhibits proteolytic enzymes of white cells from lysing lung tissue; genetic deficiency of this protein can cause emphysema.

status asthmaticus

a severe, life-threatening asthma attack that is refractory to usual treatment and places the patient at risk for developing respiratory failure.

thoracentesis

a surgical procedure done to remove fluid from the pleural space.

pursed-lip breathing

a technique of exhaling against pursed lips to prolong exhalation, preventing bronchiolar collapse and air trapping; done to increase expiratory airway pressure, improve oxygenation of the blood, and help prevent early airway closure.

pulmonary embolism

a thromboembolic occlusion of the pulmonary vasculature resulting from thrombi in the venous circulation or right side of the heart and from other sources, such as amniotic fluid, air, fat, bone marrow, and foreign IV material that travel as emboli until lodging in the pulmonary vessels.

pneumonia

an acute inflammation of the lungs, often caused by inhaled pneumococci of the species Streptococcus pneumoniae.

OB nurse

an adult who had a colon resection yesterday. An OB nurse is usually experienced in caring for abdominal surgical wounds (C-section) and IV infusions.

The nurse is assessing a child with coarctation of the aorta, which would be an assessment finding

absence of femoral pulse

Aneurysm precautions

absolute bed rest, elevate hob 30 degrees to promote venous drainage or lay bed flat to increase cerebral perfusion, avoid activity that increases BP or ICP, exhale through mouth when voiding or defecating to decrease strain, dim lights, no reading, no TV, no radio, prevent constipation, restrict visitors

hemothorax

accumulation of blood in the pleural space.

The PCP has prescribed Elavil - Amitriptyline 150 mg daily for a client with major depression. Name some nursing considerations

admin meds with meals instruct the client that medication may cause dry mouth medications may cause photosensitivity

metered-dose inhaler

aerosolized drug delivered in a specific amount by activating the inhaler or by inhaling.

Penumbra

affected area of the brain where the stroke happened, can be revitalized by administration of t-PA

absorption atelectasis

alveolar collapse that occurs when high concentrations of oxygen are given and oxygen replaces nitrogen in the alveoli; if airway obstruction occurs, the oxygen is absorbed into the bloodstream and the alveoli collapse.

pulmonary edema

an abnormal accumulation of fluid in the alveoli and interstitial spaces of the lungs caused most commonly by heart failure; an acute, life-threatening situation in which the lung alveoli become filled with serous or serosanguineous fluid caused most commonly by heart failure.

pleural effusion

an abnormal accumulation of fluid in the intrapleural spaces of the lungs.

hyperreactivity

an abnormal condition in which responses to stimuli are exaggerated.

Causes of Ischemic stroke

artery thromboses, cardiogenic emoblic, cryptoenic

Bowel and bladder control in stroke patients

assess and schedule voiding, may be on program to retrain, implement measures to prevent constipation (fiber, fluid, activity)

Nursing Management after dialysis

assessment, v/s, weight, assess access site, monitor for s/s of bleeding or infection, watch for signs of confusion, decreased level of consiousness, sudden severe headache, monitor lab results, administer meds

TB

close door to their room

Nursing management before dialysis

complete assessment, v/s, weight, assess the access site (listen for bruit on fistula), review lab results, check meds

early signs of ICP

confusion

rhonchi

continuous rumbling, snoring, or rattling sounds from obstruction of large airways with secretions; most prominent on expiration; change often evident after coughing or suctioning.

best foods for high protein

cottage cheese, yogurt, skim milk

pleural friction rub

creaking or grating sound from roughened, inflamed surfaces of the pleura rubbing together, evident during inspiration, expiration, or both and no change with coughing; usually uncomfortable, especially on deep inspiration.

The nurse is counseling a healthy 30-year-old female client regarding osteoporosis prevention. Which activity would be most beneficial in achieving the client's goal of osteoporosis prevention?

cross country skiing

pt admitted to the hospital with depression, what should the nurse determine first

do they have plans to harm self

The nurse should intervene is a staff member

discussing a client's DX with visiting family member

NANDA for eating disorder

decreased self esteem

Suicidal thoughts are a priority with what patient

depressed

Suicide observation may be needed during what phase of bipolar

depressed

DABDA

depression, anger, bargaining, denial, anger

Confabulation

describes the clients story that is made up to fill the gaps of memory when one is unable to remember something that might have happened.

Woods lamp

determines the presence of fungi

Abruptly stop TNP pt will become

diaphoretic

What is creatinine not affected by?

dietary intake or hydration

dysarthria

difficulty speaking

dysphagia

difficulty swallowing

diverticulosis diet

don't increase fiber with inflammation and avoid nuts or corn that can lodge in the pouches

pneumonia precautions

droplet

Myxedema is a type of hypothyroidism, what kind of swelling is associated with this disease if it is severe?

dry and waxy

dry powder inhaler

dry powdered drug delivered by inhalation.

Removing all liquids is not necessary for a client with what syndrome?

dumping

pulmonary hypertension

elevated pulmonary pressure resulting from an increase in pulmonary vascular resistance to blood flow through small arteries and arterioles.

adventitious sounds

extra breath sounds that are not normally heard, such as crackles, rhonchi, wheezes, and pleural friction rubs.

Adrenal glands produce what hormones?

epi, norepi, corticosteroids, aldosterone

hyperresponsiveness

excessive or exaggerated response to a stimulus; in asthma leads to bronchoconstriction in response to physical, chemical, or pharmacologic stimuli.

The nurse is caring for a 49 year old female client who have frequent vaginal yeast infections. The client is 35% over idea body weight. The client has had several DX blood test prescribed. It would be a priority for the nurse to review the result for an elevated

fasting blood glucose

patient with ABD injuries what to assess first

femoral pulses

Nursing management for ARF

fluid volume deficit or excess, imbalanced nutrition, impaired skin integrity, risk for infection, anxiety

Frequent loose stools & brisk reflexes in kids/babies may be a sign of what?

hyperthyroidism

cor pulmonale

hypertrophy of the right side of the heart, with or without heart failure, resulting from pulmonary hypertension.

Prerenal cause of ARF

hypovolemia, fluid shifts, gram negative sepsis, decreased C.O., Decreased peripheral vascular resistance, vascular obstruction

What other reasons would BUN be elevated?

hypovolemia, starvation, dietary, hydration, nephrotoxic, UTI's

Creatinine will do what when renal function is lost?

increase by 50%

flail chest

instability of the chest wall resulting from multiple rib fractures.

Causes of Hemorrhagic Stroke

intracerebral hemorrhage, cerebral aneurysm, arterovenous malformation

A client with Parkinson disease is taking carbidopa-levodopa (Sinemet). Which observation by the nurse would indicate that the desired outcome of the medication is being achieved?

lessing of tremors

Peritoneal dialysis

maintain themselves at home, used for clients with severe cardiovascular disease

UAP can

measure urine out put from foley

Generativity vs stagnation

middle age

What foods contain a lot of phosphorus?

milk and cheese

Acute tubular necrosis?

most common cause of intrinsic renal failure. abrupt damage b/c of decrease blood flow associated with shock. characterized by destruction of tubular epithelial cells

Hepatic encephalopathy

muscle tremors

2 year old expected finding

no bowel control during daytime hours

Sickle cell anemia teaching

no cold water (vasoconstriction), no contact sports, no kids with infections

aphasia

no speech

normocapnia

normal arterial carbon dioxide pressure (PaCO2 35 to 45 mm Hg).

S/S of hypocalcemia

numbness and tingling in face,neck and around mouth, respiratory wheezing

chronic bronchitis

obstructive pulmonary disease characterized by excessive production of mucus and chronic inflammatory changes in the bronchi, resulting in a cough with expectoration for at least 3 months of the year for more than 2 consecutive years.

Teaching instructions for a client with ulcerative colitis

occurs in Jewish men 30-50 years Prednisone may be prescribed for exacerbations Flagyl, Cipro, and commonly prescribed during acute exacerbations

It would be appropriate to assign which of these task to the CNA

one - on - one client observation for safety

A 25-year-old client was admitted yesterday after a motor vehicle collision. Neurodiagnostic studies showed a basal skull fracture in the middle fossa. Assessment on admission revealed both halo and Battle signs. Which new symptom indicates that the client is likely to be experiencing a common life-threatening complication associated with a basal skull fracture?

oral temp 102 F

organ donation

patient family can request for changes to be made

The nurse is teaching a group of parents about the expected growth and development of 3 year old children. The nurse should include that a 3 year old should

play in a group of 2-3 with one being the leader

Paranoid schizo pt make what meals?

pre packaged

Anything that stops perfussion or is volume related is what kind of cause of ARF?

prerenal

vibration

pressing on the chest with the flat of the hands while repeatedly tensing the hand and arm muscles to facilitate movement of secretions to larger airways.

Thrombolytic Therapy

process in which drugs, such as streptokinase (SK) or tissue-type plasminogen activator (tPA), are injected into a blood vessel to dissolve clots and restore blood flow, can be used only within first 3 hours of an Ischemic Stroke

The nurse on a psychiatric unit is caring for a client with a paranoid schizophrenia who has lost 15 pounds within the past three weeks. The client accuses the staff of trying to poison all of the clients on the unit. Which of the following nursing interventions would be a priority for the nurse to include in the clients plan of care

provide the client with pre package foods that the client likes

Aneurysm interventions

relieve sensory deprivation and anxiety, keep sensory stimulation to a minimum, implement reality orientation, provide teaching, seizure precautions, rehab

The charge nurse of a med-surg floor notice a nurse walking with an unsteady gait, slurred speech and a faint smell of alcohol on the breath immediately following a lunch break, The charge nurse priority action would be to

relieve the nurse of assigned clients

What might a patient with hyperparathyroidism present with?

renal calculi, nausea and vomiting, cardiac dysrythmias, musle and bone weakness

Industry vs. Inferiority age

school age

What is the patient susceptible to with lacerations?

sepsis

Fibromyalgia

severe exhaustion, pain, muscle stiffness, spasms with sensory changes

crackle

short, low-pitched sounds consisting of discontinuous bubbling caused by air passing through airway intermittently occluded by mucus, unstable bronchial wall, or fold of mucosa; evident on inspiration and, at times, expiration; similar sound to blowing through a straw under water.

dyspnea

shortness of breath; difficulty breathing that may be caused by certain heart conditions, strenuous exercise, or anxiety.

What does aging do to the kidneys?

shrinks them, creates urgency to void and incontinence, blood flow decreases by 10% per decade

dustus arteriosus

shunted blood from the pulmonary trunk to the aortic arch during fetal life

chadwicks sign

sign of early pregnancy in which the vaginal, cervical, and vulvar tissues develop a bluish color.

The nurse is assessing a 2 month old. The nurse should anticipate the infant should

smile socially

Risks for kidney cancer?

smoking, HTN, obesity, Exposure to lead (Painters)

Dong Quai herb is for

smooth muscle relaxant regulates menstrual periods

goodells sign

softening of the cervix

The nurse is caring for a client who has oxalate kidney stones. The nurse should teach the client to avoid

spinach and rhubarb

Diuretic phase of acute renal failure

starts when BUN and Creat get back to normal

Onset phase of acute renal failure

starts with the precipitating event and continues to the development of renal symptoms (labwork)

A male client has undergone insertion of a permanent pacemaker. When developing a discharge teaching plan, the nurse writes a goal of, "The client will verbalize symptoms of pacemaker failure." Which behavior indicates that the goal has been met? The client

states that changes in the pulse and feelings of dizziness are significant changes.

What is administered for rejection?

steroids

nuchal rigidity

stiff neck

What does the hypothalamus do?

stimulates another gland to release hormones

What is the most important nursing priority for a client who has been admitted for a possible kidney stone?

straining all urine

Renal transplant

surgical implantation of human kidney from a compatible donor

thoracotomy

surgical opening into the thoracic cavity.

tracheostomy

surgical opening into the trachea through which an indwelling tube may be inserted.

Signs of aspiration

tearing, red face, coughing, listen to lung sounds in stroke patients

Most common type of Peritoneal dialysis

tenckhoff

A PSA count should be obtained before performing a DRE because.....

the DRE can increase the PSA count.

What are paradoxical chest movements?

unequal chest movements

fremitus

vibration of the chest wall produced by vocalization.

tidal volume

volume of air exchanged with each breath.

What is SIADH?

water retention and decreased NA (diluted)

hemiparesis

weakness

The nurse is caring for a client with VRE wound infection. Which of the following actions would be appropriate for the nurse to take

wear a disposable gown and glove when changing clients dressing

When will you see Grey Turner's sign?

with renal trauma, especially minor and major lacerations

What group of people is most commonly affected by hypothyroidism?

women

Gonorrhea in men and women what is the difference?

women asymptomatic men urinary frequency and dysuria

Intimacy vs. isolation

young adult

Neutropenia: Pt Teaching

•Avoid crowds & other large gatherings of people who might be ill. •Do not share personal toilet articles, such as toothbrushes, toothpaste, washcloths, or deodorant sticks, with others. •If possible, bathe daily, using an antimicrobial soap. If total bathing is not possible, wash the armpits, groin, genitals, & anal area twice a day with an antimicrobial soap. •Clean your toothbrush at least weekly by either running it through the dishwasher or rinsing it in liquid laundry bleach (and then rinsing the bleach out with hot running water). •Wash your hands thoroughly with an antimicrobial soap before you eat & drink, after touching a pet, after shaking hands with anyone, as soon as you come home from any outing, & after using the toilet. •Avoid eating salads; raw fruits & vegetables; undercooked meat, fish & eggs; and pepper & paprika. •Wash dishes between use with hot, sudsy water, or use a dishwasher. •Do not drink water, milk, juice, or other cold liquids that have been standing at room temperature for longer than an hour. •Do not reuse cups and glasses without washing. •Do not change pet litter boxes. •Take your temperature at least once a day & whenever you do not feel well. •Report any of these signs or symptoms of infection to your physician immediately: •Temperature greater than 100° F. •Persistent cough (with or without sputum). •Pus or foul-smelling drainage from any open skin area or normal body opening. •Presence of a boil or abscess. •Urine that is cloudy or foul smelling or that causes burning on urination. •Take all prescribed drugs. •Wear clean disposable gloves underneath gardening gloves when working in the garden or with houseplants. •Wear a condom (if you are a man) when having sex. If you are a woman having sex with a male partner, ensure that he wears a condom.

Thrombocytopenia: Pt Teaching

•Use an electric shaver. •Use a soft-bristled toothbrush. •Do not have dental work performed without consulting your cancer health care provider. •Do not take aspirin or any aspirin-containing products. Read the label to be sure that the product does not contain aspirin or salicylates. •Do not participate in contact sports or any activity likely to result in your being bumped, scratched, or scraped. •If you are bumped, apply ice to the site for at least 1 hour. •Avoid hard foods that would scrape the inside of your mouth. •Eat warm, cool, or cold foods to avoid burning your mouth. •Check your skin & mouth daily for bruises, swelling, or areas with small reddish purple marks that may indicate bleeding. •Notify your cancer health care provider if you: •Are injured & persistent bleeding results. •Have excessive menstrual bleeding. •See blood in your urine or bowel movement. •Avoid anal intercourse. •Take a stool softener to prevent straining during a bowel movement. •Do not use enemas or rectal suppositories. •Avoid bending over at the waist. •Do not wear clothing or shoes that are tight or that rub. •Avoid blowing your nose or placing objects in your nose. If you must blow your nose, do so gently without blocking either nasal passage. •Avoid playing musical instruments that raise the pressure inside your head, such as brass wind instruments and woodwinds or reed instruments.


Related study sets

Chapter 4 Financial Services: Servings Plans and Payment Accounts

View Set

What is the long term effect of the Crusades?

View Set

CH 14: Preoperative Nursing Management

View Set

White Fang Comprehension Chapters 4 - 7

View Set

Psychology Ch.1: Psychological Foundations

View Set

BIOD 121 Nutrition Class Module 2

View Set

Chapter 14: We Respond to God through Prayer

View Set

The Neurological System (part 2)

View Set